Physiological Psychology and Psychopharmacology

Ace your homework & exams now with Quizwiz!

Sleepers typically pass through all five stages every (1) __________ minutes, but the pattern of sleep varies with age: In newborns, REM sleep precedes NREM sleep and takes up (2) __________ percent of the total sleep time, while, in adults, it accounts for only about (3) __________ percent of the sleep period.

1) 90 to 100 2) 50 3) 20

Damage to Broca's area produces (1) __________ aphasia. People with this disorder speak slowly and with difficulty and their speech is poorly (2) __________, but they usually have little or no difficulty (3) __________ language.

1) Broca's (expressive, motor, nonfluent) 2) articulated 3) comprehending (understanding)

The sedative-hypnotics are generalized (1) __________ depressants and include the barbiturates and anxiolytics. The effects of the sedative-hypnotics are synergistic, and they also produce cross- (2) __________. The barbiturates are now rarely prescribed because of their adverse effects. Their use causes a decrease in REM sleep, and abrupt cessation can produce and REM (3)__________ and nightmares.

1) CNS 2) tolerance 3) Rebound

(1)__________ aphasia results from damage to the arcuate fasciculus and involves (2) __________ (an inability to name a familiar object, attribution, or action), paraphasia, and impaired repetition.

1) Conduction 2) anomia

(1) __________ syndrome is often triggered by anxiety or other emotional state and may be confused with (2) __________ or certain respiratory disorders such as asthma.

1) Hyperventilation 2) coronary heart disease

According to (1) __________ theory, emotions reflect perceptions of bodily reactions ("I'm scared because my knees are shaking"), while (2) __________ theory proposes that emotion-provoking stimuli simultaneously produce a subjective emotional experience and bodily reactions.

1) James-Lange 2) Canaan-Baard

(1) __________, the "silent killer," is often asymptomatic but can lead to heart failure, kidney failure, or stroke. It tends to run in families and is more common in men and in (2) __________ and (3) __________..

1) Primary Hypertension 2) older adults 3) African Americans

(1) __________ (grand mal) seizures involve stiffening of the body followed by rhythmic shaking of the limbs, while (2) __________ (petit mal) seizures are characterized by a loss of consciousness without prominent motor symptoms.

1) Tonic-Clonic 2) Absence

(1) __________ aphasia is caused by lesions that isolate Broca's area and/or Wernicke's area from other regions of the brain, while (2) __________ aphasia is caused by widespread brain injury involving Broca's and Wenicke's areas as well as other areas in the left frontal, temporal and parietal lobes.

1) Transcortical 2) Global

According to the Young-Helmholtz (1) __________ theory of color vision, there are three types of color receptors (cones) that are each receptive to a different primary color - red, green, or blue. In contrast, Hering's (2) __________ theory postulates three types of bipolar receptors: red-green, yellow-blue, and white-black. Because color blindness is most often due to a sex-linked recessive trait, (3) __________ are at a higher risk for color blindness.

1) Trichromatic 2) Opponent Process 3) males

Damage to Wernicke's area results in (1) __________ aphasia. People with this form of aphasia speak rapidly and effortlessly but their speech is usually (2) __________, and they have trouble understanding written and spoken language.

1) Wernicke's (receptive, sensory, fluent) 2) devoid of content (meaningless)

Several terms are used to describe the effects of the psychoactive drugs: For example, (1) __________ produce effects similar to those produced by a neurotransmitter, while (2) __________ produce no activity on their own but reduce or block the effects of a neurotransmitter.

1) agonists 2) antagonists

Selye's general adaptation syndrome describes the stress response as involving three stages: (1) __________, (2) __________, and exhaustion. This response is mediated primarily by the adrenal and (3) __________glands.

1) alarm reaction 2) resistance 3) pituitary

Sleep is divided into five stages. During stage 1, (1) __________ waves are replaced by theta waves, and in stage 2, theta waves predominate but are interrupted by bursts of (2) __________ and K complexes. Stages 3 and 4 are characterized by large, slow (3) __________ waves. The fifth stage is marked by the presence of (4) __________. Because the sleeper is both in a deep state of sleep and physiologically active, this stage is also referred to as active or (5) __________ sleep.

1) alpha 2) sleep spindles 3) delta 4) rapid eye movements 5) paradoxical

Following bilateral medial temporal lobectomy to relieve severe epilepsy, H.M. was able to recall remote events but he exhibited severe (1) __________ amnesia and retrograde amnesia for (2) __________.

1) anterograde 2) recent

The term posttraumatic amnesia is usually used to refer to (1) __________ amnesia. When retrograde amnesia occurs, (2) __________ memories return first.

1) anterograde 2) remote

Carbamazepine is an (1) __________ drug that has been found to have similar effects on manic symptoms as lithium and may be more beneficial than lithium for people who experience (2) __________ or who have dysphoric mania.

1) anticonvulsant 2) frequent mood swings

Benzodiazepines are the most commonly-prescribed anxiolytic. They are associated with several adverse side effects including drowsiness, lethargy, slurred speech, and (1) __________ (lack of muscle coordination). The euphoric feelings produced by the benzodiazepines result in psychological dependence, and chronic use can result in the development of (2) __________ and physical dependence. Withdrawal can produce rebound (3) __________.

1) ataxia 2) tolerance 3) hyperexcitability

The temporal lobes contain the (1) __________ cortex and (2) __________ and are involved in the encoding, retrieval, and storage of long-term (3) __________ memories.

1) auditory 2) Wernicke's 3) declaritive

Huntington's disease is due to an (1) __________ dominant gene and is characterized by a combination of psychiatric, cognitive, and motor symptoms. It has been linked to degeneration of (2) __________-secreting cells and glutamate excitotoxity in certain structures of the basal ganglia.

1) autosomal 2) GABA

The (1) __________ are important in planning, organizing, and coordinating voluntary movements. These structures are involved in several disorders with prominent motor symptoms including Huntington's disease, (2) __________ disease, Tourette's Disorder, and ADHD.

1) basil ganglia 2) Parkinson's

Lithium is the drug-treatment-of-choice for "classic" (1) __________. To avoid toxicity when taking this drug, patients must be careful to avoid fluctuations in their (2) __________ intake and to avoid caffeine, alcohol, and other diuretics.

1) bipolar disorder 2) salt (sodium)

Traumatic Brain Injury (TBI) may be the result of either a closed- or open-head injury. A closed-head injury usually causes an alteration or loss of (1) __________ and some degree of anterograde and retrograde amnesia. Level of severity is often determined by considering the person's initial score on the (2) __________, duration of (3) __________, and duration of loss of consciousness.

1) consciousness 2) Glasgow Coma Scale (GCS) 3) posttraumatic amnesiaia

As described in the DSM-5, the diagnosis of Neurocognitive Disorder Due to Traumatic Brain Injury requires that symptoms meet the criteria for Major or Mild Neurocognitive Disorder plus evidence of a traumatic brain injury with loss of (1) __________, (2) __________, disorientation and confusion, and/or neurological signs.

1) consciousness 2) posttraumatic amnesia

For most sensory and motor functions, the cerebral cortex exhibits (1) __________ representation, which means that the left side of the brain controls the right side of the body and vice versa. While both hemispheres play a role in most behaviors, the (2) __________ (usually left) hemisphere is primarily responsible for language, (3) __________ memory, and (4) __________ emotions. The (5) __________ (usually right) hemisphere governs (6) __________ recognition, nonverbal memory, and (7) __________ emotions.

1) contralateral 2) dominant 3) verbal 4) positive 5) non-dominant 6)facial 7) negative

Most neurons consist of three distinct components: The branch-like (1) __________ receive information from other cells and conduct it toward the (2) __________, which integrates information received from the dendrites and passes it on to the (3) __________.

1) dendrites 2) soma (cell body) 3) axon

Parkinson's disease is due to the degeneration of (1) __________-containing cells in the substantia nigra and involves tremor at rest, (2) __________("cruel restlessness"), muscle rigidity, and (3) __________ (a reduction or absence of spontaneous movement.

1) dopamine 2) akathisia 3) akinesia

Damage to specific areas of the prefrontal cortex may produce one of three syndromes: Damage to the dorsolateral area results in dorsal convexity (1) __________ syndrome; damage to the orbitofrontal area produces orbitofrontal disinhibition syndrome, which is also known as (2) __________, and damage to the mediofrontal area produces mesial frontal apathetic syndrome, which is also referred to as (3) __________.

1) dysexecutive 2) pseudopsychopathy 3) psudodepression

Conduction of information within neurons is an (1) __________ process. With sufficient stimulation, the interior of the neuron becomes less negative and triggers an (2) __________, which operates on the (3) __________ principle. Communication between neurons usually involves the release of a (4) __________ by the presynaptic cell into the synaptic cleft.

1) electrical impulse 2) action potential 3) "all or nothing" 4) neurotransmitter

Electrical stimulation of the amygdala can produce a (1) __________ and/or rage response, while damage to the hypothalamus can result in rage or uncontrollable (2) __________.

1) fear 2) laughter

The pancreas releases insulin, and too much of this hormone results in (1) __________, which is characterized by hunger, dizziness, headaches, depression and confusion. Too little insulin can produce (2) __________.

1) hyperglycemia 2) diabetes mellitus

The primary secretion of the thyroid gland is thyroxine. Too much of this hormone produces (1) __________, which is characterized by speeded-up metabolism, (2) __________ body temperature, weight loss, agitation, and insomnia. In contrast, too little thyroxine result in (3) __________, which involves slowed metabolism, weight gain, lowered body temperature, depression, lethargy, and impaired (4) __________.

1) hyperthyroidism 2) elevated 3) hypothyroidism 4) memory and concentration

The (1) __________ governs a number of vital functions (e.g., hunger, thirst, temperature), integrates and coordinates emotional reactions and initiates the responses needed to maintain the body's internal (2) __________.

1) hypothalamus 2) homeostasis

The (1) __________ hemisphere governs positive emotions and damage to this area results in (2) __________. In contrast, the (3) __________ hemisphere mediates negative emotions, and lesions produce (4) __________, emotional lability and/or undue cheerfulness.

1) left (dominant) 2) catastrophic reactions 3) right (non-dominant) 4) indifference and apathy

At the neural level, (1) __________ in the hippocampus is believed to be important for long-term memory. There is also evidence that long-term memory depends on increased (2) __________ during the minutes or hours following training. When the synthesis of protein or (3) __________ is inhibited at the time of training, long-term memory is impaired.

1) long term potentiation (LTP) 2) protein synthesis 3) RNA

The hindbrain consists of the pons, the medulla, and the cerebellum. The (1) __________ regulates a number of vital functions and damage to this structure is often fatal. The (2) __________ is important for balance, posture, and coordination, and damage can produce (3) __________, which involves slurred speech, severe tremors, and a loss of balance.

1) medulla oblongata 2) cerebellum 3) ataxia

The SSRIs are particularly effective for (1) __________ depression. They are less cardiotoxic than the TCAs, are safer in (2) __________ and are less likely to produce (3) __________ impairments. The SSRIs exert their effects by blocking the reuptake of (4) __________.

1) melancholic 2) overdose 3) cognitive 4) seratonin

The limbic system is associated primarily with emotional activities. The amygdala attaches emotions to (1) __________, and damage to the amygdala and temporal lobes can produce (2) __________ syndrome, which is characterized by reduced fear and aggression, psychic blindness, and hypersexuality.

1) memories 2) Kluver-Bucy

Multiple sclerosis is a progressive disease of the nervous system that involves degeneration of the (1) __________ that surrounds nerve fibers in the brain and spinal cord. Initial systems include optic neuritis, fatigue, motor impairments, and (2) __________ abnormalities.

1) myelin 2) sensory

Most neurons have a single axon that divides into numerous branches (collaterals), and many axons are covered by a (1) __________ that is made up of (2) __________.

1) myelin sheath 2) glial cells

Amphetamines are currently used to treat (1) __________ and ADHD, while methylphenidate is used to treat the latter disorder. Common side effects of methylphenidate include decreased (2) __________ insomnia, and dysphoria. Higher doses of methylphenidate may cause (3) __________ suppression, but occasional (4) __________ can help minimized this effect. Methylphenidate is contraindicated for people with pre-existing motor tics or a family history of (5) __________.

1) narcolepsy 2) appetite 3) growth 4) "drug holidays" 5) Tourette's

Common symptoms of a stroke (cerebrovascular accident) include contralateral hemiplegia, contralateral (1) __________, and contralateral visual field loss. When the stroke involves the (2) __________ cerebral artery, symptoms may also include dysarthria, aphasia, or apraxia.

1) neglect 2) middle

The MAOIs are most effective for (1) __________ depressions. The most dangerous side effect is hypertensive crisis, which can occur when MAOI is taken in conjunction with certain drugs or foods containing (2) __________.

1) non-endogenous and atypical 2) tyramine

The antidepressants include TCAs, SSRIs, and MAOIs. The TCAs exert their effects by blocking the reuptake of (1) __________, serotonin, and/or dopamine. Their effects on the former support the (2) __________ hypothesis. Undesirable effects include (3) __________ effects (e.g., blurred vision, dry mouth, constipation), weight gain, drowsiness, and confusion. These drugs are also (4) __________ and can cause hypertension, extreme hypotension, and tachycardia.

1) norepinephrine 2) catecholamine 3) anticholinergic 4) cardiotoxic

The narcotic-analgesics have both sedative and analgesic effects. Research on the effects of these drugs led to the discovery of natural (1) __________ receptors in the spinal cord and brain. Chronic use results in tolerance and (2) __________ dependence.

1) opioid 2) psychological and physical

With regard to olfaction, the (1) __________ cortex is believed to be involved in the conscious perception of odors, while the (2) __________ plays an important role in olfactory memory.

1) orbitofrontal cortex 2) amygdala

Of the senses, (1) __________ is the only one that is not linked to a single type of stimulus. According to (2) __________ theory, the perception of pain can be blocked ty special cells in the spinal cord. Although the results of studies comparing active and passive strategies for coping with pain are inconsistent, they generally confirm that (3) __________ strategies are associated with lower levels of reported pain and greater improvement in psychological and physical function.

1) pain 2) Gate Control 3) active

Propanolol and other beta-blockers are used to treat cardiovascular disease, tremors, and migraine headaches and have been found useful for reducing (1) __________ symptoms of anxiety. Propanolol and other beta-blockers should not be discontinued (2) __________ because doing so can cause sweating, headache, tremulousness, and cardia arrhythmia.

1) physical 2) abruptly

Computed tomography (CT) and magnetic resonance imaging (MRI) provide information on the (1) __________ of the brain, while (2) __________, SPECT, and fMRI provide information on the brain's (3) __________ activities.

1) physical structures 2) functional

The atypical antipsychotic drugs (e.g., clozapine) are associated with several advantages. They alleviate both the (1) __________ symptoms of Schizophrenia and are often effective when traditional drugs have failed. In addition, they are less likely to produce (2) __________ and other extrapyramidal side effects (except akathisia). However, they can cause (3) __________ or other blood dyscrasias.

1) positive and negative 2) tardive dyskinesia 3) agranulocytosis

Each hemisphere of the cerebral cortex contains four lobes. Within the frontal lobe, the (1)__________ is located on the precentral gyrus and is involved in the execution of movement, while the (2) __________ plays a role in the learning of new motor sequences and, in conjunction with the primary motor cortex, prefrontal cortex, inferior parietal cortex, basal ganglia, and cerebellum, mediates motor (3) __________.

1) primary motor cortex 2) supplementary motor area (SMA) 3) imagery

Hormone replacement therapy (HRT) affects estrogen levels or a combination of estrogen and (1) __________ levels and is effective for alleviating (2) __________, mood swings, and vaginal dryness.

1) progesterone 2) hot flashes

The central nervous system (CNS) includes the brain and the spinal cord. The consequences of damage to the spinal cord depend on its location and extent, For example, complete transection at the cervical level results in (1) __________, while damage at the thoracic level causes (2) __________.

1) quadriplegia 2) paraplegia

Migraine headaches are precipitated or aggravated by a broad range of factors including stress and (1) __________ after stress, alcohol, and foods containing tyramine or phenylethylamine. While migraines involve (2) __________ pain, usually on one side of the head, (3) __________ headaches are characterized by a non throbbing pain on both sides of the head.

1) relaxation 2) throbbing 3) tension

The traditional antipsychotic drugs are used primarily as a treatment for (1) __________ and are especially effective for alleviating its (2) __________ symptoms such as delusions and hallucinations.

1) schizophrenia 2) positive

At puberty, an increase in sex hormones influences the emergence of the (1) __________ sex characteristics. This process is mediated by the (2) __________ axis.

1) secondary 2) hypothalamic-pituitary-gonadal

Brain lateralization has been demonstrated by studies examining the effects of severing the corpus callosum in order to control severe (1) __________. When a familiar object is presented to the (2) __________ visual field of a "split-brain" patient, the patient can name the object but cannot pick it out with their (3) __________ hand.

1) seizures 2) right 3) left

Psychophysics is the study of the relationship between physical stimulus magnitudes and their corresponding psychological (1) __________. According to (2) __________ Law, the more intense the stimulus, the greater the increase in intensity needed to produce a just noticeable difference. Fechner's Law proposes that physical stimulus changes are (3) ___________ related to their psychological sensations; and according to (4) __________ Law, sensation is an exponential function in stimulus intensity.

1) sensations 2) Weber's 3) logarithmically 4) Steven's Power

Research on the effects of the traditional antipsychotic drugs provides support for the dopamine hypothesis, which attributes Schizophrenia to (1) __________ to dopamine or excessive dopamine levels. These drugs are associated with several undesirable side effects including (2) __________ effects (e.g., dry mouth, constipation, blurred vision) and extrapyramidal symptoms. Of the latter (3) __________ is the most serious side effect, but its symptoms may eventually decrease if the drug is (4) __________.

1) sensitivity 2) anticholinergic 3) tardive dyskinesia 4) gradually withdrawn

During fetal development, differentiation of the gonads initially depends on the (1) __________ but later is largely the result of hormonal exposure. For a genetic male to develop male genitalia, the fetus must be stimulated during the critical period by (2) __________.

1) sex chromosome 2) androgens

Lazarus' cognitive appraisal theory proposes there are differences in how emotion-arousing events are interpreted or appraised and distinguishes between three types of appraisal: Primary Appraisal refers to a person's evaluation of a (1) __________ as irrelevant, positive-benign, or stressful with regard to their own well-being; secondary appraisal refers to the person's evaluation of the resources they have to cope with a situation that has been identified as stressful; and (2) __________ occurs when the person monitors the situation and, as necessary, modifies their primary and/or secondary appraisals.

1) situation 2) Re-appraisal

The peripheral nervous system (PNS) is divided into the (1) __________ nervous system, which carries information between the CNS and the body's sensory systems and skeletal muscles, and the (2) __________ nervous system, which regulates the activities of the visceral muscles and glands.

1) somatic 2) autonomic

The parietal lobes contain the (1) __________ cortex, which is located on the post central gyrus. Damage to this area may cause tactile (2) __________, apraxia, anosognosia, and disturbances in spatial orientation.

1) somatosensory 2) agnosia

The brain and spinal cord are protected by three meninges and cerebrospinal fluid (CSF). Accumulation of the CSF in the (1) __________ space, as the result of an obstruction, causes enlargement of the ventricles, which can cause hydrocephalus and is also implicated in (2) __________.

1) subarachnoid 2) schizophrenia

The autonomic nervous system has two subdivisions: The (1) __________ branch prepares the body for "fright or flight," while the (2) __________ branch is associated with the conservation of energy and states of rest and relaxation.

1) sympathetic 2) parasympathtic

A number of areas of the brain play an important role in learning and memory. The (1) __________ lobes are essential for the storage and retrieval of long-term declarative memories; the (2) __________ is involved in memory consolidation and explicit memory; and the (3) __________ attaches emotion to memory. In addition, the (4) __________ cortex plays a role in episodic memory, prospective memory, and working memory, while the (5) __________, cerebellum, and motor cortex contribute to procedural and implicit memory.

1) temporal 2) hippocampus 3) amygdala 4) prefrontal 5) basal ganglia

The forebrain consists of subcortical and cortical structures. One of the subcortical structures, the (1) __________ , acts as a relay station for all of the senses except (2) __________.

1) thalamus 2) olfaction

Disulfiram and naltrexone are both used to prevent alcohol use. Disulfiram does so by causing (1) __________ that deter the individual from drinking, while naltrexone blocks the (2) __________ and reinforcing effects of alcohol.

1) unpleasant reactions (symptoms) 2) craving for

Finally the occipital lobes contain the (1) __________ cortex, which is responsible for visual perception, recognition, and memory. Damage may produce (2) __________, which is an inability to recognize familiar objects. Lesions at the junction of the occipital, temporal, and parietal lobes can produce (3) __________, which is the inability to recognize familiar faces.

1) visual 2) visual agnosia 3) prosopagnosia

To detect changes in regional cerebral blood flow to facilitate diagnoses of certain brain-impairing conditions, which of the following would be most useful: a) CT scan and PET b) MRI and CT scan c) PET and fMRI d) CT, MRI, and PET

A functional brain imaging technique is used to assess regional cerebral blood flow. a. Incorrect A CT scan is a structural imaging technique. b. Incorrect MRI and CT scan are both structural techniques. C. CORRECT PET (positron emission tomography) and fMRI (functional magnetic resonance imaging) are functional techniques.

Prospective memory is the ability to remember to execute an intended act in the future and is believed to be supported by activity in which of the following areas of the brain? a) basal ganglia b) corpus callosum c) prefrontal cortex d) raphe nuclei

A number of brain regions and structures are involved in memory including the hippocampus, amygdala, thalamus, basal ganglia, and prefrontal cortex. a. Incorrect The basal ganglia play a role in procedural and implicit memory. b. Incorrect The corpus callosum has not been identified as a mediator of prospective memory. c. CORRECT Areas in the prefrontal cortex play an important role in several aspects of memory including prospective, episodic, and working memory. d. Incorrect The raphe nuclei are located in the reticular formation and play a role in sleep and arousal that may be involved in depression.

Increasing age is LEAST likely to have which of the following effects on a man's sexual response cycle: a) the time to achieve an erection will increase b) complete penile erection may not occur until just prior to orgasm c) the resolution phase and refractory period will both increase d) all of the above

A number of predictable changes in the sexual response cycle accompany aging. a. Incorrect This is a characteristic change. Unfortunately, this change, if unexpected, is often misinterpreted as a sign of impotence. b. Incorrect This is also a characteristic change. C. CORRECT Although the refractory period does begin to increase in the 30s or 40s (and may last for several hours by age 60), the resolution phase actually decreases, with loss of erection following orgasm being quite rapid.

Central achromatopsia is caused by bilateral lesions in the occipitotemporal region of the brain and is characterized by an inability to: a) name colors b) distinguish between different hues c) associate a color with an object d) perceive variations in brightness

A number of visual impairments are associated with damage to the occipital lobe. a. Incorrect The inability to name colors despite an intact ability to discriminate between them is referred to color anomia. B. CORRECT For people with central achromatopsia, everything looks grey or "washed out." c. Incorrect The inability to associate a color with an object is referred to as color agnosia. A person with this disorder would, for example, consider a pink hippopotamus to be plausible. d. Incorrect Achromatopsia does not involve an inability to perceive variations in brightness.

Agranulocytosis, a potential side effect of the atypical antipsychotic clozapine (Clozaril) is a ______ disorder. a) seizure b) hormonal c) gastrointestinal d) blood

d. CORRECT Agranulocytosis is an abnormally low level of white blood cells. It is a rare but dangerous side effect of clozapine.

Long-term potentiation is believed to play a critical role in which of the following? a) emotional experience and expression b) hunger and thirst c) sexual behavior d) learning and memory

d. CORRECT LTP is a widely accepted model for the synaptic changes that underlie learning and memory.

Anosognosia is best described as an impairment in: a) coordination and balance b) speech c) self-awareness d) visual perception

Anosognosia is most often caused by brain trauma that affects the right hemisphere of the brain and involves a lack of insight about one's symptoms on the left side of the body. C. CORRECT Anosognosia is defined as a deficit in self-awareness and, more specifically, awareness of one's own symptoms.

Which of the following best describes research findings on the impact of serotonin levels on eating behaviors in individuals with an Eating Disorder: a) low levels of serotonin are believed to precipitate binge eating in Bulimia and starvation in Anorexia b) high levels of serotonin are believed to precipitate binge eating in Bulimia and starvation in Anorexia c) high levels of serotonin are believed to precipitate starvation in Anorexia while low levels are believed to precipitate binge eating in Bulimia d) low levels of serotonin are believed to precipitate starvation in Anorexia while high levels are believed to precipitate binge eating in Bulimia

Abnormalities in serotonin levels have been linked to both Anorexia and Bulimia. C. CORRECT High levels of serotonin have been linked to both appetite suppression and anxiety. Reduced caloric intake by those with Anorexia apparently fosters a sense of calm and personal control by lowering serotonin levels. Low levels of serotonin have been linked to depression and apparently contribute to binge eating in Bulimia - i.e., binging on sweets and carbohydrates increases serotonin levels and thereby elevates mood.

As the result of a brain tumor, Alma A. is no longer able to recognize her family members or friends by looking at their faces. Alma's condition is known as: a) prosopoplegia. b) anosmia. c) prosopagnosia. d) anosognosia.

Alma is unable to recognize familiar faces, a condition that is caused by damage to certain areas of the occipital and temporal lobes. a. Incorrect Prosopoplegia is facial paralysis. b. Incorrect Anosmia is the absence of the sense of smell. c. CORRECT Prosopagnosia is a type of visual agnosia that involves an inability to recognize familiar faces, often including one's own face. d. Incorrect Anosognosia is the inability to recognize (or the denial of) one's own neurological symptoms.

Recent research using brain imaging techniques, has found that significant atrophy in the __________ distinguishes people with Alzheimer's dementia from healthy peers. a) vestibular nucleus b) hypothalamus c) entorhinal cortex d) somatosensory cortex

Although pathology in several areas of the brain has been linked to Alzheimer's Dementia, recent research has found that one particular area most clearly distinguishes between individuals with and without the disorder. C. CORRECT MRIs have revealed that individuals in the early stages of Alzheimer's Dementia have a reduction in the volume of the entorhinal cortex of up to 30% when compared to healthy peers and that those in the later stages show a reduction of up to 45%. See, e.g., M. Bobinski et al., MRI of entorhinal cortex in mild Alzheimer's disease, Lancet, 1999, 353, 38-40.

Reduced levels of estrogen and progesterone associated with menopause are most likely to be the direct cause of: a) diminished sexual drive/interest b) discomfort during intercourse c) difficulty in achieving orgasm d) multiple orgasms

Although reductions in estrogen and progesterone can have a negative impact on sexuality, it is testosterone that affects the sex drive. B. CORRECT Reduced levels of estrogen and progesterone lessen lubrication and have other adverse effects on vaginal tissues and, consequently, can cause discomfort during intercourse. In terms of sexual desire, the change in balance between estrogen, progesterone, and testosterone can actually result in an increase in desire since testosterone is the hormone responsible for sex drive.

Research on the role of hormones on sexual arousal suggests that: a) estrogen is responsible for arousal in females, while androgen is responsible for arousal in males b) estrogen is responsible for arousal in males, while androgen is responsible for arousal in females c) estrogen is responsible for arousal in both males and females d) androgen is responsible for arousal in both females and males

Although research on the role of hormones in sexual behavior remains somewhat inconclusive, there is sufficient evidence to conclude that certain hormones do play a role in human sexual arousal. D. CORRECT The research suggests that estrogen plays an insignificant role in female sexual motivation and arousal and that androgen, which is produced by the adrenal cortex, plays an important role in the sexual functioning of both males and females.

Following abrupt cessation of a sedative after long-term use, REM sleep would be expected to: a) increase to above normal levels. b) increase to normal levels. c) decrease to below normal levels. d) decrease to normal levels.

Although sedative drugs increase the total sleep time, they decrease the amount or proportion of REM sleep. a. CORRECT Abrupt cessation of a sedative following long-term use typically results in a "REM rebound," which is characterized by an above-normal amount or proportion of REM sleep.

Research has most consistently linked the behavioral and psychological symptoms of Alzheimer's Dementia and related types of Dementia to low levels of ____________ in regions of the brain that control behavior and emotion: a) acetylcholine b) GABA c) glutamate d) adrenalin

Although several neurotransmitters have been linked to Alzhemier's disease and other types of dementia, only one of those listed in the responses has most consistently been associated with their symptoms. A. CORRECT Acetylcholine was initially linked to memory loss in Alzheimer's disease but has since been found to play a role in other symptoms of this disorder. See, e.g., P. Robert (2002). Understanding and managing behavioral symptoms in Alzheimer's disease and related dementia: Focus on rivastigmine, Current Medical Research Opinion, 18, 156-171.

Lesions in the right hemisphere of the cerebral cortex are most likely to have which of the following effects? a) increase catastrophic reactions to benign events b) produce indifference or excessive cheerfulness c) produce excessive fear and anger d) decrease talkativeness

Although the studies suggest that simple left-right distinctions about the brain are not entirely accurate, some generalizations can be made. With regard to emotions, the left hemisphere mediates positive emotions, while the right hemisphere mediates negative emotions. a. Incorrect This is associated with damage to the left hemisphere. b. CORRECT The right hemisphere governs negative emotions, and lesions to this side of the brain produce an opposite effect - i.e., indifference, emotional lability, and atypical humor (e.g., either not responding to jokes or cartoons or laughing even when they are not understood). c. Incorrect This is the opposite of what is true. d. Incorrect This is also the opposite of what is true. Lesions in the right hemisphere often increase talkativeness.

Unilateral damage to the left (dominant) hemisphere is least likely to result in an inability to: a) memorize vocabulary words. b) construct a daily schedule. c) read a map. d) use the process of elimination to answer multiple-choice questions.

Although the two hemispheres of the cerebral cortex play a role in most behaviors, each hemisphere specializes with regard to certain functions. For most people language, math, and analytical thinking are left hemisphere functions, while the understanding of spatial relations, creativity, intuitive thinking, and the ability to recognize faces are right hemisphere functions. c. CORRECT Of the activities listed in the answers, the ability to read a map would most likely be impaired by damage to the right hemisphere. In contrast, damage to the left hemisphere would have the effects listed in answers a, b, and d.

As the result of a traumatic head injury sustained in a motor vehicle accident, a middle-aged man has mild impairments in attention and other executive cognitive functions. Damage to which of the following areas of the brain is most likely responsible for these deficits: a) dorsolateral prefrontal region b) orbitofrontal region c) medial temporal region d) posterior parietal region

Although traumatic brain injury can produce diffuse brain injury, the frontal and temporal lobes are the areas of the cortex that are usually most adversely affected. A. CORRECT The dorsolateral prefrontal region is involved in working memory and the executive functions. Damage to this area is associated with deficits in attention, planning, problem-solving, and other higher-order cognitive abilities. b. Incorrect The orbitofrontal region is involved in inhibition. Damage to this area may produce disinhibition, impaired judgment and insight, and inappropriate affect. c. Incorrect The temporal lobes are involved in learning and memory, and damage to the medial temporal lobes along with certain adjacent structures can produce amnesia.

The occurrence of amphetamine-induced psychosis supports the link between _______ and Schizophrenia: a) dopamine b) acetylcholine c) GABA d) glutamate

Amphetamines exert their effects by increasing levels of dopamine. A. CORRECT Amphetamine psychosis and Schizophrenia share a number of symptoms. Because amphetamines are known to exert their effects on dopamine pathways, this similarity confirms that dopamine also plays a role in Schizophrenia.

As a method for detecting the consequences of a traumatic brain injury, a standard EEG (electroencephalogram) is most useful for evaluating which of the following: a) seizure activity b) vestibular disturbances c) acute hematomas d) sleep disturbances

An EEG measures electrical activity of the brain and, in terms of traumatic brain injury, has been used to detect focal slowing and epileptiform activity. See, e.g., Mild Traumatic Brain Injury: Identification, Assessment, and Treatment, http://www.neuroskills.com/index.shtml?main=/tbi/injury.html. A. CORRECT Research on the use of standard EEG as a diagnostic tool for traumatic brain injury has found that it is most effective for detecting seizure activity. b. Incorrect Several methods are used to identify vestibular disturbances including ENG (electronystagmography) and VAT (vestibular autorotation test). c. Incorrect CT (computerized tomography) is useful for detecting acute hematomas and other types of hemorrhage. d. Incorrect Polysomnography is used to assess pattern and type of sleep disturbance (e.g., to evaluate nocturnal seizures, sleep apnea, or narcolepsy).

When taking an anticholinergic drug, a person is least likely to experience which of the following side effects: a) muscle rigidity and tremor b) blurred vision c) constipation d) nausea and vomiting

Anticholinergic drugs block the effects of acetylcholine and are used in the treatment of Parkinson's disease and diarrhea and as a preanesthetic. A. CORRECT In fact, these drugs are useful for alleviating muscle rigidity and tremor. b. Incorrect Blurred vision is a common side effect of the anticholinergic drugs. c. Incorrect Constipation is also a common side effect. d. Incorrect Nausea and vomiting are also side effects of the anticholinergic drugs.

Constructional and dressing apraxia are most often caused by lesions in the: a) right occipital lobe b) left frontal lobe c) right parietal lobe d) left temporal lobe

Apraxia is an inability to perform skilled movements that is not due to muscle weakness, a movement disorder, cognitive deficits, or a lack of cooperation. C. CORRECT Constructional apraxia is characterized by an inability to copy complex geometric figures, while dressing apraxia involves inattention to one side of the body when dressing (most often the left side of the body). Both are most frequently caused by lesions in the right parietal lobe. (Note that, while constructional and dressing apraxia have been observed in patients with damage in certain areas of the frontal lobes, they are most often caused by damage to the right parietal lobe.)

When taken by an older adult as a treatment for bipolar disorder, lithium is most likely to produce which of the following side effects? a) fine hand tremor, polyuria, and confusion b) elevated blood pressure and tachycardia c) nausea, appetite loss, and headache d) confusion, memory loss, and ataxia

As a general rule, older adults are more susceptible than younger adults to the therapeutic and side effects of many drugs. a. CORRECT Relatively low doses of lithium can produce these side effects in older adults. b. Incorrect Stimulant drugs are associated with these side effects. c. Incorrect These are side effects of the SSRIs. d. Incorrect These are common side effects of the benzodiazepines.

In the context of exposure to the human immunodeficiency virus (HIV), the term "seroconversion" refers to: a) the point in time at which transmission of the virus occurred b) the process of converting from antibody negative to antibody positive c) the period during which the infected individual is asymptomatic d) the body's reaction to an anti-retroviral drug

As a general term, seroconversion refers to the development of antibodies to a particular antigen (e.g., HIV). B. CORRECT In the context of HIV, seroconversion refers to the conversion from HIV negative (seronegative) to HIV positive (seropositive) as the result of the presence of antibodies. Note that, because seroconversion often does not occur until several weeks after infection, an infected individual may develop symptoms of acute HIV infection while still having a negative HIV antibody test.

Recent research with mice suggests that the pleasurable effects of cocaine are mediated by ____________ and serotonin: a) acetylcholine b) glutamate c) dopamine d) epinephrine

As long as you are familiar with the major functions of the various neurotransmitters, you wouldn't have to be familiar with recent research using mice to have identified the correct response to this question. C. CORRECT Research by B. A. Rocha et al. (Cocaine self-administration in dopamine-transporter knockout mice, Nature, 1998, 1[2], 132-137) confirmed that dopamine plays an important role in cocaine addiction. It also found that other neurotransmitters--especially serotonin--are involved.

Fluoxetine and other SSRIs act by: a) increasing the release of serotonin at synapses b) increasing the manufacture of serotonin by nerve cells c) increasing the sensitivity of nerve cells to serotonin d) increasing the availability of serotonin at synapses

As long as you know what "SSRI" stands for, you should have been able to identify the right answer to this question. D. CORRECT SSRI stands for selective serotonin re-uptake inhibitor, which means that the SSRIs exert their effects by reducing the uptake of serotonin at nerve synapses, thereby increasing their availability.

The onset of puberty in humans occurs when certain cells in the __________ secrete gonadotropin-releasing hormones: a) tectum b) thalamus c) hippocampus d) hypothalamus

At puberty, the gonads (testes and ovaries) produce their hormones, which, in turn, are responsible for physical sexual maturation. D. CORRECT Secretion of the gonadotropin-releasing hormones by the hypothalamus stimulates production and release of the gonadotropic hormones by the pituitary gland, which then stimulate the gonads to release the sex hormones.

The circadian clock is located in the hypothalamus and is responsible for sleep-wake patterns. The circadian clock is the: 1. pineal gland 2. suprachiasmatic nucleus 3. substantia nigra 4. reticular activating system (RAS)

BIOLOGICAL BASES OF BEHAVIOR 4. The circadian clock is located in the hypothalamus and is responsible for sleep-wake patterns. The circadian clock is the: • 1. pineal gland • 2. suprachiasmatic nucleus • 3. substantia nigra • 4. reticular activating system (RAS) Feedback: CORRECT - Answer 2! It is the cluster of nerves called the suprachiasmatic nucleus (SCN) that functions as the circadian clock. Sleep-wake patterns run on 24-hour cycles with two natural daily peak times for sleeping, at mid-day and at night. The SCN signals the pineal gland to produce melatonin, which in turn causes a drop in body temperature and sleepiness (Response 1). The substantia nigra, specifically loss of cells in this area, has been implicated in Parkinson's disease (Response 3). While the reticular activating system (RAS) filters incoming sensory information, mediates alertness, and is involved in the sleep-wake cycle (Response 4), it consists of a set of cells in the brainstem, and not the hypothalamus.

Propranolol and other beta-adrenergic blocking drugs: a) are most effective for reducing palpitations, tachycardia, tremor, and other somatic manifestations of anxiety. b) are most effective for reducing worry, apprehension, and other psychic (subjective) manifestations of anxiety. c) are equally effective for reducing the somatic and the psychic manifestations of anxiety. d) are equally ineffective for reducing the somatic and the psychic manifestations of anxiety.

Beta-adrenergic blocking drugs (beta-blockers) are used to treat a number of disorders including anxiety. a. CORRECT The beta-blockers reduce the somatic (peripheral) symptoms of anxiety but do not directly affect its subjective manifestations. b. Incorrect Through their effects on the somatic symptoms of anxiety, these drugs may indirectly affect (reduce) the subjective experience of anxiety. However, their direct effects are on the somatic symptoms of anxiety.

Beta-adrenergic blocking drugs, such as propranolol, are most effective for: a) reducing the peripheral manifestations of anxiety such as palpitations, tachycardia, tremor, and sweating b) reducing the psychic experience of anxiety including worry and apprehension c) reducing both the peripheral manifestations and psychic experience of anxiety d) neither reducing the peripheral manifestations nor the psychic experience of anxiety

Beta-adrenergic blocking drugs have been used for several reasons, including the treatment of anxiety. A. CORRECT These drugs seem to exert their effects on the peripheral manifestations of anxiety. b. Incorrect Through their effects on the physical manifestations of anxiety, these drugs can indirectly affect (reduce) the psychic experience of anxiety. However, their direct effects are on the peripheral manifestations.

Brain lateralization begins to develop: a) during the first year of life b) during the second year of life c) during the third year of life d) after the fourth year of life

Brain lateralization refers to the process in which different functions become specialized in one of the two hemispheres. A. CORRECT Evidence suggests that brain lateralization is already present in the first few months of life.

Broca's aphasia is to Wernicke's aphasia as: a) the temporal lobe is to the parietal lobe b) the frontal lobe is to the parietal lobe c) the parietal lobe is to the occipital lobe d) the frontal lobe is to the temporal lobe

Broca's (expressive) aphasia is caused by damage to Broca's area, while Wernicke's (receptive) aphasia is caused by damage to Wernicke's area. D. CORRECT Broca's area is located in the frontal lobe adjacent to the primary motor cortex, while Wernicke's area is located in the temporal lobe adjacent to the primary auditory area.

Damage to Broca's area produces _______________, while damage to Wernicke's area results in ______________. a) expressive aphasia; conduction aphasia b) conduction aphasia; transcortical aphasia c) expressive aphasia; receptive aphasia d) receptive aphasia; expressive aphasia

Broca's and Wernicke's areas are the two primary language areas of the brain. a. Incorrect Damage to Broca's area does cause expressive aphasia, but damage to Wernicke's area does not produce conduction aphasia. Conduction (associative) aphasia is characterized by relatively intact comprehension and fluent speech with anomia and an inability to repeat words. It is caused by damage to the arcuate fasciculus, which connects Wernicke's and Broca's areas. b. Incorrect Damage to Broca's area does not cause conduction aphasia, and damage to Wernicke's area does not cause transcortical aphasia, which results when lesions outside Broca's and Wernicke's areas isolate these areas from other regions of the brain. c. CORRECT Damage to Broca's area causes an inability to produce language (expressive aphasia), while damage to Wernicke's area is associated with an inability to understand written or spoken language with fluent but unintelligible speech (receptive aphasia). d. Incorrect This is the opposite of what is true.

Broca's area is to the left frontal lobe as Wernicke's area is to the: a) right frontal lobe b) left temporal lobe c) right temporal lobe d) left occipital lobe

Broca's and Wernicke's areas are the two primary speech areas in the brain. a. Incorrect In the majority of individuals, speech is mediated by the left hemisphere. b. CORRECT Wernicke's area is located in the left temporal lobe - or, more specifically, on the posterior portion of the left superior temporal gyrus.

Semantic memory involves: 1. the transfer from sensory memory to short term memory. 2. the transfer from short-term memory to long-term memory. 3. the meaning of the words. 4. the pronunciation of the words.

CORRECT - Answer 3! Semantic memory is memory that has to do with the meaning of words.

Carbamazepine and other anticonvulsant drugs were originally used to treat seizure disorders but are now also used to treat: a) paraphilias. b) obsessive-compulsive disorder. c) bipolar disorder. d) autism.

Carbamazepine and other anticonvulsants (e.g., valproic acid and clonazepam) have been identified as effective treatments for only one of the disorders listed in the answers to this question . c. CORRECT Carbamazepine and other anticonvulsant drugs are often useful for treating bipolar disorder for individuals who have not responded to lithium, and there is some evidence that they are particularly effective as mood stabilizers for "rapid cyclers" (individuals who experience frequent mood swings).

The most common cause of cerebral palsy is: a) a congenital endocrine dysfunction. b) brain damage during fetal development. c) nutritional deficiencies immediately following birth. d) early exposure to a teratogen that affects the body's muscles

Cerebral palsy includes a number of neurological disorders that permanently impair the ability to control voluntary muscle movements. In most cases of cerebral palsy, the exact cause is unknown. a. Incorrect See explanation for response b. b. CORRECT Cerebral palsy is the result of brain damage, and, in the majority of cases, the damage occurs prior to birth. One cause of the damage is a lack of oxygen (asphyxia) prior to birth or during the birth process. Other possible causes include exposure to infections, head injury, intracranial hemorrhage, and genetic mutations.

Of the following aspects of memory, which is least likely to be adversely affected by mild to moderate Alzheimer's dementia: a) working memory b) procedural memory c) episodic memory d) semantic memory

Certain aspects of memory are more likely than others to be adversely affected during the early stages of Alzheimer's dementia. a. Incorrect Impairments in working memory are evident in the early stages of Alzheimer's dementia. B. CORRECT Procedural memory is not substantially affected by mild to moderate Alzheimer's dementia, although it may show impairments in the later stages. c. Incorrect Deficits in episodic memory are the most obvious memory deficits in the early stages of Alzheimer's dementia. d. Incorrect Deficits in certain aspects of semantic memory are usually evident in the early stages of Alzheimer's dementia and become more severe and widespread as the disease progresses.

The effectiveness of clomipramine as the treatment for obsessive-compulsive disorder‭ (‬OCD‭) ‬provides support for the hypothesis that this disorder is related to abnormalities in brain levels of: a) acetylcholine. b) epinephrine. c) dopamine. d) serotonin.

Clomipramine and other antidepressants exert their effects by altering levels of norepinephrine, serotonin, and/or dopamine. . d. CORRECT A current theory about the cause of OCD is that it stems from low serotonin levels. This theory is supported by the fact that clomipramine (a tricyclic antidepressant that has anti-obsessional effects) blocks the reuptake of serotonin by presynaptic neurons.

The effectiveness of clomipramine in alleviating symptoms of Obsessive-Compulsive Disorder and trichotillomania suggests that these disorders are due to: a) oversensitivity to GABA b) an imbalance between acetylcholine and dopamine c) oversensitivity to serotonin or dopamine d) a shortage of serotonin

Clomipramine is a tricyclic antidepressant that has been found to be an effective treatment for OCD. D. CORRECT Clomipramine is believed to exert its effects by blocking the reuptake of serotonin from synaptic clefts.

Cocaine is believed to exert its effects primarily by: a) increasing dopamine levels b) decreasing serotonin levels c) increasing sensitivity to epinephrine d) decreasing sensitivity to norepinephrine

Cocaine exerts its effects primarily on neurons in the limbic system. A. CORRECT It appears that cocaine blocks the reuptake of dopamine, thereby allowing it to accumulate in the synapses and continue to stimulate receiving cells.

Combining an MAOI with certain cold, cough, sinus, or allergy medication can produce: a) neuroleptic malignant syndrome b) a hypertensive crisis c) agranulocytosis d) tardive dyskinesia

Combining an MAOI with certain other drugs or certain foods can produce potentially fatal side effects. B. CORRECT An MAOI in combination with certain drugs can produce a hypertensive crisis (noradrenergic syndrome), which involves a severe headache, diaphoresis, elevated blood pressure, neck stiffness, and neuromuscular agitation; or a hyperpyrexic crisis (serotonin syndrome), which is characterized by hyperthermia, tachycardia, nausea, shivering, restlessness, confusion, and insomnia.

Complex partial seizures originating in the temporal lobe are most likely to cause which of the following symptoms? a) a "drop attack" b) pain or tingling on one side of the body c) a feeling of déjà vu or jamais vu d) visual hallucinations or vision loss

Complex partial seizures are due to abnormal electrical activity in one region of the brain. a. Incorrect A "drop attack" is characteristic of atonic seizures and involves sudden falling down. b. Incorrect Pain, tingling, or other bodily sensations are caused by abnormal electrical activity in the parietal lobe. c. CORRECT Typical symptoms of seizure activity in the temporal lobe are a change in perception, a sudden alteration in emotions and/or a sense of déjà vu or jamais vu (a sense of familiarity or unfamiliarity, respectively). d. Incorrect Visual abnormalities (e.g., hallucinations, flashing lights, or vision loss) are symptoms of abnormal electrical activity in the occipital lobe.

Constructional apraxia is caused by damage to the: a) corpus callosum. b) parietal lobe. c) basal ganglia. d) temporal lobe.

Constructional apraxia is characterized by an inability to copy or draw figures or to arrange blocks in a pattern. b. CORRECT Apraxia involves an inability to perform skilled movements that are not due to muscle weakness, sensory loss, general intellectual deterioration, or lack of cooperation. Most forms of apraxia (including constructional apraxia) are caused by damage to the parietal lobe.

As the result of brain damage, a 37-year-old woman has difficulty regulating the rate, rhythm, pitch, and loudness of her speech. This is referred to as: a) dysarthria b) dysosmia c) dyspraxia d) dysprosody

Damage to certain areas in the brain can disrupt the pattern and flow of speech. a. Incorrect Dysarthria involves problems in articulation. b. Incorrect Dysosmia is a disorder in the sense of smell. c. Incorrect Dyspraxia is characterized by severe impairments in writing, drawing, and other tasks requiring fine motor skills. D. CORRECT Prosody refers to the pattern of stress and intonation in speech. Dysprosody is a characteristic of Broca's aphasia and other nonfluent aphasias. Although dysprosody has traditionally been linked to right hemisphere damage, it's presence in Broca's aphasia suggests that prosody is also governed, to some degree, by the left hemisphere.

In its initial stages, Huntington's disease is most likely to cause: a) memory loss with confabulation b) depression or irritability c) mania and impulsivity d) delirium

Depression is the most common psychiatric symptom of Huntington's Disease and is often one of the initial symptoms. B. CORRECT Signs of depression in Huntington's disease include an inability to experience pleasure, lack of energy, and hostility/irritability.

It is generally believed that, of the lobes of the cerebral cortex, the __________ lobes are the last to fully develop and myelinate: a) frontal b) temporal c) parietal d) occipital

Development of the cortex corresponds to the emergence of various capabilities. A. CORRECT The frontal lobes mediate a number of functions including higher-order cognitive skills, which do not fully develop until early adulthood.

While relaxing in front of the TV, Dexter D. is suddenly startled by strange noises coming from his basement and, as a result, his blood pressure increases, his heart starts racing, and he begins to sweat. Dexter's physical reactions to the strange noises are mediated by his __________ nervous system. a) enteric b) somatic c) sympathetic d) parasympathetic

Dexter is exhibiting physical signs associated with the "fight-or-flight" response. a. Incorrect The enteric nervous system is sometimes categorized as part of the autonomic nervous system. It is a network of nerves located in the gastrointestinal tract and is believed to be involved in several functions including gastrointestinal motility and secretions, activities of the endocrine system, and regulation of immune and inflammatory processes. b. Incorrect The somatic nervous system consists of sensory nerves that carry signals from the sensory receptors to the CNS and motor nerves that carry signals from the CNS to the skeletal muscles. It is responsible for activities that are ordinarily considered voluntary. c. CORRECT The sympathetic nervous system is a branch of the autonomic nervous system. It mediates arousal and the expenditure of energy and prepares the body for fight or flight. d. Incorrect The parasympathetic nervous system is another branch of the autonomic nervous system. It is involved in the conservation of energy and is active during digestion and periods of rest and relaxation.

When in a restful, relaxed state, your brain emits regular high-amplitude, low frequency ______ waves. a) beta b) delta c) theta d) alpha

Different states of wakefulness and sleep are associated with different brain wave patterns. a. Incorrect Beta waves occur during awake, alert states. b. Incorrect Delta waves are characteristic of Stage 3 and Stage 4 (deep) sleep. c. Incorrect Theta waves are associated with deep relaxation and drowsiness and predominate during Stage 2 sleep. d. CORRECT Alpha waves are regular high-amplitude, low frequency waves that are recorded during states of restfulness and relaxation. Alpha waves predominate during Stage 1 sleep.

Which of the following describes the "rebound effect" that is associated with the use of a benzodiazepine as a treatment for anxiety? a) the persistence of symptoms when the drug is prescribed in high dose b) the re-emergence of symptoms after long-term use of the drug c) an initial paradoxical increase in symptoms when the drug is prescribed in a low dose c) a temporary increase in the severity of symptoms when the drug is discontinued

Discontinuation of a benzodiazepine may result in a "rebound effect." d. CORRECT A person taking a benzodiazepine for anxiety, for example, may experience "rebound anxiety" when he or she stops taking the drug, especially when cessation is abrupt rather than gradual. Rebound symptoms are often more severe than the original symptoms.

The "rebound effect" associated with benzodiazepine use refers to: a) the persistence of symptoms despite an increase in the dose of the drug b) the re-emergence of symptoms after long-term use of the drug c) an initial paradoxical increase in symptoms d) a temporary increase in severity of symptoms when the drug is discontinued

Discontinuation of a benzodiazepine may result in a variety of undesirable symptoms including a "rebound effect." D. CORRECT A person taking a benzodiazepine for anxiety, for instance, may experience "rebound anxiety" when he/she stops taking the drug. Rebound anxiety occurs within hours to days after the drug is stopped and is more severe than the original symptoms.

The __________ is a key structure in the mediation of conditioned fear: a) Medulla b) Thalamus c) Amygdala d) RAS

Don't let the term "conditioned fear" confuse you. To answer this question, you just need to know which area of the brain is associated with emotion. C. CORRECT The research (including research investigating conditioned fear) has shown that fear is mediated by circuitry in the amygdala.

__________ plays a role in schizophrenia and is involved in the regulation of movement and has been linked to several disorders with prominent motor symptoms including Tourette's Disorder and Parkinson's disease.

Dopamine

A dopamine agonist would be least useful for treating which of the following: a) the cognitive impairments of Alzheimer's disease b) the motor symptoms of Parkinson's disease c) the symptoms of neuroleptic malignant syndrome d) the symptoms of restless leg syndrome

Dopamine agonists stimulate dopamine receptors. A. CORRECT Cholinesterase inhibitors are currently used to treat the memory and other cognitive symptoms of Alzheimer's disease. These drugs reduce the breakdown of acetylcholine in the brain. b. Incorrect Dopamine agonists are increasingly being used as the first-line defense in the treatment of Parkinson's disease as well as in conjunction with levadopa during the later stages of this disorder. Dopamine agonists help control tremor, muscle stiffness, balance problems, and other motor symptoms. c. Incorrect Most theories of neuroleptic malignant syndrome (NMS) attribute it to dopamine blockade, and some experts recommend the use of dopamine agonists to alleviate its symptoms, especially in severe cases. d. Incorrect Pramipexole, a dopamine agonist, has been found to be an effective treatment for restless leg syndrome for some patients.

A drug that mimics or enhances the effects of a neurotransmitter is referred to as: a) an agonist b) an antagonist c) ionotropic d) metabotropic

Drugs that increase or decrease the effects of neurotransmitters and hormones are referred to, respectively, as agonist and antagonists. A. CORRECT An agonist increases activity of nerve cells by stimulating the production of a neurotransmitter or hormone, or stimulation of receptors for that neurotransmitter or hormone. b. Incorrect An antagonist blocks the effects of a neurotransmitter or hormone. c. Incorrect An ionotropic effect is a synaptic effect that depends on the rapid opening of a "gate" in the synaptic membrane. d. Incorrect A metabotropic effect is a synaptic effect that depends on metabolic reactions.

The neurotransmitter associated with voluntary movement is: 1. acetylcholine. 2. serotonin. 3. norepinephrine. 4. GABA.

Feedback: CORRECT - Answer 1! Acetylcholine (Response 1) is involved in voluntary movement, as well as learning and memory. Serotonin (Response 2) is involved with mood, sleep, appetite, sex, and aggression. Norepinephrine (Response 3) is also involved with mood, and GABA (Response 4) is associated with anxiety and seizures. Dopamine, while not an option in this question, is also involved with voluntary movement.

Which of the following is least true about the consequences of electroconvulsive shock therapy: a) its effects on memory loss tend to be cumulative b) persisting effects on memory most often involve autobiographical memories c) when administered to the right side of the brain, verbal memory is relatively undisturbed d) the majority of memory deficits are reversible, with a return to premorbid level of functioning within 2 to 3 weeks

ECT continues to be used as a treatment for certain types of depression. Its use is associated with several negative side effects, most notably memory loss. a. Incorrect This is true. b. Incorrect This also seems to be the case. c. Incorrect When administered to the right hemisphere only, it is nonverbal memory that is disturbed. D. CORRECT This is a tricky question: Most cognitive and memory deficits are reversible but do not return to premorbid levels for six or seven months. See B. Kolb and I. Q. Whishaw, Fundamentals of Human Neurology, New York, W. H. Freeman & Co., 1996.

A menopausal woman who is considering estrogen replacement therapy (ERT) should be advised that this treatment may do all of the following except: a) reduce the risk for osteoporosis b) cause nausea, vomiting, and anorexia or increased appetite c) increase sex drive d) reduce mood symptoms

ERT continues to be controversial because it is not only associated with a number of benefits but also with some negative side effects. a. Incorrect This is one of the major advantages of ERT. b. Incorrect Estrogen can cause these side effects. C. CORRECT In fact, one of the possible side effects of ERT is a decrease in libido. d. Incorrect ERT alleviates many of the undesirable effects of menopause including mood swings, hot flashes, and vaginal dryness.

Which of the following is incorrectly matched? a) alpha waves: relaxed alertness b) beta waves: waking consciousness c) delta waves: deep sleep d) theta waves: sudden arousal from sleep

Each of the brain waves listed in the answers is associated with a different physiological state. Note that this question is asking which response represents an incorrect match. a. Incorrect Alpha waves predominate during states of relaxed wakefulness. b. Incorrect Beta waves predominate during states of alert wakefulness. c. Incorrect Delta waves occur during periods of deep sleep. d. CORRECT Theta waves occur when a person is in a deeply relaxed, drowsy state. Theta waves predominate during Stage 2 sleep.

The positive symptoms of __________ include hand tremor that has been described as "pill-rolling" since it resembles the motion of rolling a pill between the thumb and forefinger: a) Parkinson's Disease b) Huntington's Chorea c) Multiple Sclerosis d) Myasthenia Gravis

Each of the disorders listed involves abnormalities in motor functioning. A. CORRECT Parkinson's Disease includes both positive and negative symptoms, and the symptom described in this question is one of the former. Other positive symptoms are involuntary movements and muscular rigidity; negative symptoms include postural fixation, disturbances in equilibrium and locomotion, akinesia, and speech disturbances. b. Incorrect Huntington's chorea involves frequent involuntary irregular movements that affect the whole limbs or parts of limbs. c. Incorrect Multiple sclerosis is a progressive disease that is due to a loss of myelin and that begins with paresthesia, muscle weakness, and visual disturbances. d. Incorrect Myasthenia gravis means "severe muscle weakness" and is characterized by severe muscle fatigue after muscles have been exercised a few times.

__________ seizure is characterized by a loss of consciousness without prominent motor symptoms. a) atonic b) absence c) clonic d) simple partial

Each type of seizure listed in the answers is associated with a different etiology and combination of symptoms. a. Incorrect Atonic seizures involve a sudden loss of muscle tone that may cause the individual to fall down. The individual usually remains conscious during the seizure. b. CORRECT The question accurately describes an absence (petit mal) seizure. c. Incorrect Clonic seizures are characterized by jerky movements. d. Incorrect A simple partial seizure begins on one side of the brain and, at least initially, affects only one side of the body. It causes changes in movement and sensation but does not produce a loss of consciousness.

Older people are especially sensitive to both the therapeutic and toxic effects of psychoactive drugs. For example, for elderly (vs. younger) adults, benzodiazepine use is more likely to produce: a) unsteadiness, confusion, wandering, and memory loss b) bizarre or hostile behavior, paranoia, and confusion c) extrapyramidal symptoms and cardiovascular effects d) tremor, weight gain, restlessness, and fatigue

Elderly adults are more susceptible to the toxic effects of drugs (including benziodiazepines), even when the drugs are taken at a relatively low dose. A. CORRECT These side effects are much more common in elderly (vs. younger) adults. b. Incorrect These side effects are associated with analgesics and anti-inflammatories. c. Incorrect These side effects are associated with neuroleptic drugs and are more likely in elderly than younger adults.

A partial seizure is characterized by: a) a focal onset in one hemisphere of the brain with or without a loss of consciousness b) a focal onset in one hemisphere of the brain without a loss of consciousness c) a focal onset in one hemisphere of the brain with a loss of consciousness d) a simultaneous onset in both hemispheres of the brain without a loss of consciousness

Epileptic seizures are categorized as either partial or generalized. A. CORRECT Partial seizures begin in one hemisphere and affect movement and sensations beginning on one side of the body. Simple partial seizures do not affect consciousness while complex partial seizures do alter consciousness.

Disinhibition, apathy, and deficits in executive functioning are most associated with damage to the __________ lobes. a) parietal b) frontal c) temporal d) occipital

Familiarity with the primary functions of each of the lobes of the cerebral cortex would have helped you identify the correct answer to this question. a. Incorrect Damage to the parietal lobes affects somatosensory functioning. b. CORRECT The prefrontal area of the frontal lobes is involved in a variety of complex behaviors including emotions, memory, self-awareness, and executive cognitive functions. Depending on its location, damage to this area can cause disinhibition (impulsivity, loss of tact), apathy (limited motivation and initiative), and/or deficits in higher-order cognitive skills. c. Incorrect Temporal lobe damage may cause memory loss and auditory deficits. d. Incorrect Damage to the occipital lobes produces disturbances in visual processing.

Radiation treatment for children with tumors in the cerebellum (cerebellar astrocytomas) may result in: 1. learning problems. 2. impaired visuospatial abilities. 3. memory problems. 4. apraxias.

Feedback: CORRECT - Answer 1! Among childhood brain tumors, 15-25% are cerebellar astrocytomas, which can be either benign or malignant. Chemotherapy is usually the preferred mode of treatment to avoid the problematic side effects of radiation. In young children, radiation can cause learning problems as well as reduction in growth. Other common side effects of radiation treatment for cerebellar astrocytomas include fatigue, skin rashes, and decreased appetite.

Diseases that are X-linked are: 1. more common in men than women. 2. more common in women than men. 3. about equally prevalent in men and women. 4. more common in adults than children.

Feedback: CORRECT - Answer 1! Diseases that are X-linked (such as color blindness) are much, much more common in males than females. If a mother is a carrier and the father is not color blind, each son has a 50% chance of being color blind. The daughters in this scenario can only be carriers and will not be color blind.

Parkinson's disease is due to loss of cells in the: 1. substantia nigra. 2. basal ganglia. 3. caudate nucleus. 4. hippocampus.

Feedback: CORRECT - Answer 1! The basal ganglia (Response 2) includes, among other structures, the substantia nigra and the caudate nucleus. Thus, although Parkinson's disease is a result of damage to the basal ganglia, it is more specific and accurate to state that it results from a loss of cells in the substantia nigra (Response 1) as well as a decrease in dopamine. Characteristics of Parkinson's disease are resting tremors, akinesia (difficulty initiating movement), and shuffling gait. Huntington's disease results from degeneration of the caudate nucleus (Response 3). The hippocampus (Response 4) is a limbic structure involved in memory and emotion. Damage to the hippocampus can result in problems with consolidation of memory.

As you start to take this practice test you say to yourself, "My heart is racing. I must be very nervous." This kind of thinking supports: 1. the Cannon-Bard theory. 2. the James-Lange theory. 3. Schacter's Two-Factor theory. 4. General Adaptation Syndrome.

Feedback: CORRECT - Answer 2! According to the James-Lange theory, emotions result from perceiving bodily reactions or responses. In this case, you perceived your bodily reaction (heart racing) and concluded you must be very anxious. The Cannon-Bard theory proposes that emotions and bodily reactions occur simultaneously. When an event is perceived (e.g., taking the exam), messages are sent at the same time to the hypothalamus, which arouses the body, and to the limbic system, which causes the subjective experience of anxiety (Response 1). Schacter's Two-Factor theory proposes that emotion results from both internal information (hypothalamus and limbic system) and external information (the context). According to this theory you would be experiencing physiological arousal (racing heart), and would then look to the environment (the licensing exam) to help you label the emotion as anxiety, rather than, for example excitement (Response 3). Selye's General Adaptation Syndrome is a model or response to severe stress consisting of three stages: alarm, resistance, and exhaustion.

A man is undergoing a test of his blood cortisol levels. One reason for this may be: 1. he is baseball player being tested for recent steroid use. 2. his stress level is being monitored. 3. his kidney functioning is being monitored. 4. he is undergoing a test for illicit drug use.

Feedback: CORRECT - Answer 2! Cortisol testing measures the level of corticosteroids in the blood, saliva or urine. Blood cortisol tests are used to monitor stress level (Response 2), as well as to test adrenal function in suspected cases of Cushing's or Addison's diseases. Cortisol would not be monitored in a test for anabolic steroids (as distinguished from corticosteroids). Anabolic steroids are artificial forms of testosterone which are tested for using urinalysis (ruling out Response 1). Drug tests are usually performed by checking for traces of specific illicit substances such as opiates, amphetamines, or marijuana (ruling out Response 4). Blood cortisol level does not reflect level of kidney functioning (ruling out Response 3).

Damage to the temporal lobe is most likely to affect which type of memory function? 1. Short-term explicit memory. 2. Long-term explicit memory. 3. Short-term implicit memory. 4. Long-term implicit memory.

Feedback: CORRECT - Answer 2! It is always difficult to make generalizations about complex functions like memory, but temporal lobe damage (and brain damage in general) does not typically result in damage to implicit memory (Responses 3 and 4). Implicit memory refers to memories that don't require conscious recollection, like driving a car or playing the piano. Explicit memory involves the conscious recollection of facts and autobiographical events. Temporal lobe damage generally results in problems forming long-term memories. The individual with damage can remember things for a short period of time (Response 1) but cannot hold onto the information or consolidate it into long-term memory. Left temporal lobe lesions result in impaired memory for verbal material, while right temporal lobe lesions result in impaired recall of non-verbal material such as music and drawings.

Dopamine is implicated as a causative agent which of the following? 1. Major depressive disorder. 2. Schizophrenia. 3. Generalized anxiety disorder. 4. Substance use disorder.

Feedback: CORRECT - Answer 2! The 'dopamine hypothesis' posits that an excess of dopamine results in schizophrenia. The neurotransmitters implicated in depression (Response 1) are norepinephrine and serotonin. Low levels of GABA are linked with anxiety (Response 3) and epilepsy. Some drugs, such as cocaine or methamphetamine, cause the release of abnormally large amounts of natural neurotransmitters (mainly dopamine). Thus, excess dopamine can result from substance use disorders, but dopamine does not cause substance use disorders (ruling out Response 4).

Wernicke's is to Broca's as: 1. expression is to reception. 2. reception is to expression. 3. agnosia is to aphasia. 4. alexia is to agraphia.

Feedback: CORRECT - Answer 2! Wernicke's area is involved in receptive speech, while Broca's is involved in expressive speech. Agnosia (Response 3) is a failure to recognize sensory stimuli. Aphasia (Response 3) involves a problem with speech. Wernicke's and Broca's are both considered aphasias. Alexia (Response 4) is an acquired reading disorder, while agraphia (Response 4) is an acquired inability to write.

The right hemisphere plays an important role in: 1. language expression and reception. 2. logical analytic functions. 3. abstract thinking. 4. visuospatial organization.

Feedback: CORRECT - Answer 4! The right hemisphere is involved with perceptual, visuospatial, artistic, musical, and intuitive activities. The left hemisphere is involved with language (Response 1), as well as with being rational, analytical, logical (Response 2), and abstract (Response 3).

Which of the following terms refers to a disease of unknown origin? 1. iatrogenic. 2. idiopathic. 3. idiosyncratic. 4. nocebic.

Feedback: CORRECT - Answer 2! When the cause of a disease is unknown, it is referred to as idiopathic. Iatrogenic (Response 1) is a very important medical term that refers to illness or complications, especially infections, that are caused in the course of receiving medical treatment (e.g., a patient contracts TB while in the hospital). Idiosyncratic (Response 3) is a non-technical term for unusual, eccentric, or unique responses. A nocebo (Response 4) is something that should be ineffective but which causes symptoms of ill health; a nocebo is the opposite of a placebo. A nocebo effect is an ill effect caused by the suggestion or belief that something is harmful (e.g., expectation of unpleasant side effects leading to experiencing them despite receiving a sugar pill).

Which of the following predominates in a state of relaxed wakefulness? 1. Theta waves. 2. REM sleep. 3. Alpha waves. 4. Delta waves.

Feedback: CORRECT - Answer 3! Alpha waves occur during periods of relaxed wakefulness, and are predominant just before falling asleep. Theta waves (Response 1) occur in early stage 1 sleep. Delta waves (Response 4) are slow waves that occur in stages 3 and 4, the deepest, non-dreaming part of sleep. REM sleep (Response 2) is dreaming sleep, during which there is rapid eye movement, increased respiration and heart rate, and relaxed muscle tone.

The thalamus is best described as: 1. the master endocrine gland. 2. the seat of consciousness. 3. the relay station for all senses except olfaction. 4. the house of the somatosensory cortex.

Feedback: CORRECT - Answer 3! An all-time favorite on the licensing exam, the thalamus integrates and processes all sensory information except for olfaction (i.e., smell). The pituitary is considered the master endocrine gland (Response 1), as it releases hormones that activate other endocrine glands. The parietal lobes house the somatosensory cortex (Response 4). There is no one brain structure considered to be the seat of consciousness (Response 2).

A high percentage of children with ADHD are responsive to psychostimulants. When a clinician is uncertain as to whether a child's diagnosis is ADHD, the reason the child should not be given a trial of psychostimulants for purposes of differential diagnosis is because: 1. the child who does not have ADHD would likely experience some negative side effects of the medication (e.g., increased heart rate). 2. the child who does not have ADHD would likely experience a bit of a high. 3. the child who does not have ADHD would likely have an increased attention span and improved task performance. 4. the child who does not have ADHD would likely exhibit restlessness and distractibility.

Feedback: CORRECT - Answer 3! Even children without ADHD, when given psychostimulants, have improved attention span and concentration. Therefore, response to such medications would not be useful for differentiating children with ADHD from non-ADHD children.

The part of the brain responsible for consolidation of long-term memory is: 1. the medulla. 2. the hypothalamus. 3. the hippocampus. 4. the cerebellum.

Feedback: CORRECT - Answer 3! The hippocampus, within the temporal lobes, is involved in the consolidation of long-term memory. Response 1, the medulla, is a part of the brainstem, and is involved in basic functions including respiration, cardiovascular activity, sleep, and consciousness. Response 2, the hypothalamus (along with the pituitary), constitutes the master endocrine gland, involved in temperature regulation, the sleep-wake cycle, general arousal, and movement. Response 4, the cerebellum, is involved in movement and balance.

Partial complex seizures typically originate in: 1. the frontal lobes. 2. the brain stem. 3. the temporal lobes. 4. the limbic system.

Feedback: CORRECT - Answer 3! The temporal lobes are most commonly the originating site of complex partial seizures. Keep in mind, though, that such seizures can originate in any cortical region. Complex partial seizures are frequently preceded by an aura, and usually include purposeless behavior, lip smacking, unintelligible speech, and impaired consciousness. The frontal lobes (Response 1) are critical to personality, emotionality, inhibition, planning and initiative, abstract thinking, judgment, and higher mental functions. The brain stem (Response 2) includes the pons, medulla, and reticular activating system (RAS). The limbic system (Response 4) is an interconnected group of organs and structures (e.g., amygdala, hippocampus) involved in memory and emotion.

While playing baseball, you are struck in the head by a fly ball. You are unable to remember the plays leading up to the time you were struck. Most likely you are suffering from: 1. proactive inhibition. 2. retroactive inhibition. 3. anterograde amnesia. 4. retrograde amnesia.

Feedback: CORRECT - Answer 4! For the EPPP, it is important to be very clear about the difference between the two kinds of amnesia. Retrograde amnesia is loss of memory for events occurring before the trauma (retro = backward); anterograde amnesia is loss of memory for events occurring after the trauma (don't think about the prefix 'ante' because it gets very confusing!). Note that amnesia refers to memory loss, and inhibition (Responses 1 & 2) refers to interference in learning.

Neurotransmitter imbalances have been identified in people with eating disorders. Most notably, persons with eating disorders frequently have a deficiency of: 1. acetylcholine. 2. GABA. 3. dopamine. 4. serotonin.

Feedback: CORRECT - Answer 4! Serotonin and norepinephrine are significantly involved in regulation of appetite, as well as mood and sleep. Persons with eating disorders as well as depressive disorders commonly have low levels of these neurotransmitters. This may also partly explain the link that frequently exists between eating disorders and depressive illness. Acetylcholine deficiencies are a hallmark of Alzheimer's disease (Response 1). Low levels of GABA have been found in persons with anxiety disorders (Response 2). A decrease in dopamine is common in Parkinson's disease (Response 3).

The neurotransmitter involved with sleep and mood is: 1. RAS. 2. acetylcholine. 3. dopamine. 4. serotonin.

Feedback: CORRECT - Answer 4! Serotonin is involved with both sleep and mood. Although the RAS or reticular activating system (Response 1) is involved in sleep, it is not a neurotransmitter but rather a diffuse set of cells in the medulla, pons, hypothalamus, and thalamus. Acetylcholine (Response 2) is involved in voluntary movement and memory. Dopamine (Response 3) is involved with Schizophrenia.

Vision is to hearing as: 1. temporal lobe is to occipital lobe. 2. occipital lobe is to parietal lobe. 3. parietal lobe is to occipital lobe. 4. occipital lobe is to temporal lobe.

Feedback: CORRECT - Answer 4! The lobes responsible for vision are the occipital lobes. The temporal lobes contain the primary auditory cortex. The parietal lobes process somatosensory information (e.g., touch, pain, temperature, and proprioception).

Withdrawal from which substance is most likely to cause hallucinations? 1. LSD. 2. Amphetamines. 3. Heroin. 4. Phenobarbital.

Feedback: CORRECT - Answer 4! The substances whose withdrawal syndromes involve hallucinations are alcohol, and the category of sedatives, hypnotics, and anxiolytics. Withdrawal from these substances can potentially be fatal. Phenobarbital is a barbiturate, which is a type of hypnotic. Very commonly used in the 1950s, barbiturates are rarely prescribed as hypnotics now. They still have a prominent place in the treatment of seizures. There is no characteristic withdrawal syndrome for hallucinogens (Response 1). Withdrawal from amphetamines (Response 2) is characterized by dysphoria, fatigue, unpleasant dreams, increased appetite, and psychomotor agitation or retardation. Opioid withdrawal (Response 3) is known for flu-like symptoms.

A researcher investigating the chemical correlates of memory would be most likely to find that administration of which of the following would improve the memory of elderly adults: a) LGN b) THC c) RNA d) GABA

Following administration of yeast RNA, senile adults have been found to exhibit a significant improvement in memory (Cameron, 1958). Although this research result has not always been replicated in humans, it has been consistently shown in animals. a. Incorrect This is the abbreviation for the lateral geniculate nucleus in the brain. b. Incorrect THC, the active ingredient in marijuana, has not been found to improve memory. C. CORRECT As noted above, there is some evidence that RNA is associated with memory consolidation. d. Incorrect GABA, an inhibitory neurotransmitter found in the central nervous system, has not been linked with improved memory.

Following a severe closed head injury caused by a car accident, a 23-year-old male exhibits anterograde and retrograde amnesia. Most likely, which of the following memories will return first during his recovery: a) memory for the events that occurred just after the accident b) memory for events immediately before the accident c) memory for his birthday dinner six months prior to the accident d) memory for his high school graduation

Following moderate to severe head injury, a person is likely to show some degree of both anterograde and retrograde amnesia. In this situation, recovery of long-term memories typically involves a predictable pattern. a. Incorrect Anterograde amnesia is usually more severe than retrograde amnesia, and the individual usually never recalls the incident that caused the amnesia or the events occurring after it because those events were never stored in long-term memory. D. CORRECT Memories of more remote events ordinarily return first.

Early memory loss associated with Alzheimer's dementia is believed to be caused by deterioration of neurons in the hippocampus that secrete: a) GABA. b) serotonin. c) glycine. d) acetylcholine.

For the exam, you should be familiar with the functions that have been associated with the neurotransmitters listed in the answers to this question. a. Incorrect GABA is an inhibitory neurotransmitter. It is believed to play a role in anxiety, sleep, and eating disorders and is affected by CNS depressants. b. Incorrect Serotonin has been linked to a variety of functions including memory. However, abnormalities in serotonin levels have not been as consistently linked to the early memory loss of Alzheimer's dementia as have been abnormalities in acetylcholine. Instead, serotonin is believed to contribute to other symptoms of this disorder including sensory disturbances, mood swings, and sleep disruption. c. Incorrect Glycine plays a role in spinal reflexes and motor behavior. d. CORRECT Acetylcholine (ACh) mediates motor functions as well as memory processes, and decreased levels of ACh in the hippocampus have been linked to the memory loss associated Alzheimer's dementia.

The _____________ predicts that the size of an action potential is independent of the intensity of the stimulus that initiated it. a) all-or-none principle b) rate law c) principle of equipotentiality d) doctrine of specific nerve energies

For the exam, you want to be familiar with the basic properties of the neuron. a. CORRECT According to the all-or-none principle, as long as a nerve cell receives the minimal degree of stimulation required to trigger an action potential, the action potential is the same size regardless of the amount of stimulation. b. Incorrect The rate law predicts that variations in the intensity of a stimulus are coded by the frequency of the axon's firing - i.e., the more intense the stimulus, the greater the rate of responding. c. Incorrect The principle (law) of equipotentiality proposes that, when an area of the cortex is damaged, other intact areas of the cortex can take over the functions of the damaged area. d. Incorrect The doctrine of specific nerve energies predicts that each sensory neuron is sensitive to all forms of energy but always evokes the same sensation.

The _____________ is the large band of nerve fibers that connects the left and right cerebral hemispheres and permits interhemispheric communication of sensory, motor, and higher-order information. a) corpus callosum b) arcuate fasciculus c) cingulate gyrus d) globus pallidus

For the exam, you want to be familiar with the primary functions of the brain structures listed in the answers to this question. a. CORRECT The corpus callosum is the largest band of fibers connecting the two hemispheres of the brain. Abnormalities in corpus callosum functioning have been linked to several disorders including dyslexia, ADHD, Tourette's syndrome, and schizophrenia. b. Incorrect The arcuate fasciculus connects Wernicke's and Broca's areas and carries signals related to the production and comprehension of language. c. Incorrect The cingulate gyrus is part of the limbic system and is believed to be responsible for focusing attention on emotionally significant events, linking sensory input to emotions, mediating emotional reactions to pain, and regulating aggressive behavior. d. Incorrect The globus pallidus is part of the basal ganglia and is involved in motor control.

_____________ involves partial or complete colorblindness due either to a loss of functioning of the cone cells or damage to the occipitotemporal region of the brain. a) Anosognosia b) Alexia c) Achromatopsia d) Akinetopsia

For the exam, you'll want to be familiar with the characteristics of all of the conditions listed in the answer to this question. a. Incorrect Anosognosia is the failure to recognize one's own neurological symptoms. b. Incorrect Alexia is an inability to read or understand written words. c. CORRECT Even if you're unfamiliar with achromatopsia, you would have been able to identify this as the correct answer if you know that "chroma" refers to color. d. Incorrect Akinetopsia is an inability to see objects that are in motion.

____________, a late-appearing side effect of the conventional antipsychotic drugs, is characterized by repetitive, involuntary tic-like movements of the face, eyelids, mouth, tongue, extremities, and/or trunk. a) Acute dystonia b) Parkinsonism c) Tardive dyskinesia d) Akathisia

For the exam, you'll want to be familiar with the symptoms of all of the conditions listed in the answers to this question. b. Incorrect Parkinsonism is an intermediate-onset side effect of the antipsychotic drugs. Its symptoms include muscle rigidity, impaired balance, gait changes, tremors, changes in facial expressions and speech, and muscle cramps. a. Incorrect Acute dystonia is an early-onset side effect of the antipsychotics. Its primary symptoms are muscle spasms and slow abnormal movements, most often in the eyes, jaw, and tongue. c. CORRECT Tardive dyskinesia is a serious and potentially irreversible side effect of the antipsychotics (especially the conventional drugs), and the question describes its primary symptoms. d. Incorrect Akathisia is another intermediate-onset side effect of the antipsychotics. It involves an uncomfortable sense of restlessness that may be accompanied by foot tapping, leg shaking, or pacing.

Sweating, speeded up mental processes, inability to sleep or relax, trembling, nervousness, palpitations, and loss of weight despite increased food intake are characteristic of which of the following endocrine disorders? a) hyperglycemia b) hyperthyroidism c) adrenal insufficiency d) hypoparathyroidism

For the licensing exam, you want to be familiar with the symptoms of the endocrine disorders listed in the answers to this question. a. Incorrect Hyperglycemia is due to excessive glucose in the blood. Its symptoms include fatigue, polydipsia, polyuria, polyphagia, weight loss, poor wound healing, and recurrent infections. b. CORRECT The symptoms listed in this question are characteristic of hyperthyroidism, which is caused by excessive secretion of thyroxin by the thyroid gland. c. Incorrect Adrenal insufficiency (also known as Addison's disease) is due to a lower-than-normal production of cortisol by the adrenal glands and is characterized by weight loss, orthostatic hypotension, weakness, fatigue, and hyperpigmentation. d. Incorrect Hypoparathyroidism is caused by lower-than-normal levels of parathyroid hormone. Its symptoms include muscle cramping and twitching, tingling in the lips and fingers, hair loss, and dry skin.

Benzodiazepines reduce anxiety by enhancing the effects of __________.

Gaba

Lesions in the left parietal lobe may produce __________ syndrome, which involves a combination of finger agnosia, right-left confusion, agraphia, and acalculia.

Gertsmann's

The neurotransmitter glutamate is believed to be responsible for: a) the effects of alcohol on cognitive abilities. b) the increased sociability that sometimes accompanies alcohol use. c) alcohol cravings in alcohol abusers. d) dietary changes associated with chronic alcohol use.

Glutamate plays a role in learning and memory and, more specifically, in long-term potentiation which is believed to mediate the formation of long-term memories. a. CORRECT Alcohol alters glutamate levels in the brain, which can cause memory impairment and may play a role in alcohol-related blackouts. b. Incorrect GABA is an inhibitory neurotransmitter; and alcohol enhances GABA activity which may contribute to the increased sociability associated with alcohol consumption. c. Incorrect Serotonin is believed to be involved in alcohol craving. d. Incorrect Dietary changes have not been linked to glutamate.

Several recent studies on the progression of HIV infection have evaluated the impact of demographic factors. With regard to age, these studies have most consistently found that, among adults: a) age is unrelated to HIV disease progression b) younger age is associated with a more rapid HIV disease progression c) middle age is associated with a more rapid HIV disease progression d) older age is associated with a more rapid HIV disease progression

HIV prognosis has been linked to several psychosocial risk factors including age. D. CORRECT In a recent study by Farinpour et al. (2003), 1,231 HIV-seropositive men, ages 21 to 66, were followed for approximately 13 years after baseline testing. The results of the study indicated that age, general intellectual functioning, and somatic symptoms of depression were significant predictors of HIV progression and survival - i.e., older age, lower IQ, and the presence of somatic symptoms of depression were associated with a more rapid progression to AIDS, HIV-related dementia, and death. [Farinpour, R., et al., Psychosocial risk factors of HIV morbidity and mortality: Findings from the Multicenter Aids Cohort Study (MACS), Journal of Clinical and Experimental Neuropsychology, 2003, 25(5), 654-670.] These findings are consistent with the results of other studies.

Which of the following is NOT true about heroin: a) it is one of the most reinforcing drugs b) use quickly produces tolerance and physical dependence c) respiratory depression accompanies its euphoric effects d) "flashbacks" are a common side effect of long-term use

Heroin was originally developed as an attempt to eliminate opium addiction. Unfortunately, heroin (also an opiate derivative) turned out to be even more addictive than opium. a. Incorrect This is true about heroin. b. Incorrect This is also true. c. Incorrect This is also true. D. CORRECT Because of the relative short half-life of heroin and other opioids, flashbacks are actually quite rare.

The _____________ mediates the body's physical homeostasis. a) cingulate gyrus b) hypothalamus c) cerebellum d) adrenal medulla

Homeostasis refers to maintaining functions within a fixed or optimal range. a. Incorrect The cingulate gyrus is part of the limbic system and is believed to be responsible for focusing attention on emotionally significant events, linking sensory input to emotions, mediating emotional reactions to pain, and regulating aggressive behavior. b. CORRECT A primary function of the hypothalamus is to maintain the body's homeostasis with regard to temperature, blood pressure, fluid and electrolyte balance, and other bodily states. c. Incorrect The cerebellum is important for balance and posture and is vital to the performance of coordinated and refined motor movements. d. Incorrect The adrenal medulla is the core of the adrenal gland and is involved in several functions that are similar to those of the sympathetic nervous system. It plays an important role in the body's reaction to stress.

An MRI of a patient with Huntington's chorea is most likely to show atrophy in which brain structure: a) caudate nucleus b) suprachiasmatic nucleus c) septum d) medulla

Huntington's disease (chorea) is a basal ganglia disorder, and the basal ganglia are involved in the control of motor movements. A. CORRECT The caudate nucleus and putamen are structures of the basal ganglia and have been found to be the brain areas most severely affected by Huntington's disease.

A primary target of interventions for patients in the early stages of Huntington's Disease is: a) depression b) disorientation c) tremor d) visual disturbances

Huntington's disease involves affective, cognitive, and motor symptoms. A. CORRECT The earliest signs of Huntington's disease are often affective and include depression, anxiety, and emotional lability. For this reason, Huntington's may initially be erroneously diagnosed as an Affective Disorder.

Huntington's disease is an: a) autosomal dominant disorder. b) autosomal recessive disorder. c) X-linked dominant disorder. d) X-linked recessive disorder.

Huntington's disease is an inherited neurodegenerative disorder that is transmitted by a single dominant gene. a. CORRECT The term "autosome" refers to a chromosome that is not an X or Y sex chromosome. Huntington's disease is transmitted by a single dominant autosomal chromosome.

Hydrocephalus is due to abnormalities in which of the following: a) brain stem b) ventricles c) cerebral arteries d) tegmentum

Hydrocephalus ("water head") involves an accumulation of fluid in the head. B. CORRECT Hydrocephalus is caused by an accumulation of cerebrospinal fluid in the ventricles of the brain. In infants, it causes separation of the cranial sutures and usually results in mental retardation.

Which of the following would be most useful for alleviating an attack of hyperventilation? a) an ice pack b) an antacid c) a glass of water d) a paper bag

Hyperventilation is a rapid deep-breathing attack that involves a drop in carbon dioxide, leading to respiratory alkalosis and cerebral hypoxia. Symptoms include chest pain, shortness of breath, tingling and numbness in the hands and feet, dizziness, and impaired concentration and memory. d. CORRECT Breathing into a paper bag alleviates hyperventilation by increasing the level of carbon dioxide in the bloodstream.

The symptoms of hypoglycemia include headache, dizziness, sweating, impaired concentration, confusion, clumsy or jerky movements, weakness, and, in extreme cases, convulsions or a loss of consciousness. The cause of hypoglycemia is: a) low levels of blood glucose b) excessive salt intake c) dehydration d) low insulin levels

Hypoglycemia may be a condition by itself or may be a complication of diabetes, severe liver disease, insulinoma (a pancreatic tumor that produces insulin), or other disorder or a consequence of using certain drugs. A. CORRECT Hypoglycemia is an abnormally low level of blood glucose (sugar) that reduces the brain's ability to function properly. b. Incorrect Excessive salt intake may result in high blood pressure. c. Incorrect Symptoms of dehydration (too little water) include thirst, fatigue, irritability, muscle weakness or cramps, headache, dizziness, nausea, forgetfulness, confusion, and, in severe cases, diminished consciousness, lowered blood pressure, and a rapid, feeble pulse. d. Incorrect A low insulin level is associated with a high level of blood glucose.

Endocrine disorders often mimic psychiatric disorders. For example, hypothyroidism often includes symptoms suggestive of: a) a phobic disorder, delirium, or mania. b) mania, anorexia, or OCD. c) OCD, depression, or a phobic disorder. d) depression, psychosis, or dementia.

Hypothyroidism and other endocrine disorders may produce symptoms that are characteristic of certain psychiatric disorders. d. CORRECT The symptoms of hypothyroidism (e.g., impaired memory and concentration, depression, apathy, confusion, thought disorders, hallucinations) overlap with those associated with depression, psychosis, and dementia.

Hypothyroidism is most associated with: a) depressive symptoms, mental slowing, and memory impairment b) emotional lability, unrealistic fears and doubts, and impaired concentration c) agitated paranoia, confusion, and hallucinations and delusions d) unprovoked anger, episodic confusion, and lethargy or fatigue

Hypothyroidism is caused by underactivity of the thyroid gland. Like other endocrine disorders, hypothyroidism produces symptoms suggestive of a mental disorder. A. CORRECT These are the classic symptoms of untreated hypothyroidism, which produces a slowing down of functioning and diminished capabilities. b. Incorrect These symptoms are more suggestive of hyperthyroidism. c. Incorrect These symptoms are characteristic of hyperparathyroidism. d. Incorrect This sounds more like hypoglycemia.

Sedation (drowsiness) is most likely to be a side effect of which of the following antidepressants: a) Prozac b) Wellbutrin c) Zoloft d) Pamelor

In general, the tricyclic antidepressants are more sedating than the SSRIs and certain other newer antidepressants. a. Incorrect Prozac (fluoxetine) is an SSRI and is less likely than Pamelor to produce sedation. b. Incorrect Wellbutrin (bupropion) is an NDRI and is less likely than Pamelor to produce sedation. c. Incorrect Zoloft (sertraline) is an SSRI and is less likely than Pamelor to produce sedation. D. CORRECT Pamelor (nortriptyline) is a tricyclic antidepressant and, of the antidepressants listed in the answers, is most likely to produce sedation.

Which of the following imaging techniques would be used to obtain a "metabolic map" of the brain during various mental and physical activities: a) CT b) PET c) MRI d) NMR

In this situation, it is the function of the brain that is of interest. a. Incorrect CT is a structural imaging technique. B. CORRECT PET scans provide information on the functions of the brain (e.g., metabolic functions). c. Incorrect MRI is also a structural technique. d. Incorrect NMR (nuclear magnetic resonance) is another name for MRI.

Which of the following is true about the sleep of older adults? a) Older people have less REM sleep but more Stage 3 and Stage 4 sleep. b) Older people have a decreased need for sleep. c) Older people often exhibit an advanced sleep phase. d) Older people often exhibit a delayed sleep phase.

Increasing age is associated with a number of sleep changes and problems. a. Incorrect Increasing age is associated with decreases in REM, Stage 3, and Stage 4 sleep. b. Incorrect This is not true as evidenced by the fact that a lack of sleep at night is accompanied by increased sleepiness during the day. C. CORRECT As we age, our circadian clock advances, causing "advanced sleep phase syndrome," which means that we get sleepier earlier in the evening and wake up earlier in the morning regardless of when we actually went to sleep.

Left-right disorientation is most likely to be caused by a lesion in the: a) corpus callosum b) basal forebrain c) left parietal region d) right occipital region

Left-right disorientation (confusion) is one of the symptoms of Gerstmann's syndrome, which also includes acalculia, agraphia, and finger agnosia. C. CORRECT Left-right disorientation is ordinarily caused by lesions in the left angular gyrus, which is located near the boundary between the parietal and temporal lobes.

A brief period of electrical stimulation of the hippocampus enhances nerve cell electrical activity in that area of the brain for minutes to hours. This is referred to as: a) graded potentiation b) hyperpolarization c) absolute refractory period d) long-term potentiation

Interestingly, the phenomenon described in this question not only occurs in the living brain but can also be induced in cells that have been removed from the brain. a. Incorrect Graded potentiation is a made-up term. b. Incorrect Hyperpolarization refers to a state of inhibition. c. Incorrect During the absolute refractory period, a cell cannot "fire" regardless of the amount of stimulation. D. CORRECT This is the name given to the phenomenon described in this question. Long-term potentiation is believed to underlie certain types of learning and memory.

Which of the following brain structures is responsible for interhemispheric communication: a) precentral gyrus b) arcuate fasciculus c) cingulate gyrus d) corpus callosum

Interhemispheric communication refers to communication between the right and left hemispheres of the brain. a. Incorrect The precentral gyrus is located in the frontal lobe and contains the motor cortex. b. Incorrect The arcuate fasciculus connects Wernicke's area to Broca's area. c. Incorrect The cingulate gyrus is part of the limbic system and is involved in the processing and formation of emotions. D. CORRECT The corpus callosum is the primary bundle of nerve fibers connecting the left and right hemispheres. Its role in transferring information between the hemispheres was identified in the 1950s by Myers and Sperry.

During the night, the mother of an 8-month old baby sleeps soundly. Although street noise and the barking of the neighbor's dog do not awaken her, she immediately awakens to the sound of her baby's cries. The brain structure most responsible for this phenomenon is the: a) frontal lobe. b) ARAS. c) hippocampus. d) substantia nigra

Knowing that ARAS stands for "ascending reticular activating system" would have helped you identify the correct answer to this question. a. Incorrect Certain areas of the frontal lobe are involved in emotion, language, memory, and higher-order cognitive abilities. b. CORRECT The ARAS acts as a "sensory screening system" that arouses the brain whenever important information must be processed. c. Incorrect The hippocampus plays an important role in memory consolidation. d. Incorrect The substantia nigra is part of the basal ganglia and is involved in movement and certain types of learning. Abnormalities in the substantia nigra have been linked to Parkinson's disease, schizophrenia, and epilepsy.

The benzodiazepines exert their therapeutic effects by: a) decreasing GABA levels. b) increasing GABA activity. c) decreasing ACh levels. d) increasing ACh activity.

Knowing that GABA is an inhibitory neurotransmitter and that the benzodiazepines are anti-anxiety drugs would have helped you identify the correct answer to this question. b. CORRECT The benzodiazepines enhance the efficiency of GABA activity at receptor sites and thereby decrease anxiety and enhance sleep.

The initial therapeutic effect of a conventional (traditional) antipsychotic drug will most likely be: a) an increase in goal-directed behaviors. b) an improvement in dysphoria. c) a reduction in memory deficits. d) a reduction in hallucinations and delusions.

Knowing that the conventional antipsychotics are most effective for reducing the positive symptoms of schizophrenia would have helped you identify the correct answer to this question. d. CORRECT The positive symptoms of schizophrenia represent an excess or distortion of normal functions and include hallucinations and delusions, while the negative symptoms represent a reduction or absence of normal functions and include avolition, anhedonia, and alogia.

For the assessment of Huntington's disease, magnetic resonance imaging (MRI) is: a) not useful because it is not sufficiently sensitive to detect the brain anomalies associated with this disorder. b) not useful because it provides information only on brain morphology, which is not helpful for assessing this disorder. c) useful for distinguishing between active and older inactive plaques in the periventricular regions. d) useful for detecting volume reduction in the basal ganglia, even in some asymptomatic individuals.

Knowing that Huntington's disease is caused by abnormalities in the basal ganglia would have helped you identify the correct answer to this question. a. Incorrect An advantage of MRI is that it's very sensitive to anomalies and has better resolution than a CT scan. b. Incorrect As noted below, information about structure is useful in the diagnosis of Huntington's disease. c. Incorrect Plaques in the periventricular regions are characteristic of multiple sclerosis, not Huntington's disease. d. CORRECT Structural and functional brain imaging techniques have both been used to assess brain pathology related to Huntington's disease. There is evidence that MRI (a structural technique) can identify reduced volume in the basal ganglia among genetically at-risk individuals who have not yet exhibited overt symptoms of the disorder.

Prolonged exposure to stress may lead to chronic __________ by the adrenal glands: a) suppression of cortisol release b) secretion of cortisol c) suppression of oxytocin release d) secretion of oxytocin

Knowing that cortisol is secreted by the adrenal cortex would have helped you answer this question. B. CORRECT Cortisol and the other corticosteroids are collectively known as the "stress hormones." Chronic stress leads to chronic secretion of these hormones, which can cause a number of problems including increased susceptibility to infection. c. Incorrect Oxytocin is secreted by the pituitary gland and, in females, controls uterine contractions and milk production.

Recent research indicates that people with Parkinson's disease have lost at least 80% of __________-producing cells in the substantia nigra. a) norepinephrine b) dopamine c) serotonin d) glutamate

Knowing that dopamine is involved in the control of voluntary motor movements would have helped you select the correct response to this question. B. CORRECT Degeneration of neurons that secrete dopamine contributes to movement disorders such as Parkinson's disease and Huntington's chorea.

Second generation (atypical) antipsychotic drugs such as clozapine (Clozaril) and risperidone (Risperdal) are: a) serotonin-dopamine antagonists b) serotonin-dopamine agonists c) serotonin agonists/dopamine antagonists d) serotonin antagonists/dopamine agonists

Knowing that drugs that block dopamine and serotonin receptors have been found useful for alleviating the symptoms of Schizophrenia would have helped you identify the correct response to this question. A. CORRECT To choose the correct response to this question, you also need to know that an antagonist is a drug that blocks the effects of a neurotransmitter and that an agonist is a drug that increases or mimics a neurotransmitter's effects. Many of the second generation antipsychotics exert their beneficial effects on the positive and negative symptoms of Schizophrenia primarily by blocking the effects of dopamine and serotonin.

Gate control theory predicts that: a) a neural mechanism in the spinal cord either blocks pain signals or allows them to be transmitted to the brain. b) although each sensory neuron is sensitive to all forms of energy, it always evokes the same sensation. c) some auditory stimuli are encoded in terms of the frequency of neural firing. d) motivation arises from constant imbalances in homeostasis.

Knowing that gate control theory is a theory of pain would have helped you eliminate answers c and d as correct responses. a. CORRECT According to gate control theory, certain areas of the spinal cord receive input not only from pain receptors but also from other skin receptors; and if these other receptors are sufficiently active, they "close" the gate to pain signals and, thereby, keep those signals from reaching the brain. Gate control theory explains why applying heat or cold to the site of an injury immediately relieves the pain. b. Incorrect This is predicted by Muller's doctrine of specific nerve energies. c. Incorrect This describes frequency (volley) theory, which is a theory of audition. d. Incorrect This sounds like drive reduction theory.

The second phase of a grand mal seizure involves: a) muscle contractions b) sleep c) jerky movements of the limbs d) automatisms

Knowing that grand mal epilepsy is also now known as tonic-clonic epilepsy would have helped you answer this question if you also know what clonic seizures are. a. Incorrect Muscle contractions are characteristic of the first (tonic) stage. b. Incorrect The clonic stage often ends in a period of sleep. c. CORRECT This describes a clonic seizure, which is the second stage of grand mal epilepsy. d. Incorrect Automatisms are characteristic of complex partial seizures.

___________ is caused by an abnormal accumulation of cerebrospinal fluid in the ventricles. a) Spina bifida b) Meningitis c) Hydrocephalus d) Encephalitis

Knowing that hydrocephalus means "water head" would have helped you identify it as the correct answer to this question. a. Incorrect Spinal bifida is a neural tube defect caused by a failure of the fetus's spine to close appropriately during early development. b. Incorrect Meningitis is inflammation of the membranes of the brain and spinal cord and is a possible cause of acquired hydrocephalus. c. CORRECT The accumulation of cerebrospinal fluid in the ventricles (usually as the result of an obstruction) causes hydrocephalus which results in increased intracranial pressure and can lead to brain damage. d. Incorrect Encephalitis is an inflammation of the brain that is usually caused by a virus.

There is now evidence that nicotine enhances alertness and memory by mimicking __________ at nicotinic receptor sites. a) endorphins b) GABA c) serotonin d) acetylcholine

Knowing that nicotinic receptors are a type of acetylcholine receptor would have helped you choose the correct response to this question. D. CORRECT There is evidence that the nicotine in tobacco exerts its effects on physiology and behavior by binding to nicotinic receptors in the brain.

PET scans of people with _______________ show that these individuals often have increased activity levels in the orbitofrontal cortex, cingulate cortex, and caudate nucleus. a) Alzheimer's Dementia b) Narcolepsy c) Obsessive-Compulsive Disorder d) Schizophrenia

Knowing that the caudate nucleus is one component of the basal ganglia and that the basal ganglia are involved in voluntary movement and that the orbitofrontal cortex and cingulate cortex mediate emotional reactions may have helped you identify the correct response to this question. C. CORRECT Activity in these areas of the brain is higher in individuals with OCD, especially during provocation of symptoms. In contrast, drug and behavioral treatments for OCD reduce activity in these areas.

A stroke involving the left middle cerebral artery is most likely to cause which of the following? a) right-sided hemiplegia or hemiparesis, nystagmus, and vertigo b) left-sided hemiplegia or hemiparesis, double vision, and facial paralysis c) right-sided hemiplegia or hemiparesis, right-sided hyposthesia, and aphasia d) left-sided hemiplegia or hemiparesis, left-sided hyposthesia, and prosopagnosia

Knowing that the left hemisphere controls functions on the right side of the body, while the right hemisphere controls functions on the left side of the body would have helped you eliminate answers b and d. a. Incorrect Nystagmus and vertigo are signs of a stroke involving the vertebral artery. b. Incorrect Double vision and facial paralysis are signs of a stroke involving the basilar artery. c. CORRECT The middle cerebral artery provides blood to many regions of the brain, and the most common symptoms of stroke involving this artery include contralateral hemiplegia or hemiparesis and hyposthesia (lack of sensation). In addition, when the stroke involves the left cerebral artery, aphasia and apraxia may also occur. d. Incorrect These symptoms would be caused by a stroke involving the right middle cerebral artery.

A lesion in the right parietal lobe is most likely to cause which of the following? a) unilateral neglect. b) apperceptive agnosia c) receptive aphasia d) pseudodepression or pseudopsychopathy

Knowing that the parietal lobe controls the somatosensory functions of the body would have helped you identify the correct answer to this question. a. CORRECT Unilateral (contralateral) neglect involves a lack of attention to or interest in one side of the body. It is usually results from damage to the right parietal lobe and involves the left side of the body. b. Incorrect Apperceptive agnosia (an inability to recognize the basic shape of objects) is caused by lesions in certain areas of the occipital lobe. c. Incorrect Receptive (Wernicke's) aphasia is caused by damage to Wernicke's area, which is in the temporal lobe. d. Incorrect Pseudodepression and pseudopsychopathy are caused by damage to certain areas of the frontal lobe.

Following a stroke involving the middle cerebral artery and the right (nondominant) hemisphere, a person is most likely to exhibit: a) paralysis on the left side of his body and vision loss in his left visual field b) paralysis on the left side of his body and global aphasia c) paralysis on the right side of his body and visual agnosia d) paralysis on the right side of his body and visual loss in his right visual field

Knowing that the right hemisphere controls the left side of the body and the left visual field would have helped you choose the correct answer to this question. A. CORRECT Damage to certain areas of the right hemisphere could cause paralysis on the left side of the body as well as vision loss involving the left visual field. b. Incorrect In most people, language is mediated by the left hemisphere so that damage to the right hemisphere would not cause aphasia (language disturbances).

Our peripheral vision is mediated by the: a) anterior occipital lobe b) posterior occipital lobe c) anterior temporal lobe d) posterior temporal lobe

Knowing that vision is mediated by the occipital lobe would have helped you narrow the choices down to responses a and b. A. CORRECT Peripheral vision is mediated by the anterior occipital lobe, while central vision is mediated by the posterior occipital lobe.

The primary symptoms of Korsakoff's syndrome are: a) amnesia and confabulation. b) visual and auditory hallucinations. c) irritability and aggression. d) aphasia and disorientation.

Korsakoff's syndrome is a brain disorder that is caused by a thiamine deficiency, often related to chronic alcoholism. a. CORRECT The primary symptoms of Korsakoff's syndrome are anterograde and retrograde amnesia and confabulation (inventing events to fill in memory gaps).

L-dopa is used to alleviate the symptoms of: a) tardive dyskinesia. b) Parkinson's disease. c) schizophrenia. d) Tourette's syndrome.

L-dopa (levodopa) is a precursor to dopamine. a. Incorrect In some patients with tardive dyskinesia, L-dopa actually exacerbates symptoms, apparently because the long-term use of an antipsychotic drug has increased their sensitivity to dopamine. b. CORRECT Parkinson's disease is due to deficiencies of dopamine in certain areas of the brain; and administering L-dopa increases dopamine levels and alleviates the symptoms of the disorder, especially in its initial stages. c. Incorrect Schizophrenia is due to oversensitivity to dopamine, and L-dopa would exacerbate its symptoms. d. Incorrect L-dopa may actually induce the symptoms of Tourette's syndrome.

Following a head injury, a middle-aged man experiences a loss of sensation in the fingers of his left hand. Most likely the damage involves the: a) postcentral gyrus b) precentral gyrus c) lateral fissure d) precentral sulcus

Loss of sensation due to brain injury is likely to involve the somatosensory cortex. A. CORRECT The somatosensory cortex is located on the postcentral gyrus in the parietal lobe.

Eating aged cheese, soy products, processed meats, fermented foods, or other foods containing tyramine while taking an MAOI is contraindicated because doing so can cause which of the following? a) rebound effect b) agranulocytosis c) tardive dyskinesia d) hypertensive crisis

MAOI's are a type of antidepressant drug. Combining these drugs with certain foods or other drugs can produce potentially fatal side effects. d. CORRECT Taking an MAOI in combination with foods containing tyramine can cause a hypertensive crisis which involves elevated blood pressure, severe headache, nausea and vomiting, palpitations, blurred vision, stiff neck, sweating, and confusion.

Neuroleptic __________ is characterized by a rapid onset of muscle rigidity, tachycardia, hyperthermia, and altered consciousness.

Malignant Syndrome

A 35-year old woman is being treated by a psychiatrist and is taking medication for her disorder. At a party, after eating some aged cheese and drinking some wine, the woman develops an extreme headache. A friend drives her to the emergency room of a local hospital where the doctor finds the woman to have extremely elevated blood pressure. The woman is apparently taking which of the following drugs: a) a tricyclic b) lithium c) an MAO inhibitor d) chlorpromazine

Many of the psychoactive drugs can have lethal effects when taken in conjunction with certain foods or other drugs. C. CORRECT Consuming foods or beverages containing tyramine (e.g., aged cheese, chicken liver, beer, wine, soy sauce) when taking an MAO inhibitor is likely to produce a hypertensive crisis, which involves the symptoms exhibited by the woman in this question.

___________ is a sexually transmitted disease that is caused by a virus and involves blisters on or near the genitals, painful urination, a burning or tingling sensation, and, in some cases, headache, fatigue, chills, fever, and other flu-like symptoms: a) Genital herpes b) Candidiasis c) Trichomoniasis d) Genital warts

Many sexually transmitted diseases (STDs) have similar symptoms. However, only one of the STDs listed in the answers is caused by a virus and has the symptoms listed in the question. A. CORRECT Genital herpes is caused by a herpesvirus (most often HSV-2) and involves the symptoms described in the question. b. Incorrect Genital candidiasis is due to a yeast infection (not a virus). It is more common in women and men; and, in women, its symptoms include itching and burning; vulvovaginal pain, irritation, and inflammation; and a thick vaginal discharge. c. Incorrect Trichomoniasis is a parasitic infection that occurs in both men and women, often along with other STDs. In women, its symptoms include a yellow-green or gray vaginal discharge, vulvovaginal itching and discomfort, painful and frequent urination, and discomfort during sexual intercourse. Men are usually asymptomatic but may have a pale white discharge from the penis and painful or difficult urination. d. Incorrect Genital warts are caused by the human papillomavirus (HPV). The warts are red, pink, white, or grey and soft, flat, and irregularly shaped. When irritated, they may become painful; as they increase in size, they may bleed, itch, and produce a discharge.

One of the most common impairments following brain injury is memory loss. Although the nature of the disturbance depends on the locus and extent of the damage, most often it involves: a) an inability to transfer information from sensory memory to short-term memory b) widespread deficits in memory for events that occurred prior to the incident that caused the injury c) "spotty" deficits in memory for both recent and past events d) problems related to retaining certain kinds of new information

Not surprisingly, brain injury typically affects memory, at least temporarily. D. CORRECT The exact nature of the memory disturbance depends, of course, on the location and severity of the injury. However, new learning is commonly impaired, and the severity of the injury is often measured by the length of this post-traumatic amnesia. See, e.g., S. J. E. Lindsay and G. E. Powell (eds.), The Handbook of Clinical Adult Psychology, Routledge, London, 1994.

ECT (electroconvulsive therapy) is most likely to produce permanent: a) anterograde amnesia for certain types of verbal information. b) anterograde amnesia for certain types of nonverbal information. c) retrograde amnesia for autobiographical memories from the recent past. d) retrograde amnesia for autobiographical memories from the remote past.

Memory deficits following ECT are more common with bilateral ECT than with unilateral ECT to the nondominant hemisphere. a. Incorrect See explanation for response c. b. Incorrect See explanation for response c. c. CORRECT Anterograde amnesia does occur with ECT but generally clears up within six months. However, loss of memory for past events (events prior to ECT) is sometimes permanent and most often involves autobiographical memories from the recent past.

The suprachiasmatic nucleus (SCN) has been implicated in the control of circadian rhythms. The SCN is located in the: a) thalamus b) hypothalamus c) amygdala d) caudate nucleus

Mention of the SCN may have "thrown" you, but what you need to know to answer this question is which region of the brain controls circadian rhythms. B. CORRECT A good rule of thumb is, "when in doubt, go with the hypothalamus." Although very small in size, the hypothalamus is involved in a variety of important functions including control of circadian rhythms.

In children with ADHD, the most common side effect of methylphenidate (Ritalin) is: a) appetite loss b) dizziness c) pruritus d) headache

Methylphenidate (Ritalin) is associated with a number of adverse side effects. Often, however, these symptoms can be alleviated by reducing the dosage of the drug. A. CORRECT In children, loss of appetite, abdominal pain, insomnia, and tachycardia are the most common side effects of Ritalin. b. Incorrect This is a less common side effect of methylphenidate.

You would be most concerned about a child with ADHD being prescribed methylphenidate if he has a co-diagnosis of: a) Tourette's Disorder b) Conduct Disorder c) Major Depression d) Mental Retardation

Methylphenidate is the primary treatment for ADHD. The studies suggest, though, that it may be contraindicated in some cases. A. CORRECT The research on this issue is far from consistent, but there is some agreement among experts that caution is necessary when prescribing methylphenidate for children with existing tics or a family history of tics since the drug may elicit or exacerbate the condition.

One of the characteristic symptoms of Kluver-Bucy syndrome is agnosia, which involves: a) abnormalities in motor functioning. b) alterations in dietary habits. c) an inability to recognize familiar objects. d) an impaired sense of touch.

Note that this question is asking specifically about agnosia, not the Kluver-Bucy syndrome. a. Incorrect This is known as parakinesia. b. Incorrect Kluver-Bucy syndrome does involve an alteration in dietary habits, but this is not referred to as agnosia. c. CORRECT Agnosia ("psychic blindness") is a characteristic symptom of Kluver-Bucy syndrome and involves an inability to recognize or interpret visual stimuli. Other symptoms of Kluver-Bucy syndrome include reduced fear, increased docility, compulsive oral behaviors, markedly different dietary habits, and hypersexuality. d. Incorrect This is referred to as dysaphia.

A woman who suffers from recurrent migraines should be advised to: a) take an assertiveness training class b) determine if certain foods precede her headaches c) avoid becoming chilled d) perform relaxation exercises immediately following stressful events

Migraine headaches are triggered and maintained by a number of factors. a. Incorrect Assertiveness (or a lack of) has not been associated with migraines. b. CORRECT In some cases, migraines are precipitated by certain foods. Although the specific foods vary from individual to individual, they often contain tyramine, phenylethylamine (chocolate), or nitrates. c. Incorrect Cold has not been found to be a predictable precipitant of migraine and, in fact, often helps alleviate it. d. Incorrect Interestingly, for some people, migraine headaches are triggered by relaxation following stress. Therefore, for people suffering from migraines, it is important to avoid or minimize stress.

Dementia due to Head Trauma is least likely to entail: a) an alteration in personality b) disturbances in executive functions c) impairments in language d) alterations in the experience and expression of emotion

Moderate to severe head trauma can produce temporary or permanent impairments in cognition, memory, emotion, and/or behavior. C. CORRECT The dementia associated with head trauma is, in the majority of cases, most similar to subcortical dementia. Impairment in language (aphasia) is more common in cortical dementias and, therefore, of the symptoms listed, is least likely to be manifested by a person with Dementia due to Head Trauma. See, e.g., J. Frank, Dementia due to head trauma, eMedicine Journal, 3(1), 2002, available at: http://www.emedicine.com/med/topics3152.htm.

Which of the following psychiatric symptoms are most associated with Huntington's disease? a) depression and irritability b) schizophrenia and delirium c) anorexia and anxiety d) sleep and sexual disturbances

Mood symptoms, especially dysthymia and depression, are early symptoms in about 40% of cases of Huntington's disease. a. CORRECT Early reports linked Huntington's disease to psychotic symptoms, but more recent, better-controlled studies have found that affective symptoms are more common and are often the first signs of the disorder.

Which of the following is least likely to be an initial symptom of multiple sclerosis: a) blurred or double vision b) fatigue that worsens in the afternoon c) impaired memory and attention d) heaviness, weakness, or loss of sensation in the legs

Multiple sclerosis (MS) is caused by demyelination of nerve fibers in the central nervous system. Although its symptoms vary from individual to individual, the progression of symptoms usually follows a predictable pattern. See, e.g., Multiple Sclerosis - Symptoms (http://www.drkoop.com/ency/93/guides/000017_3.html). a. Incorrect Visual problems are common initial symptoms of MS. b. Incorrect Fatigue (especially fatigue that worsens in the afternoon) is a common initial symptom of this disorder. C. CORRECT It is estimated that up to 70% of people with MS develop some type of cognitive dysfunction, and this most often involves impairments in attention, recent memory, and information processing speed. While some of these individuals experience cognitive problems in the early stages of the disorder, the majority do not do so until they have had the disease for an extended period. Therefore, of the answers given, this one is the best response. d. Incorrect Sensory and motor problems are initial signs of MS and most often affect the legs.

A middle-aged man with prosopagnosia: a) cannot draw a picture of a face when asked to do so b) does not recognize a picture of a face as a face c) cannot distinguish one face from another d) forgets to shave and wash his face

Prosopagnosia is the inability to recognize or identify individual faces. C. CORRECT A person with prosopagnosia is unable to recognize familiar faces including, in some cases, his/her own face in a mirror or picture.

Many antipsychotic drugs are believed to exert their therapeutic effects primarily by blocking D2 and other dopamine receptors. An exception is: a) haloperidol b) flupentixol c) perphenazine d) clozapine

Of the antipsychotic drugs listed in the responses, only one is considered to be atypical because of its effect on the nervous system. D. CORRECT Clozapine seems to have limited effects on D2 receptors and, instead, affects serotonin receptors and, to some degree, D4 receptors.

The _________ is to the conscious perception of odors as the __________ is to olfactory memory: a) insular cortex; septum b) orbitofrontal cortex; amygdala c) somatosensory cortex; thalamus d) cingulate cortex; medulla

Olfaction is mediated by several structures of the brain including the amygdala, limbic cortex (pyriform and entorhinal cortex), and the orbitofrontal cortex. a. Incorrect The insular cortex (insula) is part of the limbic system and is involved in emotions and pain. The septum is also part of the limbic system; it controls emotions and has been identified as a "pleasure area" of the brain. B. CORRECT The orbitofrontal cortex contains the secondary and tertiary olfactory cortical areas and is involved in the conscious perception of odors; while the amygdala plays an important role in olfactory memory. See, e.g., T. W. Buchanan, D. Tranel, and R. Adolphs, A specific role of the human amygdala in olfactory memory, Learning and Memory, 2003, 10, 319-325. c. Incorrect The somatosensory cortex is located in the parietal lobe and processes information related to touch. Also, olfaction is the only sensory system that does not send incoming signals directly to the thalamus. d. Incorrect The cingulate cortex is part of the limbic system and has been linked to several functions including pain perception and reward-based decision making. The medulla is part of the hindbrain and mediates vital bodily functions such as breathing and heartbeat.

A researcher administers a drug to Bonzo, a monkey, that will inhibit RNA synthesis during and some time after Bonzo is learning a simple task. This will most likely have which of the following effects: a) impair Bonzo's ability to learn the task at the time of training b) impair Bonzo's ability to perform the task on a subsequent day c) increase Bonzo's emotional reactivity for a period of time after learning the task d) decrease Bonzo's emotional reactivity while learning the task

One of the current theories about long-term memory is that formation of those memories requires protein synthesis. B. CORRECT RNA is necessary for protein synthesis, so inhibiting RNA synthesis will also inhibit protein synthesis. Because protein synthesis is necessary for long-term memory, when it is not allowed to occur, Bonzo will be unable to form long-term memories and, therefore, be unable to recall the task at a later time. (Protein synthesis does not, however, seem to be necessary for short-or intermediate-term memory and inhibiting RNA synthesis would not interfere with the performance of the task at the time of training.)

Opioid peptides that block the release of substance P and thereby reduce pain are referred to as: a) hypnotics b) endorphins c) pheromones d) catecholamines

Opiate receptors and substance P receptors are concentrated in the same areas of the brain. Opiate receptors play an important role in inhibiting the pain-producing effects of substance P. B. CORRECT The opioid peptides (which bind to opiate receptor sites) are also known as endorphins (endogeneous morphines). The endorphins produce analgesia, most likely by blocking the release of substance P.

__________ circuit was proposed in 1937 as a neural circuit that mediates the experience and expression of emotion.

Papez's

The changes in cognitive flexibility, memory, and everyday behavior (e.g., loss of spontaneity and lack of initiative) that are characteristic of Parkinson's Disease lend support to the hypothesis that this disorder involves subcortical systems that influence the: a) frontal lobes b) temporal lobes c) occipital lobes d) parietal lobes

Parkinson's Disease is characterized by neuron loss in the substantia nigra and other brain-stem nuclei and the presence of Lewy bodies in the neurons that remain in these regions. A. CORRECT A number of patients with Parkinson's Disease develop dementia, which is currently believed to be of the "subcortical type". There is some evidence that damage to these subcortical areas produces dementia because of their influence on the frontal cortex.

__________ seizures begin in one side of the brain and affect one side of the body, at least initially.

Partial

When partial seizures originate in the temporal lobe, symptoms are most likely to include: a) a sense of déjà vu b) "speech arrest" c) tingling or warmth on one side of the body d) flashing lights or strange colors

Partial (focal) seizures are due to abnormal electrical activity in one region of the brain. A. CORRECT Typical symptoms of seizure activity in the temporal lobe are a change in perception, a sudden alteration in emotions (e.g., fear, sadness, happiness), and/or a sense of déjà vu. b. Incorrect "Speech arrest" (an inability to speak) and other motor symptoms are associated with seizure activity in the frontal lobe. c. Incorrect Unusual bodily sensations - e.g., tingling or warmth on the side of the body opposite the affected lobe - are caused by abnormal electrical activity in the parietal lobe. d. Incorrect Flashing lights or strange colors are symptoms of abnormal activity in the occipital lobe.

If an adult takes phenobarbital every night for six weeks and then abruptly stops taking the drug, what will be the likely result: a) she will not experience any change in REM sleep b) she will experience an increase in REM sleep c) she will experience a decrease in REM sleep d) she will experience an increased need for sleep

Phenobarbital, a barbiturate, was originally used as a sleeping aid and an anticonvulsant. Due to the recognition of their untoward side effects and the development of safer drugs, barbiturates are now rarely prescribed. B. CORRECT Barbiturate use causes a decrease in REM sleep and abrupt cessation of use can cause an "REM rebound," as well as an inability to sleep, poor quality sleep, and nightmares.

Which of the following statements is most consistent with the current understanding of post-concussional syndrome (PCS): a) The more severe the head trauma, the more severe and the longer the duration of the symptoms of PCS. b) Most patients who develop PCS never fully recover from the motor and other physical symptoms of the disorder. c) The symptoms of PCS are often due to a combination of organic and psychological factors. d) In most cases, PCS is purely psychological in nature and motivated by a desire for financial compensation.

Post-concussional syndrome (PCS) refers to a constellation of somatic and psychological symptoms associated with head trauma, especially mild head trauma. Symptoms include headache, dizziness, fatigue, impaired memory and concentration, depression, irritability, visual impairment, and sleep disturbances. a. Incorrect PCS most commonly occurs in cases of mild traumatic brain injury (with up to 50% of people with mild brain injury having symptoms of PCS) and is rare in cases of moderate to severe brain injury. b. Incorrect This is the opposite of what is true: Most patients with PCS experience full recovery, with symptoms resolving within three to six months of the trauma. See, e.g., L. M. Binder, A review of mild head trauma. Part 2: Clinical implications, Journal of Clinical and Experimental Neuropsychology, 1997, 19, 432-457. C. CORRECT Although the etiology of PCS continues to be debated, the experts generally agree that it has both physiological and psychological origins. Psychological factors that contribute to PCS include the individual's subjective interpretation of the injury, premorbid personality characteristics, desire for secondary gain, and access to social support. See, e.g., L. M. Ryan and D. M. Warden, Post concussion syndrome, International Review of Psychiatry, 2003, 15(4), 310-316. d. Incorrect The majority of individuals with PCS are not seeking financial compensation (i.e., are not involved in litigation). Moreover, the deficits associated with the disorder are often more serious than the individual realizes, and there is evidence that, at least some symptoms, have a physiological cause.

The duration of post-traumatic amnesia (PTA): a) is unrelated to the severity of the injury b) is useful as an indicator of severity only when combined with the degree of retrograde amnesia c) is less accurate as an indicator of severity than the degree of retrograde amnesia d) is more accurate as an indicator of severity than the degree of retrograde amnesia

Post-traumatic amnesia (PTA) is a common consequence of closed-head injury. a. Incorrect PTA has been found to be a fairly good predictor of the extent of injury. b. Incorrect PTA, which is a measure of anterograde amnesia, has been found to be a better predictor than retrograde amnesia. c. Incorrect See explanation for response "b." D. CORRECT The length of the post-traumatic (anterograde) amnesia has been found to correlate with the severity and long-lasting consequences of brain injury.

Propranolol hydrochloride (Inderal) is most likely to be prescribed as a treatment for which of the following? a) hypotension b) essential tremor c) bronchial asthma d) diabetes

Propranolol hydrochloride is a beta-blocker and is used to treat cardiovascular disorders, tremors, migraine headaches, and the physical symptoms of anxiety. a. Incorrect Propranolol HCl is used to treat hypertension and is contraindicated for hypotension. b. CORRECT Essential tremor (tremor associated with purposeful movement) is one of the conditions that propranolol HCl is used to treat. c. Incorrect Propranolol HCl is contraindicated for bronchial asthma. d. Incorrect Propranolol HCl is not used as a treatment for diabetes.

The most likely side effects of propranolol (Inderal) are: a) bradycardia and depression b) tachycardia and mania c) tachycardia and tremor d) bradycardia and hypersomnia

Propranolol is a beta-blocker. Beta-blockers have been used to treat several medical conditions including hypertension and irregular heartbeat. They have also been found useful for reducing the physical symptoms of anxiety. A. CORRECT The side effects of propranolol (Inderal), which are often transient, include bradycardia, depression (as manifested by insomnia, lassitude and fatigue), nausea and vomiting, and light-headedness.

Damage to which of the following areas of the brain is most likely to cause deficits in prospective memory: a) prefrontal region b) occipito-temporal region c) corpus callosum d) parietal association cortex

Prospective memory refers to the ability to remember to perform an intended action at a particular point of time in the future - e.g., to give your colleague a telephone message the next time you see her. A. CORRECT Prospective memory has been linked to the prefrontal lobes, and it has been hypothesized that executive cognitive functions (which have also been linked to this area of the brain) are involved in this aspect of memory. See, e.g., M. McDaniel et al., Prospective memory: A neuropsychological study, Neuropsychology, 1999, 13(1), 103-110. b. Incorrect Although there is some evidence that the medial temporal region may be involved in prospective memory, the occipito-temporal area has not been implicated. c. Incorrect The corpus callosum has not been linked to prospective memory. d. Incorrect The parietal association cortex has not been linked to prospective memory.

A patient who has recently been prescribed Prozac (fluoxetine) as a treatment for depression: a) should be warned that she may temporarily experience some minor cognitive impairment b) should be advised that she may experience dry mouth, constipation, blurred vision, and other anticholinergic effects c) should be warned that sleep and anxiety problems may temporarily increase d) must avoid tyramine-rich foods

Prozac and the other SSRIs are associated with fewer side effects than the tricyclics. a. Incorrect Cognitive impairment is more likely with a tricyclic. b. Incorrect Unlike the tricyclics, the SSRIs do not cause anticholinergic symptoms. C. CORRECT One disadvantage of the SSRI's is that they may temporarily exacerbate sleep and anxiety problems. d. Incorrect This is true about the MAOIs, not the SSRIs.

According to Weber's law, the more intense a stimulus, the greater the increase in stimulus intensity required for the increase to be perceived. In other words, Weber's law deals with: a) just noticeable differences. b) absolute thresholds. c) stimulus intensities. d) absolute limens.

Psychophysics is the study of the relationship between physical stimulus magnitudes and their corresponding psychological sensations. There are several psychophysical laws that you should be familiar with for the exam. a. CORRECT A just noticeable difference is the minimum difference in stimulus intensity required for an individual to notice a difference from the previous intensity. For instance, if you are in a room lit by a 120 watt light bulb, and you don't notice a difference in brightness until the wattage is increased to 130, the just noticeable difference is 10 watts. Weber's law states that the more intense a stimulus, the greater the increase in stimulus intensity must be for the increase to be noticed. b. Incorrect An absolute threshold is the minimum stimulus intensity required to produce a specific sensation. For example, if you don't notice any light in a room unless the wattage of a light bulb is two or more, then two watts is the absolute threshold. c. Incorrect Stimulus intensities are measured in terms such as watts, decibels, and Hz. While Weber's law is concerned with stimulus intensities, it more specifically deals with just noticeable differences. d. Incorrect Absolute limen is another term for absolute threshold.

A psychophysicist is measuring "just noticeable differences" while investigating the relationship between changes in the intensity of light and subjects' perceptions of those changes. The measuring scale being used has: a) physically equal intervals b) psychologically equal intervals c) physically and psychologically equal intervals d) physically and psychologically unequal intervals

Psychophysics is the study of the relationship between stimulus magnitude and perception of that stimulus. B. CORRECT Psychophysicists use just noticeable differences (JNDs) and similar measurements to study the relationship between changes in physical stimuli and the psychological responses to those changes. JNDs are considered to be equal; however, the corresponding physical stimuli usually are not. For example, it may take only the addition of one pound to notice a difference when you start with ten pounds but the addition of ten pounds to notice a difference when you start with 100 pounds. In each case, the JND is equal to one, but the physical difference in weight is not equal.

The research suggests that RNA (ribonucleic acid) plays an important role in which of the following? a) the sleep-wake cycle b) learning and memory c) creativity d) motivation

RNA is involved in protein synthesis and other cell activities. b. CORRECT Some investigators suggest that the coding of memory may involve changes in protein molecules and, consequently, that RNA acts as a chemical mediator for learning and memory. RNA has not been linked to the sleep-wake cycle, creativity, or motivation.

Sleep deprivation is not consistently linked with any specific abnormalities, but REM sleep deprivation is usually followed by an REM __________.

Rebound

Research investigating the usefulness of fluoxetine (Prozac) for treating bulimia suggests that level of serotonin: a) is not an etiological factor b) may be an etiological factor for females only c) may be an etiological factor for males only d) may be an etiological factor

Recent research on the biological causes of eating disorders has focused on the role of the neurotransmitters. D. CORRECT Several neurotransmitters have been implicated in the eating disorders, but the greatest amount of evidence links low serotonin levels to bulimia. For example, fluoxetine and other SSRIs have been found useful for alleviating the symptoms of bulimia.

Recovery of cognitive ability following head trauma follows a predictable pattern in most patients. For example, with regard to disorientation, recovery usually occurs in the following sequence: a) orientation to personal information, then to place, and finally to time b) orientation to time and place followed by orientation to personal information c) orientation to place, then to personal information, and finally to time d) orientation to personal information, then to time, and finally to place

Recovery of cognitive functions following head trauma usually follows a set pattern. For example, with regard to retrograde amnesia, the most remote memories return first. A. CORRECT Orientation to personal information is recovered first, followed by place and then time. Patients who are still disoriented to personal information rarely exhibit orientation to time or place.

Tourette Syndrome, Autistic Disorder, and Obsessive-Compulsive Disorder have been most consistently linked to pathology in which areas of the brain: a) amygdala and parietal lobes b) thalamus and parietal lobes c) basal ganglia and frontal lobes d) thalamus and frontal lobes

Research using brain imaging techniques has consistently linked abnormalities in certain areas of the brain to Tourette Syndrome, Autistic Disorder, and OCD. C. CORRECT Abnormalities in the basal ganglia and frontal lobes have been implicated in all three disorders.

The frontal lobes mediate executive functions as well as complex emotions and behaviors. A person with damage to which area of the frontal lobes is most likely to engage in disinhibited, impulsive behaviors and exhibit emotional lability, distractibility, and poor judgment and insight: a) medial frontal area b) anterior cingulate area c) dorsolateral prefrontal area d) orbitofrontal area

Research using neuroimaging techniques has pinpointed the location of specific emotional and behavior changes caused by frontal lobe damage. a. Incorrect Damage to the medial frontal area produces akinesia, mutism, and weakness and loss of sensation in the lower extremities. b. Incorrect Damage to the anterior cingulate area is associated with apathy and paucity of speech and movement. c. Incorrect Damage to the dorsolateral prefrontal area produces impaired executive functioning, slowed information processing, and mood and personality changes. D. CORRECT Damage to the orbitofrontal areas produces pseudopsychopathy (disinhibition and impulsive behavior), euphoria, lack of judgment and social tact, and distractibility

The word HEART is projected onto a screen so that HE is projected to the subject's right visual field only while ART is projected to the subject's left visual field only. If the subject in the experiment is a "split brain" patient, you would expect him/her to report seeing: a) HE only b) ART only c) ART HE d) HE ART

Results of studies on split-brain patients have shown that the left hemisphere in most people is specialized for language. Don't forget that information in the right visual field is processed initially in the left hemisphere, while information in the left visual field initially goes to the right hemisphere. A. CORRECT This is the result reported by Sperry (1970). Because HE was projected to the right visual field, it was processed by the left hemisphere which is specialized for language in most people. Thus, split-brain patients could report seeing it. b. Incorrect When ART is projected to the right hemisphere of a split-brain patient, it cannot be passed to the left hemisphere and the patient cannot, therefore, say it. c. Incorrect Split-brain patients did not report reversals in Sperry's study. d. Incorrect Split-brain patients can only verbally report what is projected to the left hemisphere.

Depth perception depends on a combination of binocular and monocular cues. __________, one of the binocular cues, refers to the fact that our two eyes see objects in the world from two different views, and the closer an object, the greater the disparity of the two images.

Retinal disparity

An advantage of risperidone is that it: a) is less likely to produce extrapyramidal side effects than traditional antipsychotics b) is not associated with weight gain and sexual dysfunction c) does not produce a neuroleptic malignant syndrome d) has a rapid onset of beneficial effects

Risperidone is classified as an atypical antipsychotic. A. CORRECT Although extrapyramidal side effects may occur, they are much less common with risperidone than with the typical (traditional) antipsychotics. See, e.g., M. J. Owens & S. C. Risch (2001), Atypical antipsychotics, in A. F. Schatzberg & C. B. Nemeroff, Essentials of Psychopharmacology (pp. 125-154), Washington, DC, American Psychiatric Publ. b. Incorrect Weight gain and sexual dysfunction are common side effects of risperidone. c. Incorrect Risperidone can produce NMS, although this side effect is rare. d. Incorrect One problem with risperidone is that it has a delayed onset.

Administering Ritalin to a child to confirm or rule out a diagnosis of ADHD is: a) contraindicated since Ritalin increases activity levels and reduces attention in normal children b) not useful because only a minority of children with ADHD respond to Ritalin c) not useful since the effects of Ritalin are similar for children with and without ADHD d) useful, especially in cases where the child's symptoms are highly variable

Ritalin, a CNS stimulant, has been found to have a "paradoxical effect" in children and adults with and without ADHD. C. CORRECT Using a drug is usually not the best strategy for confirming or ruling out a diagnosis. It is not useful in ADHD because, as noted above, individuals with or without the disorder have a similar response to the drug.

There is evidence that the _____________ plays a role in the etiology of seasonal affective disorder (SAD). a) red nucleus b) suprachiasmatic nucleus c) tectum d) striatum

Seasonal affective disorder is a recurrent type of depression that occurs in the fall and winter months and has been linked to reduced exposure to daylight. a. Incorrect The red nucleus is a midbrain structure and is involved in motor coordination. b. CORRECT Knowing that the suprachiasmatic nucleus (SCN) mediates the sleep-wake cycle and other circadian rhythms may have helped you identify the correct answer to this question. The SCN's role in SAD is believed to be related to its impact on melatonin levels, which are generally higher in individuals with this disorder and are affected by exposure to light. c. Incorrect The tectum is part of the midbrain and is involved in auditory and visual processing. d. Incorrect The striatum is part of the basal ganglia and is involved in movement and certain cognitive processes.

_____________________ seizures begin in and are often restricted to the temporal lobe and involve impaired consciousness and automatisms such as lip smacking, chewing, fidgeting, and walking in circles: a) Grand mal b) Complex partial c) Myoclonic d) Absence

Seizures are categorized as "generalized" or "partial." Generalized seizures are caused by electrical impulses throughout the entire brain, while partial seizures are produced - at least initially - by electrical impulses in a specific area in the brain. a. Incorrect A grand mal seizure is a type of generalized seizure. It involves unconsciousness, convulsions, and muscle rigidity. B. CORRECT Complex partial seizures are called "complex" because they impair consciousness and "partial" because they begin in a limited area of the brain, most often the temporal lobe. Because of the involvement of the temporal lobe, these seizures are also known as temporal lobe epilepsy. c. Incorrect A myoclonic seizure is a type of generalized seizure. Its primary symptoms are brief jerky contractions of muscles in different parts of the body, most often in the arms and legs. d. Incorrect Absence seizures are a type of generalized seizure and involve a brief loss of consciousness with few or no other symptoms.

According to Hans Selye, the body's reaction to sustained stress can be described in terms of three phases: These phases, in order, are: a) arousal, readiness, fight or flight b) alarm, resistance, fight or flight c) arousal, readiness, exhaustion d) alarm, resistance, exhaustion

Selye referred to the body's physiological reaction to prolonged stress as the general adaptation syndrome (GAS). D. CORRECT These are the three stages described by Selye. Alarm refers to the "fight or flight" reaction, which is mediated by hypothalamic stimulation of the sympathetic branch of the autonomic nervous system. It is followed by the resistance phase, in which hypothalamic stimulation of the endocrine glands results in the release of several hormones including cortisol. Finally, if the stress continues, exhaustion occurs and the body's homeostatic mechanisms begin to fail.

Hans Selye's (1956) general adaptation syndrome predicts that we respond to stress with a predictable pattern that involves which of the following three stages? a) disorganization, mobilization, and adaptation b) exhaustion, accommodation, and recovery c) preparation, survival, and rebuilding d) alarm, resistance, and exhaustion

Selye's general adaptation syndrome (GAS) is based on the assumption that people respond to all types of prolonged stress in the same manner. a. Incorrect These have been identified as stages in reaction to a crisis. b. Incorrect These are not the three stages identified by Selye. c. Incorrect These have been identified as the three stages of responding to a disaster. d. CORRECT Alarm, resistance, and exhaustion are the three stages identified by Selye. Each is defined by different physiological responses that are mediated primarily by the adrenal and pituitary glands.

Severe deficits in remotely acquired semantic knowledge are most likely due to damage to the: a) frontal lobes. b) parietal lobes. c) temporal lobes. d) occipital lobes.

Semantic memory refers to memory for objects, facts, word meanings, and other types of general knowledge. C. CORRECT Impairments in semantic memory have been linked to damage to the hippocampus and certain areas of the temporal lobe, especially the anterolateral temporal lobe.

Elevated levels of __________ contribute to Schizophrenia, Autistic Disorder, and food restriction associated with Anorexia, while low levels play a role in aggression, depression, suicide, bulimia, PTSD, and obsessive-compulsive disorder.

Seratonin

__________ was introduced in the 1970s as a method for measuring regional cerebral blood flow using radioactive isotopes. a) MRI b) CT c) SPECT d) SHEFC

Several brain imaging techniques are available. Some provide information on brain structure, while others yield information on brain functioning (e.g., regional cerebral blood flow). a. Incorrect MRI is a structural brain imaging technique. b. Incorrect CT is also a structural technique. C. CORRECT SPECT (single proton emission computed tomography) is used to assess regional cerebral blood flow. d. Incorrect This "distractor" is the name of a brain imaging research center in Scotland.

According to Gustav Fechner, the relationship between ________________ is logarithmic in nature: a) consistency of reinforcement and strength of the reinforced behavior b) duration of the delay of symptom onset of PTSD and degree of risk for chronicity c) duration of exposure to stress and level of cortisol d) intensity of physical stimulation and intensity of psychological sensation

Several psychophysical laws have been proposed to explain the relationship between physical stimulus magnitude and the corresponding psychological sensation. D. CORRECT Based on his research, Fechner concluded that the intensity of a sensation equals a constant times the logarithm of the intensity of the stimulus.

The Sickness Impact Profile is used to: a) assess the impact of disease on physical and emotional functioning b) assess emotional reactions to chronic illness c) diagnose physical illnesses d) diagnose personality disorders

The Correct Answer is A The Sickness Impact Profile (SIP) is one of the most comprehensive quality of life measures available. It is used to assess the impact of disease on both physical and emotional functioning. Its focus is on behavioral measures of daily living such as sleep, eating, social interactions, and emotional behaviors. Contrary to "B," however, it is not used to assess emotional or other subjective reactions to illness.

Which of the following is an example of sexual dimorphism? a) The hypothalamus, corpus callosum, and several other regions of the human brain exhibit consistent gender-related differences in structure and activity. b) The gonads in human males and females are undifferentiated until six to eight weeks after conception. c) Sex hormones play a more important role in the adult sexual behavior of lower animals than in the adult sexual behavior of humans. d) The effects of spinal cord injury on sexual functioning depend on the severity and location of the injury.

Sexual dimorphism refers to systematic gender-related differences in physical characteristics. Recent research has confirmed that certain structures in the brain are sexually dimorphic. a. CORRECT The hypothalamus is one of the structures of the human brain that is sexually dimorphic. For example, the size of the hypothalamus and its secretion of growth hormone differs for males and females.

REM sleep is ordinarily preceded by a period of Stage 2 sleep except in: a) elderly adults b) infants c) insomniacs and "long-sleepers" d) infants and elderly adults

Sleep patterns vary somewhat over the lifespan. B. CORRECT In infants, an EEG indicates only two distinct sleep patterns: REM and NREM (quiet) sleep, and in the first few months, REM precedes NREM (the sleep period begins with REM sleep).

L-dopa acts as a: a) dopamine agonist b) dopamine antagonist c) dopamine neurotoxin d) dopamine metabolite

Some chemicals act as either agonists or antagonists in the brain. a. CORRECT L-dopa is a precursor to dopamine, which means that it is transformed to dopamine in the brain. An agonist is a chemical that increases the amount or action of a neurotransmitter. Consequently, L-dopa is a dopamine agonist. b. Incorrect An antagonist reduces the amount or availability of a neurotransmitter in the brain.

The amino acid __________ is one of the most important inhibitory neurotransmitters in the brain, and there is evidence that abnormalities in this neurotransmitter contribute to anxiety, sleep disorders, and seizures. a) dopamine b) GABA c) glutamate d) norepinephrine

Some neurotransmitters consistently exert either an excitatory or inhibitory effect on post-synaptic neurons, while others have different effects depending on their location in the nervous system. a. Incorrect Dopamine is a monoamine that has both excitatory and inhibitory effects on post-synaptic neurons depending on location. B. CORRECT GABA acts as an inhibitor at synapses throughout the brain and has been linked to the disorders listed in the question. c. Incorrect Glutamate is also an amino acid, but it is one of the most important excitatory neurotransmitters in the brain. d. Incorrect Norepinephrine is a monoamine and has both excitatory and inhibitory effects in the nervous system.

Sperry and Gazzaniga's (1967) research on "split-brain" patients provided evidence for which of the following? a) hemispheric specialization b) neural networks c) pleasure centers in the brain d) the fight-or-flight response

Sperry and Gazzaniga's "split-brain" patients had their corpus callosum severed in order to control severe epilepsy. A consequence of the surgery was that communication between the two hemispheres was reduced. a. CORRECT Results of the studies on these patients confirmed that each hemisphere specializes with regard to certain functions - e.g., in most people, the left hemisphere is dominant for language and logical, analytical thinking, while the right hemisphere is primarily responsible for spatial perception, holistic thinking, and creativity.

Spinal cord injury below the first thoracic vertebra is most likely to result in: a) Quadriplegia b) Paraplegia c) Hemiplegia d) Paralysis

Spinal cord injuries typically cause varying degrees of paraplegia or quadriplegia. a. Incorrect Quadriplegia is more likely to occur if the injury occurs above the first thoracic vertebra. B. CORRECT The 12 thoracic vertebrae lie below the cervical vertebrae and above the lumbar vertebrae. Injury at this level ordinarily does not affect the upper limbs but is associated with paraplegia, which involves motor and/or sensory loss in the lower limbs. c. Incorrect Hemiplegia involves only one side of the body and is not associated with injury below the first thoracic vertebra. d. Incorrect Paralysis may or may not be the result of a spinal cord injury.

In terms of total sleep time, the proportion of Stage 4 sleep in adulthood: a) increases from early to late adulthood. b) decreases until middle age (about age 45) and thereafter stabilizes. c) remains stable until late adulthood (age 70) and thereafter increases. d) decreases from early to late adulthood.

Stage 4 sleep is the deepest level of non-REM sleep. Like the other sleep stages, it follows a predictable pattern over the lifespan. d. CORRECT Stages 3 and 4 (especially Stage 4) sleep show a gradual decline over the lifespan, with older adults spending most of their sleep time in Stages 1 and 2 sleep.

There is evidence that, as a group, Asians have unique responses to psychotropic medications in terms of both therapeutic and side effects. Specifically, in comparison to Caucasian patients, Asian patients often: a) require lower doses to obtain the same therapeutic effects and experience more severe side effects from the same dose b) require lower doses to obtain the same therapeutic effects but experience less severe side effects from the same dose c) require higher doses to obtain the same therapeutic effects but experience more severe side effects from the same dose d) require higher doses to obtain the same therapeutic effects and experience less severe side effects from the same dose

Studies have shown that, in some cases, the effects of the psychotropic drugs are related to race/ethnicity. A. CORRECT As a group, Asians are more sensitive to psychotropic drugs in terms of both therapeutic and side effects. For this reason, the experts generally agree that the best course of action, when starting an Asian patient on a new drug, is to begin with a lower dosage than is recommended for Caucasian patients. See e.g., K. M. Lin and F. Cheung, Mental health issues for Asian Americans, Psychiatric Services, 1999, 50(6), 774-780.

__________ is a rare condition in which the stimulation of one sensory modality triggers a sensation in another sensory modality.

Synesthesia

Synesthesia is best conceptualized as: a) a learned phenomenon b) the result of imagination c) a physical process in the brain d) a manifestation of psychosis

Synesthesia occurs when two or more senses are automatically and involuntarily joined so that stimulation of one sense automatically elicits another (e.g., the sound of a voice elicits a specific color). c. CORRECT Although the cause of synesthesia is not clear, it is believed to be due to brain functioning. One theory attributes it to unusually dense connections between sensory regions of the brain.

Recent studies suggest that, for patients who develop tardive dyskinesia as a result of long-term neuroleptic use: a) symptoms are always irreversible b) symptoms are alleviated with a dopamine agonist in most cases c) symptoms may eventually improve to some degree following neuroleptic withdrawal d) symptoms worsen over time following neuroleptic withdrawal even when withdrawal is gradual

Tardive dyskinesia is a serious side effect of the antipsychotic drugs. C. CORRECT Although early research suggests that tardive dyskinesia is irreversible, recently-published longitudinal studies indicate that, in 30 to 60% of cases, symptoms improve after five years following withdrawal of the drug. This is especially true for younger people and when the drug is withdrawn gradually. Note also that continued use of a low-dose of neuroleptics can, at least temporarily, suppress the symptoms of tardive dyskinesia.

Spinal cord injury at the ________ level is most likely to result in quadriplegia. a) lumbar b) sacral c) cervical d) thoracic

The 31 pairs of spinal nerves are divided into five groups or regions. These are, from the top to the bottom of the spinal cord, cervical, thoracic, lumbar, sacral, and coccygeal. a. Incorrect Damage at the lumbar level is likely to cause some loss of functioning in the hips and legs. b. Incorrect Damage at the sacral level also causes loss of functioning in the hips and legs. c. CORRECT Quadriplegia (the loss of sensory and motor functioning in the arms and legs) results from damage at the cervical level. d. Incorrect Paraplegia (loss of functioning in the legs) is caused by damage at the thoracic level.

Symptoms of Bipolar Disorder are typically treated with lithium and/or any of the following medications EXCEPT: a) phenelzine b) carbamazepine c) divalproex d) valproic

The Correct Answer is A A. Currently, lithium and/or an anti-seizure medication is the drug treatment-of-choice for Bipolar Disorder. Unlike the others, phenelzine is not used for the treatment of Bipolar Disorder as it is an MAOI antidepressant. Carbamazepine (Tegretol), divalproex (Depakote) and valproic acid (Depakene) are anti-seizure drugs that have been found to be useful for treating patients with Bipolar Disorder who have not responded well, or cannot tolerate, other treatments.

The areas of the brain associated with declarative memory include all of the following except the: a) striatum b) diencephalon c) peripheral cortex d) hippocampus

The Correct Answer is A A. Declarative memory consists of both episodic memory, past and personally experienced events, and semantic memory, knowledge of the meaning of words and how to apply them. In general, declarative memory is associated with the hippocampus (d.) and temporal cortex. More specifically, declarative memory, primarily involves the diencephalon (b.). The diencephalon, comprised of the thalamus and hypothalamus, is located between the two hemispheres of the brain, sometimes referred to as the "between brain," or the anterior to midbrain regions. The diencephalon contains important connections and relays for sensory, motor as well as limbic pathways that are involved with memory. Additional areas that are associated with declarative memory are the peripheral cortex (c.), the amygdala and the neocortex. In particular, this includes the right frontal & temporal lobes for the episodic component, and the temporal lobes for the semantic component. The striatum (a.) controls various muscular activities such as walking and balance. (See: Long, C. J. (2000). Neuropsychology and Behavioural Neuroscience. Memory, March 30, 2000.).

Tumors, head injuries, viral infection, vascular or other neurological disorders that damage the medial temporal region of the brain affect: a) retrograde and anterograde memory b) retrograde memory and general intellectual abilities c) anterograde memory and general intellectual abilities d) retrograde and anterograde memory and general intellectual abilities

The Correct Answer is A. Neurological disorders that damage the medial temporal region can cause organic amnesia. In organic amnesia, anterograde amnesia (an impaired ability to form new permanent memories) is often the outstanding cognitive impairment, though retrograde amnesia (impairment of memory for events that occurred before the onset of amnesia) can also occur. Despite the severe impairment in new learning ability, general intellectual or language abilities and short-term memory are generally intact.

Naltrexone (ReVia, Depade), an opiate antagonist, has been approved to treat alcohol abuse disorders by reducing the desire for alcohol after drinking has stopped and blocking the reinforcing effects or parts of the brain that "feel" pleasure when using alcohol. The most commonly experienced sides effects associated with naltrexone are: a) nausea, headache, insomnia, and anxiety. b) abdominal cramping, night sweats, blurred vision, shortness of breath. c) blurred vision, dizziness, excessive perspiration and gastrointestinal pain. d) shortness of breath, increased dreaming, nausea, diarrhea.

The Correct Answer is A A. Naltrexone, when used as an adjunct to psychosocial treatments for alcoholism, can provide an important period of sobriety, during which an individual learns to stay sober without it. Naltrexone therapy has been shown to improve treatment outcomes, reducing the percentage of days spent drinking (after stopping), the amount of alcohol consumed on a drinking occasion and relapse to excessive and destructive drinking. In other words, naltrexone is likely to reduce the urge or craving to drink and the risk of returning to heavy drinking. While individuals on naltrexone are less likely to relapse to heavy drinking following a lapse in abstinence, naltrexone does not make people "sober up", does not alter alcohol's acute effects on cognitive functioning and is approved as an adjunct to psychosocial treatment not a replacement. Unlike disulfiram (antabuse), naltrexone does not appear to alter the absorption or metabolism of alcohol and does not have major adverse effects when combined with alcohol although some have increased nausea caused by drinking alcohol while taking naltrexone. Common side effects noted at the initiation of treatment in approximately 10 percent of individuals include nausea, headache, dizziness, fatigue, nervousness, insomnia, vomiting, constipation and anxiety.

Of the following, which would be most useful for reducing neuropathic pain? a) amitriptyline b) atomoxetine c) olanzapine d) fluoxetine

The Correct Answer is A A. Neuropathic pain is chronic pain that is due to a nervous system injury or dysfunction. Analgesics, including certain antidepressants, anticonvulsants, opioids, and local anesthetics are the first-line treatments for neuropathic pain. Antidepressants such as tricyclics may follow the failure of conventional analgesics or be used in addition to conventional analgesics. Tricyclic antidepressants, such as amitriptyline (Elavil), have been found to be most effective particularly if the pain is neuropathic (pain in a numb area, burning or shooting pain) or one of the headache syndromes. SSRI antidepressants, such as fluoxetine (d.), have been found to be effective at preventing headaches, including migraines, but are considered less effective than tricyclics and since they don't have analgesic properties are not useful for alleviating neuropathic pain. Atomoxetine (b.), a SNRI often used as a non-stimulant treatment for ADHD and neuroleptics, like olanzapine (c.), are also not useful for reducing neuropathic pain.

Post-concussional syndrome (PCS): a) refers to somatic and psychological symptoms associated with head trauma. b) refers to somatic symptoms due to psychological factors following head trauma. c) is associated with cases of severe head trauma. d) is associated with poorer prognosis for full recovery.

The Correct Answer is A A. Post-concussional syndrome (PCS) refers to somatic and psychological symptoms associated with head trauma including dizziness, impaired memory and concentration, headache, fatigue, depression, irritability, visual impairment, and sleep disturbances. Symptoms of PCS are experienced by up to 50% of individuals with mild brain injury and it is rare in moderate to severe cases (c.). The majority of individuals with post-concussional syndrome fully recover with their symptoms resolving within three to six months of the trauma. Psychological and physiological factors appear to cause the symptoms of PCS and the associated deficits are often more serious than the individual realizes. Psychological factors include: premorbid personality characteristics, the subjective interpretation of the injury, access to social support and desire for secondary gain. (See: Ryan, L. M. & Warden, D.M. (2003). Post concussion syndrome, International Review of Psychiatry,15(4), 310-316.)

Of the following, which is most frequently used to detect brain damage in the early stages of emergency room (ER) treatment? a) CT b) EEG c) PET d) MRI

The Correct Answer is A A. The CAT (Computerized Axial Tomography) or CT scan, and the Magnetic Resonance Imaging (MRI) take a series of images at different levels of the brain giving information and a direct visualization of structures and features. The MRI does this with magnetic fields; the CAT scan uses x-rays. The CAT scan is superior to the MRI in detecting fresh blood in and around the brain and often is repeated to ensure that a brain injury is not becoming more extensive, usually in the early stages of ER treatment. Another difference is CT scans cost substantially less than MRIs. In general, the MRI (d.) provides more detail than the CAT scan and can be used in examining the central nervous system, as well as, used to identify tumors, strokes, degenerative diseases, inflammation, infection, and other abnormalities in organs and other soft tissue of the body. A procedure that uses electrodes on the scalp to record electrical activity of the brain is an Electroencephalogram (EEG) (b.). It is used for detection of epilepsy, coma, and brain death. A PET (Positron Emission Tomography) scan measures metabolic processes, thus allowing an appraisal of how the brain is functioning. It tracks natural compounds, such as glucose, as the brain metabolizes them. By showing the areas of different metabolic activity, it then becomes easier to make diagnoses, such as determining the areas responsible for epileptic seizures.

Depression and chronic pain are both associated with which of the following neurotransmitters? a) norepinephrine b) epinephrine c) acetylcholine d) dopamine

The Correct Answer is A A. The brain pathways that process pain signals use some of the same neurotransmitters that are involved in the regulation of mood, notably norepinephrine and serotonin. Pain, as well as anxiety, sadness, and hopelessness, are intensified when regulation fails and both chronic pain and chronic depression can alter the nervous system functioning and perpetuate itself. Antidepressants, especially tricyclic antidepressants (TCAs), have proved to be effective in the treatment of diabetic neuropathy, fibromyalgia, chronic low back pain, chronic headache and postherpetic neuralgia. Tricyclics heighten the activity of the neurotransmitters norepinephrine and serotonin; SSRIs act more selectively on serotonin. Amitriptyline (Elavil), a tricyclic, is one of the antidepressants most often recommended as an analgesic, partly because its sedative qualities.

Disinhibition, indifference, and jocularity are most likely caused by damage to the: a) right frontal lobe b) left frontal lobe c) corpus callosum d) left parietal lobe

The Correct Answer is A Disinhibition/impulsiveness, happy indifference, and jocularity frequently result from damage to the right frontal lobe. Damage to the left frontal lobe (B) most often results in reduced speech, depression, and apathy.

In a patient with anterograde amnesia, which of the following functions is most likely to remain relatively unimpaired? a) ability to remember new events a day after they occur. b) ability to repeat a set of 5 digits read to him by an experimenter. c) ability to recall a story two hours after it is told to the patient. d) ability to recognize a doctor encountered two hours after an initial meeting.

The Correct Answer is B Anterograde amnesia refers to loss of memory for events that occur after the event (e.g., disease or trauma) that causes the amnesia. In other words, people with anterograde amnesia cannot form new memories. Specifically, the term refers to inability to form new long-term memories. Therefore, the person will lack memory for information learned hours or a day ago. By contrast, short-term memory (also referred to as immediate or working memory), which is the capacity to store 5 to 9 items of information for about 30 seconds, is usually not affected.

Premature ejaculation is most effectively treated with which of the following medications? a) risperidone b) fluoxetine c) lithium d) bupropion

The Correct Answer is B B. An estimated 30 to 40 percent of the male population is affected by premature ejaculation (PE), which is categorized into two types based on onset of the problem. The most common form is primary premature ejaculation, with symptoms having been present throughout adulthood. Secondary premature ejaculation begins during adulthood in a man who hasn't experienced the problem before and is sometimes associated with a neurological disorder. The most frequently used therapy for PE are SSRIs and some tricyclic antidepressants as it is well-established they often induce a variable delay in the ejaculatory reflex by increasing central serotoninergic transmission. Fluoxetine (Prozac) is the best choice because it is the only SSRI listed. The only other antidepressant choice in this question is bupropion (Welbutrin), which is an atypical antidepressant known for not causing the same sexual side effects as the SSRIs. In fact, there is some evidence that bupropion increases sexual responsiveness. (See: Waldinger, M.D. (2005). Lifelong premature ejaculation: definition, serotonergic neurotransmission and drug treatment. World Journal of Urology. 23:102 -108.)

Perseverating is an indication of damage in the: a) Temporal lobe b) frontal lobe c) parietal lobe d) occipital lobe

The Correct Answer is B B. Difficulty in interpreting feedback from the environment is one of the most common characteristics of frontal lobe damage. Perseverating or uncontrollable repetition of a particular response, risk taking, lack of social consciousness and non-compliance with rules are examples of this type of deficit.

Kluver-Bucy Syndrome is associated with damage to the: a) hippocampus b) amygdala c) septum d) thalamus

The Correct Answer is B B. Kluver-Bucy Syndrome is associated with lesions in the amygdala and characterized by reduced fear and aggression, increased acquiescence, and hypersexuality. Damage to the amygdala, which normally attaches emotional significance to information received from the senses, can also produce a lack of emotional response to stimuli. You may have considered response "C" since the septum inhibits emotionality; however, animal research on this structure indicates lesions produce hyper-emotionality and vicious behavior, known as "septal-rage syndrome." The hippocampus is associated more with memory than emotion and the thalamus functions as the central relay for all sensory information to and from the cortex except olfaction.

Which of the following forebrain structures are located around the brain stem and are important for motivated and emotional behavior? a) basal ganglia b) limbic system c) tegmentum d) reticular formation

The Correct Answer is B B. The limbic system is a set of structures controlling motivated and emotional behaviors such as eating, drinking, sexual behavior and aggression. Main structures of the limbic system include the olfactory bulb, the hypothalamus which regulates motivational behaviors, the pituitary gland which controls the timing and amount of hormone release, and the hippocampus which is associated with learning and memory. The basal ganglia (a.) codes and relays information associated with the control of voluntary movement. The tegmentum (c.) is one of two regions of the "midbrain" and contains the substantia nigra which is part of the brain's sensorimotor system. The reticular formation (d.) is a group of nerve fibers located inside the brainstem which controls sleep, arousal, and attention.

An individual taking clozapine begins exhibiting symptoms of muscle rigidity, tachycardia, hyperthermia, altered consciousness, and autonomic dysfunction. In this case, the best course of action would be to: a) reduce the dose gradually until the symptoms are alleviated b) stop the drug immediately and administer electrolytes and fluids c) switch to a traditional antipsychotic drug d) check to see what other drugs the patient is taking since clozapine does not produce these symptoms

The Correct Answer is B B. The symptoms listed are characteristic of neuroleptic malignant syndrome (NMS), which is a rare, but serious and potentially lethal syndrome that can result from the use of any neuroleptic or antipsychotic drugs. The syndrome commonly develops within the first 2 weeks of treatment in most cases however it may develop any time during drug therapy and can also occur in people taking anti-Parkinsonism drugs known as dopaminergics if those drugs are discontinued abruptly. All antipsychotics, typical or atypical, may precipitate the syndrome although potent neuroleptics (e.g., haloperidol, fluphenazine) are more frequently associated with NMS. Other agents associated with NMS include prochlorperazine (Compazine), promethazine (Phenergan), clozapine (Clozaril), and risperidone (Risperdal) as well as non-neuroleptic agents that block central dopamine pathways such as metoclopramide (Reglan), amoxapine (Ascendin), and lithium. Generally, intensive care is needed. The neuroleptic or antipsychotic drug is discontinued, and the fever is treated aggressively. A muscle relaxant may be prescribed. Dopaminergic drugs, such as a dopamine agonist, have been reported to be useful.

Dementia due to Head Trauma a) is usually progressive in cases of moderate to severe trauma. b) is usually progressive only in cases of repeated head trauma. e) unlike other forms of Dementia, does not usually involve disturbances in executive functions. d) unlike other forms of Dementia, is associated more with deficits in executive functions than with memory impairment.

The Correct Answer is B Dementia due to Head Trauma is usually nonprogressive when it is the result of a single head trauma. If it is progressive, this usually indicates a superimposed condition (e.g., hydrocephalus).

The brain system that mediates the general adaptation syndrome identified by Selye is the a) hypothalamic-hippocampal-prefrontal axis. b) hypothalamic-pituitary-adrenal axis. c) thalamic-hippocampal-limbic axis. d) thalamic-limbic-prefrontal axis.

The Correct Answer is B General adaptation syndrome is a model of stress put forth by Hans Selye in the 1930s. According to this model, exposure to stressful stimuli causes a predictable series of reactions, even if the stressor is positive. He divided these reactions into three phases: alarm, resistance, and exhaustion. In the alarm phase, the stress is identified and adrenaline is produced in order to trigger the fight-or-flight response. The resistance phase involves utilizing the body's resources in an attempt to cope with the stress. And in the exhaustion phase, the body's resources are depleted and the body cannot maintain its normal function. Autonomic nervous symptoms such as sweating and increased heart rate may reappear, and, if this phase is extended, long-term problems such as ulcers or depression could result. The hypothalamic-pituitary-adrenal axis (HPA axis) refers to physiological interactions and feedback among the hypothalamus, pituitary gland, and the adrenal gland. This axis regulates reactions to stress, including the general adaptation system. It is also involved in regulating other processes, including digestion, the immune system, emotions, and sexuality.

In most patients, the earliest signs of Huntington's disease are ·a) apathy and disorientation to time and place. b) depression and forgetfulness. c) athetosis and chorea. d) slowed movements and resting tremor.

The Correct Answer is B In Huntington's disease, personality and cognitive changes often precede motor symptoms. Athetosis and chorea (answer C) are characteristic of this disorder but are late-appearing. Slowed movements and resting tremor (answer D) are characteristic of Parkinson's disease.

Deficits in organizational ability, decision-making, and problem-solving are most likely to be the result of damage to the a) corpus callosum. b) prefrontal lobe. c) parietal lobe. d) basal forebrain.

The Correct Answer is B The activities listed in the question are "executive functions," which are mediated by the frontal lobes, or, more specifically, by the prefrontal lobes. (The basal forebrain connects with the temporal lobes and is involved in long-term memory.)

The effectiveness of thermal biofeedback as a treatment for migraine headaches is: a) less effective than relaxation therapy but more effective than electromyogram (EMG) biofeedback b) equally as effective as relaxation therapy c) more effective than relaxation therapy but less effective than electromyogram (EMG) biofeedback d) more effective than relaxation therapy

The Correct Answer is D Biofeedback, a form of complementary and alternative medicine (CAM) that falls under the category of mind-body therapies, refers to a technique that can give people better control over such body function indicators as blood pressure, heart rate, temperature, muscle tension, and brain waves. The National Institutes of Health (NIH) found thermal biofeedback, which measures skin temperature, to be more effective in treating migraines than relaxation therapy. EMG biofeedback, which measures muscle tension, is used as a treatment for tension headaches and data indicates it is equally effective as relaxation therapy. (See: National Institute of Neurological Disorders and Stroke (NIDS) of the National Institutes of Health (NIH). (2001). 21st Century Prevention and Management of Migraine Headaches. [Monograph]. Clinical Courier, 19(11).)

Broca's area is located in the: a) right temporal lobe b) right occipital lobe c) left parietal lobe d) left frontal lobe

The Correct Answer is D D. Broca's area is located in the left frontal lobe adjacent to the primary motor cortex. Wernicke's area is further back and lower, in the posterior part of the temporal lobe (a.). Broca's and Wernicke's areas are connected by a bundle of nerve fibers called the arcuate fasciculus. Damage to the arcuate fasciculus causes a disorder called conduction aphasia. The occipital lobe lies at the back of the head and is responsible for vision. A stroke or damage in the right occipital lobe (b.) can result in losing the left side of vision. The parietal lobe (c.) lies behind the frontal lobe and controls somatosensory information (sense of touch, temperature, pain).

Each of the following statements about SSRI discontinuation syndrome are true EXCEPT: a) Discontinuation symptoms have a typical onset of 1 to 3 days after use of the antidepressant is stopped and can be relieved within 24 hours by restarting anti-depressant therapy. b) The symptoms can be mistaken for physical illness or relapse into the treated illness, thereby promoting unnecessary long-term treatment. c) Discontinuation symptoms are more likely to occur in patients who take an SSRI with a shorter half-life than in patients taking an SSRI with a longer half-life. d) Discontinuation symptoms are more likely to occur in patients who take an SSRI with a longer half-life than in patients taking an SSRI with a shorter half-life.

The Correct Answer is D D. Intermittent noncompliance, abrupt cessation or, less commonly, tapering of a SSRI dose can result in discontinuation syndrome which manifests as disequilibrium, flulike symptoms, and sleep or sensory disturbances within 24 to 72 hours after use of the antidepressant is stopped. The most common symptoms included dizziness, nausea, lethargy, and headache. Although most discontinuation reactions are mild and short-lived, the symptoms can be mistaken for physical illness or relapse into the treated illness, thereby promoting unnecessary long-term treatment. Symptoms can be alleviated within 24 hours by reintroduction of the medication, however untreated these symptoms can last from 1 to 3 weeks. Psychiatric discontinuation symptoms (depressed mood, agitation, or irritability) in particular may be mistaken for a relapse of depressive symptoms. By comparison, rebound phenomenon (a return of some of the symptoms of depression) or a full depressive relapse usually takes 2 to 3 weeks to become evident and does not remit within 24 hours of restarting the antidepressant. The likelihood of discontinuation syndrome is associated with the duration of SSRI treatment, pharmacokinetic (half-life), and pharmacodynamic factors. The symptoms occur most often with paroxetine (Paxil) and fluvoxamine, which have a short half-life and no active metabolites, and less often with fluoxetine (Prozac) due to its long half-life. (Ditto KE. SSRI discontinuation syndrome: awareness as an approach to prevention. Postgrad Med 2003;114(2):79-84) (2).)

The four main stages of neural development are: a) meiosis, mitosis, migration, myelination b) mitosis, meiosis, differentiation, myelination c) differentiation, mitosis, migration, myelination d) proliferation, migration, differentiation, myelination

The Correct Answer is D D. Neural development involves the stages of proliferation, migration, differentiation, and myelination. The proliferation stage includes the production of cells. Migration is the second stage of development during which the cells move or migrate to its ultimate destination in the nervous system. Following migration is differentiation, in which cells develop the unique characteristics of nerve cells. During the fourth stage, myelination, the axons of some cells become surrounded (insulated) by glial cells. Meiosis is the process of cell division that produces cells with one-half the number of chromosomes as the parent cell. Mitosis is the process of cell division that produces cells with the same number of chromosomes as the parent cell.

An individual with apperceptive visual agnosia would have difficulty: a) locating an object in space b) perceiving the placement of their limbs in space c) identifying familiar faces d) identifying an object from an atypical view

The Correct Answer is D D. Visual agnosia refers to the inability to recognize familiar objects by sight. The type of visual agnosia described in this question is referred to as apperceptive agnosia and involves an inability to recognize familiar objects, especially in low-light conditions, when there are many shadows, when objects overlap, or from an atypical view.

Following a stroke, a woman complains of numbness in her left hand and she does not respond to images presented to her left visual field. She most likely suffered damage in: a) Broca's area b) Wernicke's area c) the right precentral gyrus d) the right postcentral gyrus

The Correct Answer is D The postcentral gyrus includes the somatosensory cortex, and is located in the parietal lobe. Damage to the right postcentral gyrus could result in numbness in the left hand and contralateral neglect, which includes neglect of the left visual field. Broca's ("A") and Wernicke's ("B") areas are both located in the left hemisphere of the brain and only affect language. The precentral gyrus ("C") is responsible for motor functioning.

Which of the following is consistent with the predictions of the James-Lange theory? a) I think, therefore I am. b) I''m trembling, so I must be scared. c) It's noon so I must be hungry. d) That was a great birthday surprise, so I must be happy.

The James-Lange theory is a peripheralist theory of emotion. b. CORRECT According to the James-Lange theory, the psychological experience of emotion follows bodily responses to external stimuli.

In comparison to tricyclic antidepressants, fluoxetine and other SSRIs: a) are more cardiotoxic b) are less likely to negatively affect sleep and anxiety symptoms c) are less likely to cause cognitive impairments d) are more effective for positive symptoms

The SSRIs (selective serotonin reuptake inhibitors) are associated with fewer side effects than the tricyclics. a. Incorrect SSRIs are not cardiotoxic, which is one of their advantages over tricyclics. b. Incorrect At least initially, the SSRIs may worsen sleep and anxiety symptoms. C. CORRECT Another benefit of the SSRIs is that they are not associated with cognitive impairments (the tricyclics are). d. Incorrect Positive symptoms refer to Schizophrenia, not depression.

The anterior cingulate cortex is believed to be involved in: a) color vision b) spatial memory c) emotional reactions to pain d) the physiological component of anxiety

The anterior cingulate cortex is located in the limbic cortex on the medial side of the cerebral hemispheres. C. CORRECT A number of studies have confirmed that exposure to painful stimuli activates the anterior cingulate cortex. However, this structure does not appear to be involved in the perception of pain but, instead, in the emotional reaction to painful stimuli.

The most common side effects of the benzodiazepines are: a) unsteadiness, insomnia, and agitation b) drowsiness, dizziness, and sedation c) sleeplessness, light-headedness, and headache d) confusion, agitation, and hallucinations

The benzodiazepines are anti-anxiety drugs that are used to treat anxiety, agitation, insomnia, and other disorders. a. Incorrect These side effects are less common than those listed in response b. B. CORRECT These are common side effects of the benzodiazepines. c. Incorrect These are not side effects of the benzodiazepines. d. Incorrect These side effects can occur but are rare.

The potential adverse side effects of the benzodiazepines include: a) insomnia, appetite loss, and restlessness. b) confusion, memory loss, and psychomotor impairment. c) muscle rigidity, catalepsy, and clammy skin. d) bradycardia, shortness of breath, and depression.

The benzodiazepines are anxiolytics that are used primarily as treatments for anxiety and sleep disturbances. a. Incorrect These are common side effects of the psychostimulant drugs. b. CORRECT Common side effects of the benzodiazepines include drowsiness, impaired psychomotor ability, disorientation and confusion, anterograde amnesia, and depression. c. Incorrect These are signs of narcotic-analgesic overdose. d. Incorrect These are side effects of the beta-blocker propranolol.

A patient taking a benzodiazepine develops several undesirable symptoms including nightmares, hallucinations, and rage reactions. These symptoms are most suggestive of which of the following: a) Overdose b) anticholinergic effects c) "paradoxical agitation" d) "rebound anxiety"

The benzodiazepines are associated with several undesirable side effects including rebound anxiety, physical and psychological dependence, and withdrawal symptoms. a. Incorrect Overdose is fairly rare and involves hypotension, depressed respiration, and, in severe cases, coma. b. Incorrect Anticholinergic effects include dry mouth and blurred vision. C. CORRECT Insomnia and the symptoms listed in the question are the symptoms of paradoxical agitation, which is most likely to occur in people with a history of aggressive behavior or unstable emotional behavior. d. Incorrect Rebound anxiety may occur when the drug is stopped. It is characterized by intense anxiety symptoms.

A patient who has been taking antipsychotic medication for a lengthy period of time is being slowly taken off the drug due to the development of tardive dyskinesia. Soon after the drug is completely stopped, the patient exhibits an increase in the symptoms of tardive dyskinesia. This: a) means that the patient should never have been given antipsychotic medication in the first place b) is common, and the initial increase in symptoms will likely be followed by a reduction c) suggests that the symptoms are not due to the antipsychotic medication but, instead, to unrelated brain pathology d) indicates that the patient will not benefit from withdrawal of the drug

The best way to alleviate tardive dyskinesia resulting from long-term antipsychotic drug use is to withdraw the drug gradually. B. CORRECT Many patients show a temporary increase in symptoms of tardive dyskinesia when an antipsychotic drug is withdrawn. However, over time, the symptoms gradually reduce in severity.

Which of the following is the least common side effect of the beta-blocker propranolol (Inderal): a) bradycardia b) fatigue and malaise c) decreased sexual ability d) coldness in extremities

The beta-blockers are associated with a number of undesirable side effects. a. Incorrect Bradycardia occurs in about 10% of cases. b. Incorrect Fatigue and malaise also occur in about 10% of cases and may lead to a misdiagnosis of depression. c. Incorrect Decreased sexual ability also occurs in about 10% of cases. D. CORRECT Coldness in the extremities occurs in less than 1% of cases. See M. A. Fuller & M. Sajatovic (1999), Drug information handbook for psychiatry, Hudson, OH, Lexi-Comp, Inc.

To reduce the symptoms of tardive dyskinesia, which of the following would be most effective: a) a drug that decreases dopamine levels b) a drug that increases norepinephrine levels c) a drug that decreases GABA levels d) a drug that increases serotonin levels

The cause of tardive dyskinesia (TD) seems to be complex and is not entirely understood. However, there is evidence that it is related to dopamine oversensitivity and/or GABA depletion. [See, e.g., M. F. Fagan, Treatment of tardive dyskinesia, Schizophrenia Bulletin, 1997, 23(4), 583-600.] A. CORRECT There is evidence that dopamine depleting drugs alleviate some symptoms of tardive dyskinesia. b. Incorrect In fact, the research has shown the opposite - i.e., drugs that decrease norepinephrine levels suppress the symptoms of Td. c. Incorrect Drugs that increase GABA (e.g., GABA agonists -especially benzodiazepine) have beneficial effects for many patients with Td. d. Incorrect Research on serotonin has found that SSRIs and other serotonergic drugs are not useful as a treatment for TD.

A person with damage to the cerebellum is most likely to experience: a) acute pain b) an inability to control body temperature c) a loss of coordination d) changes in the sleep/wake cycle

The cerebellum has two primary functions--balance and coordination. C. CORRECT Damage to the cerebellum can cause loss of balance and a lack of coordination. For example, ataxia is caused by damage to the cerebellum and involves a lack of coordination when performing voluntary movements.

Damage to the cerebellum is associated with all of the following except: a) apraxia b) ataxia c) dysarthria d) nystagmus

The cerebellum is responsible for the maintenance of posture and balance and the coordination of movement. A. CORRECT Apraxia is an inability to carry out purposeful movements despite normal muscle coordination and power. It is associated mostly with damage to the parietal lobe. b. Incorrect Ataxia (jerky, staggering gait and other uncoordinated movements) is caused by damage to the cerebellum. c. Incorrect Damage to the cerebellum is one of the causes of dysarthria (slurred speech). d. Incorrect Nystagmus involves jerky eye movements and an inability to fix the gaze in one direction. It can be produced by damage to the cerebellum.

Disturbances in the ability to respond to stimulation on the left side of the body is suggestive of damage to the: a) right parietal lobe b) right frontal lobe c) anterior cerebellum d) posterior pons

The condition described in this question is referred to as contralateral (hemispatial) neglect. A. CORRECT Contralateral neglect is one of the possible consequences of damage to the parietal lobe and seems to occur most often when the damage involves the right parietal lobe. A person with this disorder ignores stimulation on the affected side of the body and may even deny "ownership" of that side of the body.

A number of neuropsychologists interested in constructive memory have attempted to identify the areas of the brain that are involved in false recollections. Their research has most consistently linked false recollection to lesions in the: a) frontal lobe b) temporal lobe c) occipital lobe d) parietal lobe

The current view of memory is that it does not involve a literal recollection of past events but instead a constructive process that is susceptible to errors, distortions, and other factors that limit the accuracy of memories. A. CORRECT The research has most consistently linked false recollections to lesions in the frontal lobes, especially the prefrontal cortex. See, e.g., D. L. Schachter and S. D. Slotnik, The cognitive neuroscience of memory distortion, Neuron, 2004, 44, 149-160.

Among patients with Parkinson's Disease, depression: a) is only slightly more common than it is in the general population and seems to be a reaction to the diagnosis b) appears to be a reaction to the diagnosis and usually first appears when motor symptoms begin to interfere with daily functioning c) appears to be endogenous to the disorder and may precede motor symptoms, especially in younger patients d) appears to be endogenous to the disorder and usually first appears in the later stages as cognitive impairments increase

The debate over the relationship between Parkinson's Disease and depression has not been entirely settled. However, there seems to be more evidence that it is endogenous to the disorder rather than simply a reaction to it. a. Incorrect Depression is more common in those with Parkinson's Disease than in the general population and, as noted above, it is currently believed to be endogenous to the disorder. b. Incorrect As noted, depression is probably not just a reaction to the disorder. C. CORRECT One source of evidence for the endogenous nature of depression in Parkinson's Disease is the occurrence of a prodromal "parkinsonian personality," which is characterized by melancholia, introversion, and pessimism. The early appearance of depressive symptoms is most common in younger patients and those with a family history of the disorder. d. Incorrect As noted above, depression is an early-appearing symptom in some patients.

During the initial ___________ stage of the development of the central nervous system, new cells are produced inside the neural tube. a) differentiation b) proliferation c) synaptogenesis d) apoptosis

The development of the central nervous system involves five stages: proliferation, migration, differentiation, myelination, and synaptogenesis. a. Incorrect Differentiation is the third stage in brain development and occurs when neurons of different types develop their own unique features. b. CORRECT This question accurately describes the first stage of brain development - i.e., the generation of new cells is referred to as proliferation. c. Incorrect Synaptogenesis is the formation of synapses and is the last of the five stages. d. Incorrect Apoptosis refers to cell death. During the development of the CNS, too many cells are produced, and apoptosis is a normal process.

In Alzheimer's disease, neuron loss occurs throughout the brain. However, the greatest loss is in which of the following areas: a) medial temporal lobe b) anterior parietal lobe c) medial occipital lobe d) posterior frontal lobe

The distinctive characteristics of Alzheimer's disease are neuron loss, neurofibrillary tangles (tangles of protein), and amyloid plaques (clumps of scar tissue). A. CORRECT The medial temporal lobe includes the entorhinal cortex, amygdala, and hippocampus, which are involved in memory and are most affected by Alzheimer's disease. b. Incorrect It is the posterior (not anterior) parietal lobe that is affected by Alzheimer's. c. Incorrect The occipital lobe is not one of the brain areas substantially affected by this disorder. d. Incorrect The prefrontal area of the frontal cortex is affected more than the posterior area.

When the symptoms of Bipolar Disorder have not responded to lithium or valproate/divalproex, the next drug to try would most likely be: a) risperidone b) carbamazepine c) phenelzine d) clonazepam

The drug treatment-of-choice for Bipolar Disorder is currently lithium and/or an anti-seizure medication. a. Incorrect Resperidone is an atypical antipsychotic drug. B. CORRECT Carbamazepine (Tegretol) is an anti-seizure drug that has been found to be a useful treatment for patients with Bipolar Disorder who have not responded satisfactorily to other treatments or who cannot tolerate those treatments. c. Incorrect Phenelzine is an MAOI and is used to treat depression. d. Incorrect Clonazepam is an anti-anxiety drug.

In adults, the initial motor symptoms of Huntington's disease most often involve: a) tingling, numbness, and weakness in one limb b) rapid repetitive movements in the extremities and face c) slowness and poverty of movement d) difficulty swallowing and speaking

The early signs of Huntington's disease vary from individual to individual. For some, the initial symptoms are motor; for others, affective or cognitive. a. Incorrect These symptoms are characteristic of multiple sclerosis. B. CORRECT For many adults, the earliest motor symptoms are rapid, repetitive, involuntary movements in the extremities and face - e.g., "piano-playing" movements of the fingers and facial grimaces. c. Incorrect Slowness and poverty of movement are characteristic of Parkinson's disease. d. Incorrect Difficulty swallowing and speaking are characteristic of the middle stage of Huntington's disease.

Research on the use of antidepressants for alleviating chronic pain has generally found that: a) mixed serotonin and norepinephrine reuptake inhibitors are more effective than selective serotonin reuptake inhibitors b) selective serotonin reuptake inhibitors are more effective than mixed serotonin and norepinephrine reuptake inhibitors c) selective norepinephrine reuptake inhibitors are more effective than mixed serotonin and norepinephrine reuptake inhibitors d) selective norepinephrine reuptake inhibitors and selective serotonin reuptake inhibitors are about equally effective and each is more effective than mixed serotonin and norepinephrine reuptake inhibitors

The effectiveness of some antidepressants for treating chronic pain has been demonstrated for both depressed and non-depressed chronic pain sufferers. A. CORRECT Although the research is not entirely consistent, it appears that antidepressants that affect levels of both norepinephrine and serotonin (e.g., amitriptyline) are most effective for reducing chronic pain. See, e.g., A. C. Jung et al., The efficacy of selective serotonin reuptake inhibitors for the management of chronic pain, Journal of General Internal Medicine, 12(6), 1997, 384-389.

The endoplasmic reticulum is involved in: a) processing of incoming visual stimuli b) processing of incoming auditory stimuli c) synthesis of protein and fats d) storage of neurotransmitters

The endoplasmic reticulum is a system of membranes in the cell body of the neuron. C. CORRECT The rough portion of the endoplasmic reticulum plays a role in the synthesis of protein, while the smooth portion is involved in the synthesis of fats.

According to the physiologist MacLean (1949), evolution of the __________ coincided with the development of emotional responses: a) basal ganglia b) limbic system c) reticular formation d) medulla oblongata

The expression of emotion is mediated by the area of the brain that is sometimes referred to as the "emotional brain." a. Incorrect The tectum is involved in the control of movement. B. CORRECT The limbic system ("emotional brain") is involved in the expression of emotions, emotional memory, and recognition of emotion in others. c. Incorrect The reticular formation mediates sleep and arousal, attention, muscle tone, and vital reflexes. d. Incorrect The medulla oblongata is important for cardiovascular functioning, respiration, and muscle tone.

Neural development involves several stages. Following the production of cells (the proliferation stage), each cell moves to its ultimate location in the mature nervous system. This second stage is referred to as: a) meiosis b) mitosis c) migration d) myelination

The four main stages of neural development are: proliferation, migration, differentiation, and myelination. a. Incorrect Meiosis is the process of cell division that produces cells with one-half the number of chromosomes as the parent cell. b. Incorrect Mitosis is the process of cell division that produces cells with the same number of chromosomes as the parent cell. C. CORRECT Migration is the term used to describe the stage of development in which each cell moves (migrates) to its ultimate destination in the nervous system. Migration is followed by differentiation, in which cells develop the unique characteristics of nerve cells. d. Incorrect Myelination occurs after differentiation. During this stage, the axons of some cells become surrounded (insulated) by glial cells.

People who have had strokes that affected the left anterior cerebral hemisphere are most likely to exhibit which of the following: a) depression and worry b) apathy c) denial of feelings of depression d) flight of ideas and grandiosity

The left cerebral hemisphere appears to regulate positive emotions, and damage to the left hemisphere produces the opposite kind of emotional response. a. CORRECT Damage to the left hemisphere is associated with catastrophic reactions, which may include depression, anxiety, worry, and other negative emotional states. b. Incorrect This is likely to be a consequence of right hemisphere damage. c. Incorrect This is associated with damage to the right posterior inferior frontal lobe. d. Incorrect This is produced by damage to certain areas of the right hemisphere.

Agraphia, acalculia, finger agnosia, and right-left confusion are symptoms of which of the following? a) Creutzfeldt-Jakob disease b) Addison's disease c) Gerstmann's syndrome d) Geschwind syndrome

The four symptoms listed in this question are caused by damage to the left (dominant) angular gyrus, which is located at the point of convergence of the parietal, occipital, and temporal lobes. a. Incorrect Creutzfeldt-Jakob disease is a rare degenerative brain disease that usually leads to dementia and is characterized by memory loss, behavioral changes, loss of coordination, and visual disturbances. b. Incorrect Addison's disease is due to a lower-than-normal production of cortisol by the adrenal glands. Its symptoms include weight loss, orthostatic hypotension, weakness, fatigue, and hyperpigmentation. c. CORRECT These are the characteristic symptoms of Gerstmann's syndrome. d. Incorrect Gerschwind syndrome is a personality syndrome associated with temporal lobe epilepsy and is characterized by circumstantiality, hypergraphia, alterations in sexuality, emotional volatility, and, in some cases, hyperreligiosity.

Damage to the frontal lobes is least likely to have an adverse impact on: a) working memory b) planning ability c) self-awareness d) IQ test scores

The frontal lobes mediate higher-order cognitive skills and have also been linked to a number of behavioral and emotional functions. a. Incorrect Damage to certain areas in the frontal lobes can produce impairments in working memory. b. Incorrect Planning ability is one of the executive functions mediated by the frontal lobes. c. Incorrect Damage to the frontal lobes has been linked to impairments in self-awareness. D. CORRECT Damage to the frontal lobes ordinarily does not have a significant impact on IQ test scores. Researchers believe this is because IQ tests primarily assess convergent thinking, and the frontal lobes are more involved in the mediation of divergent thinking.

The cyclic release of gonadotrophins in females is the result of regulation by the: a) adrenal gland b) pituitary gland c) hypothalamus d) thalamus

The gonadotrophins are released by the pituitary gland as the result of signals from the hypothalamus. The gonadotrophins stimulate the gonads to release their hormones (estrogen in females, androgens in males). a. Incorrect Although the adrenal glands do release estrogen (and androgens), they are not responsible for the cyclic pattern of hormone release by the gonads. b. Incorrect The pituitary does release the gonadotrophins but as the result of signals from the hypothalamus. C. CORRECT The hypothalamus responds to existing hormone levels and triggers messages that regulate the female hormonal cycle. d. Incorrect It is the hypothalamus, not the thalamus, that is responsible for the hormonal cycle in females.

A 45-year old man is in a serious car accident and, as a result, suffers neurological damage that produces deficits in short-term memory. Most likely, the damage has affected which of the following: a) hippocampus b) amygdala c) hypothalamus d) septal area

The hippocampus appears to be involved in the recall of events, especially recent events. A. CORRECT As noted above, the hippocampus has been implicated in memory. b. Incorrect The amygdala is involved in the arousal and control of emotional and motivational behaviors.c. Incorrect Although the hypothalamus appears to be involved in a large number of activities including maintaining the body's internal homeostasis, it has not been linked to memory. d. Incorrect Lesions to the septal area have been found to produce aggressive and vicious behaviors, and this region is, therefore, believed to be involved in the mediation of aggression.

Damage to the hippocampus will have the greatest impact on: a) sensory memory b) spatial memory c) primary memory d) working memory

The hippocampus has been implicated in several aspects of memory. B. CORRECT The hippocampus is known to be involved in memory consolidation, explicit memory, and spatial memory.

Damage to the hippocampus is most likely to interfere with the ability to: a) recall information stored in remote long-term memory. b) manipulate information while it is in short-term (working) memory. c) transfer information from short- to long-term memory. d) retrieve implicit memories, even when given adequate cues.

The hippocampus is involved in memory consolidation. a. Incorrect Although the hippocampus contributes to memory consolidation, it does not appear to be involved in the storage of long-term memories. b. Incorrect The hippocampus has not been found to contribute to the ability to manipulate information in working memory. c. CORRECT The research has shown that the hippocampus is involved in explicit (conscious) memory and, more specifically, in transferring explicit memories from short- to long-term memory. d. Incorrect The hippocampus appears to be more involved in explicit than in implicit memory.

Damage to the right hippocampus is most likely to cause: a) impairments in motor skills involving the left side of the body. b) interruption of the normal sleep-wake cycle. c) deficits in spatial memory. d) deficits in procedural memory.

The hippocampus is vital to learning and memory. a. Incorrect The hippocampus is involved in declarative memory (learning of new facts, words, names, etc.) but is not involved in the acquisition or performance of motor skills. b. Incorrect The research has not linked the hippocampus to the sleep-wake cycle. c. CORRECT The right hippocampus is involved in memory for nonverbal information, including spatial and temporal memory. For example, in a study of cabdrivers, PET scans revealed that the right hippocampus was responsible for the drivers' ability to form and recall complex routes. d. Incorrect The hippocampus plays a role in declarative memory but does not appear to be responsible for the formation of procedural memories.

Which of the following is true about the retrograde amnesia caused by moderate to severe head trauma: a) the most remote memories are most vulnerable to amnesia and are the first to be recovered b) the most recent memories are most vulnerable to amnesia and are the first to be recovered c) the most remote memories are most vulnerable to amnesia but more recent memories are the first to be recovered d) the most recent memories are most vulnerable to amnesia but more remote memories are the first to be recovered

The impact of moderate to severe head trauma on memory was originally described by Theodore Ribot in 1882. D. CORRECT Head trauma is most likely to affect more recent long-term memories. When memories begin to return, however, those that are most remote are recovered first. For example, if you forget everything that happened to you during the year prior to a head trauma, once you begin to recover your "lost" memories, you will first recall things that occurred 12 months prior to the trauma, than 11 months prior to the trauma, and so on.

The left hemisphere of the cerebral cortex is dominant for speech and language functions: a) for most left-handers but few right-handers b) for nearly all left-handers and many right-handers c) for nearly all right-handers and the majority of left-handers d) for nearly all right-handers but a small minority of left-handers

The left hemisphere is dominant for language for most people, regardless of handedness. C. CORRECT For most right-handers and the majority of left-handers, the left hemisphere is dominant for language. While some left-handers have right hemisphere dominance, others show a lack of dominance of either hemisphere.

Several subcortical structures are involved in the mediation of emotion. Studies of decorticate animals, for instance, suggest that the __________ is responsible for a violent rage response to even trivial stimuli: a) hypothalamus b) medulla c) hippocampus d) RAS

The limbic system (especially the amygdala) is most associated with emotion, but other structures of the brain also play a role. A. CORRECT The hypothalamus is always a good guess since it's involved in so many different functions. The studies have shown that decorticate animals tend to respond to even minor stimuli with extreme rage as long as the hypothalamus is left intact. b. Incorrect The medulla is involved in many of the body's vital functions including breathing and heart rate. c. Incorrect The hippocampus is part of the limbic system, but it's involved more with memory than emotion. d. Incorrect The reticular activating system mediates attention, arousal, and the sleep-wake cycle.

A middle-aged man with brain damage cannot pantomime motor movements when requested to do so (e.g., cannot pretend to open a door with a key) but usually has less trouble performing the same movements when using actual objects. The damage causing this impairment is most likely located in the man's: a) left posterior parietal lobe b) right anterior parietal lobe c) left posterior temporal lobe d) right anterior temporal lobe

The man's symptoms are characteristic of apraxia. A. CORRECT The condition described in this question is a type of ideomotor (or limb) apraxia. It is caused by damage to the left posterior parietal lobe and, consequently, is also known as left parietal apraxia.

The area of the brain referred to as the "midbrain" contains all of the following structures except: a) inferior colliculus b) superior colliculus c) substantia nigra d) mammillary bodies

The midbrain is also known as the mesencephalon and is divided into two regions: the tectum and the tegmentum. a. Incorrect The inferior colliculus is part of the tectum and is involved in audition. b. Incorrect The superior colliculus is also part of the tectum and mediates vision. c. Incorrect The substantia nigra is located in the tegmentum and is part of the brain's sensorimotor system. D. CORRECT The mammillary bodies are part of the hypothalamus and are located in the diencephalon which, along with the telencephalon, make up the forebrain.

The neurotransmitter GABA is a(n): a) amino acid that plays an inhibitory role regardless of its location in the brain. b) amino acid that plays an excitatory role regardless of its location in the brain. c) amino acid that plays an inhibitory or excitatory role, depending on its location in the brain. d) neuropeptide that plays an inhibitory or excitatory role, depending on its location in the brain.

The neurotransmitters can be classified in terms of their chemical make-up and their effects on post-synaptic neurons. a. CORRECT Many neurotransmitters serve both excitatory and inhibitory functions. GABA, an amino acid, is one of the neurotransmitters that serve only one of these functions - i.e., it is an inhibitory neurotransmitter. d. Incorrect GABA is an amino acid, not a neuropeptide. (The neuropeptide neurotransmitters include substance P and the endorphins.)

Damage to the orbitofrontal cortex is most likely to: a) alter emotional behaviors b) impair complex motor movements c) alter dietary habits d) impair peripheral vision

The orbitofrontal cortex is the area of the prefrontal cortex that is located just above the orbits (the bones that form the eye sockets). A. CORRECT Recent research has confirmed that the orbitofrontal cortex plays an important role in emotional behavior. Damage to this area can lead to reduced inhibition and self-concern, indifference to the consequences of one's actions, and reduced emotional responsivity to noxious stimuli. b. Incorrect The dorsolateral prefrontal association cortex is involved in the mediation of complex motor movements. c. Incorrect This area has not been linked to dietary habits. d. Incorrect The orbitofrontal cortex has not been linked to peripheral vision.

Apraxia, or the inability to perform a motor act (e.g., to draw a simple picture or to wave "hello"), is most suggestive of damage to the: a) temporal lobe b) parietal lobe c) arcuate fasciculus d) corpus callosum

The parietal lobe is adjacent to the motor area and contains the somatic sensory area, which provides information about the movable parts of the body. Consequently, damage to the parietal lobe can produce certain types of sensory-motor deficits. B. CORRECT Damage to the parietal lobe is associated with a number of abnormalities in movement and sensation including various types of apraxia.

As the result of a head injury sustained in a car accident, a young woman exhibits both retrograde and anterograde amnesia. Once her memories begin to return, which of the following is likely to be recovered first? a) memory for events that occurred several weeks before the accident b) memory for events that occurred during the hour before the accident c) memory for events related to the accident d) memory for events that occurred after the accident

The pattern of memory impairment and recovery following head injury was originally described by Ribot in 1882. a. CORRECT Recent memories (including memory for the traumatic event) are most susceptible to disruption as the result of trauma. Remote memories are less affected and, when they are lost, are ordinarily the first to be recovered.

___________ is a communication disorder that is characterized by difficulties in regulating the rate, rhythm, pitch, and loudness of speech. a) Dysarthria b) Paraphasia c) Dysprosody d) Adynamia

The pattern of stress and intonation in speech is referred to as prosody. a. Incorrect Dysarthria is a motor speech disorder that produces problems related to articulation. b. Incorrect Paraphasia is a type of aphasia that involves the use of unintended syllables, words, or phrases (e.g., unintentionally substituting one word for another). c. CORRECT Dysprosody is a disruption in speech melody and rhythm and is characteristic of Broca's aphasia and other nonfluent aphasias. Although dysprosody has traditionally been linked to right hemisphere damage, its presence in Broca's aphasia suggests that prosody is also governed, to some degree, by the left hemisphere. d. Incorrect Adynamia is a type of aphasia involving difficulty in initiating speech.

In terms of drug use, a person is exhibiting "habituation" when he/she: a) has developed physical dependence on the drug b) requires higher and higher doses of the drug to achieve the same effects c) has a desire to continue using the drug with little or no desire to increase the amount of the drug d) experiences reduced symptoms following cessation of the drug due to multiple withdrawal experiences

The precise meaning of the term "habituation" depends on the context in which it is used. C. CORRECT From the perspective of pharmacology, habituation occurs when repeated use of a drug results in a desire for continued use with little or no desire to increase the drug and no physical dependence.

Damage to the prefrontal cortex is most likely to cause: a) impaired memory and attention b) "psychic blindness" c) alterations in the sleep/wake cycle d) impaired motor coordination

The prefrontal cortex has been linked to a variety of cognitive functions, including attention and certain aspects of memory. A. CORRECT A person with damage to the prefrontal cortex may exhibit a reduced attention span, distractibility, impaired working memory, and impaired prospective memory. b. Incorrect "Psychic blindness" is associated with Kluver-Bucy syndrome, which is caused by damage to certain areas of the temporal lobes.

Damage to the dorsolateral area of the prefrontal cortex is likely to have the least impact on performance on: a) standardized intelligence tests b) measures of executive functions c) recall memory tasks d) measures of creativity

The prefrontal cortex is involved in executive functioning (e.g., planning, organization, sequencing) and personality. The results of damage to this area depend on its exact location. A. CORRECT In general, performance on standardized IQ tests is less affected than performance on specific aspects of executive functioning and memory. See J. L. Bradshaw and J. B. Mattingley, Clinical Neuropsychology: Behavioral and Brain Science, San Diego, Academic Press, 1995. b. Incorrect Damage to the dorsolateral area of the prefrontal cortex does cause impaired executive functioning. c. Incorrect Damage to this area also disrupts performance on recall tasks. d. Incorrect Creativity is also adversely affected by damage to this area of the brain.

Wernicke's encephalopathy is due to: a) a deficiency of thiamine b) a deficiency of Vitamin C c) acute head trauma d) anoxia

The primary symptoms of Wernicke's encephalopathy are acute mental confusion, ataxia, and opthalmoplegia (abnormal eye movements). A. CORRECT Wernicke's encephalopathy is caused by a deficiency of thiamine, often as the result of chronic alcoholism.

Research conducted in the 1930s found that ablation of the anterior temporal lobes in male rhesus monkeys produced a variety of symptoms including hypersexuality, placidity, oral tendencies, and psychic blindness. This condition is known as: a) Kluver-Bucy syndrome b) Gerstmann's syndrome c) Gerschwind syndrome d) Wernicke-Korsakoff syndrome

The research described in this question was conducted by Kluver and Bucy. A. CORRECT This constellation of symptoms caused by lesions in the anterior temporal lobes is referred to as Kluver-Bucy syndrome.

_________________ has been used as a pharmacologic model for schizophrenia because drugs that alleviate the former also reduce the symptoms of the latter. a) Serotonin syndrome b) Neuroleptic malignant syndrome c) Alcohol delirium tremens d) Amphetamine psychosis

The research has identified several similarities between the effects of amphetamines and the symptoms of schizophrenia. d. CORRECT Research on amphetamine psychosis has provided support for the dopamine hypothesis for schizophrenia: First, amphetamines exert their effects by altering dopamine activity. Second, a high dose of amphetamine produces symptoms similar to those associated with schizophrenia, and amphetamines exacerbate the symptoms of schizophrenia. Third, drugs that reduce dopamine levels reduce the symptoms of both amphetamine psychosis and schizophrenia.

Damage to the reticular activating system is most likely to cause which of the following? a) deficits in explicit memory b) disruptions in the sleep-wake cycle c) impaired balance and coordination d) Kluver-Bucy syndrome

The reticular activating system (RAS) is a complex, net-like system of nuclei and fibers that extends from the spinal cord into the brain. a. Incorrect Damage to the hippocampus is associated with deficits in explicit memory. b. CORRECT Damage to the RAS disrupts the sleep-wake cycle and can produce a permanent, coma-like state. c. Incorrect Damage to the cerebellum would cause these symptoms. d. Incorrect Kluver-Bucy syndrome is characterized by reduced fear, increased docility, compulsive oral behaviors, altered dietary habits, an inability to recognize visual stimuli, and hypersexuality. It is caused by lesions in amygdala, hippocampus, and certain areas of the temporal lobe.

Vinny Vibrissa is unable to recognize familiar objects by touch. This is most likely due to damage to his: a) prefrontal cortex b) mesencephalon c) temporal lobe d) parietal lobe

The sensation of touch is mediated by the somatosensory cortex. D. CORRECT This impairment is due to damage to the somatosensory cortex, which is located in the parietal lobe.

Research comparing the impact of unilateral brain damage on emotion has shown that damage to the right hemisphere is most often associated with three behavioral patterns - indifference, verbal disinhibition, and: a) denial of paralysis or other symptoms resulting from the brain damage b) depressed mood and irritability c) pathological laughing and crying d) transitory visual and/or auditory hallucinations

The specific roles of the left and right hemispheres in emotion are still not entirely understood. However, there is some consistent evidence that damage to the right or left hemisphere produces a predictable pattern of symptoms. A. CORRECT In terms of emotions, damage to the right (nondominant) hemisphere has been linked to indifference and apathy, denial of one's disabilities, and verbal disinhibition (which often includes a tendency to joke in a sarcastic or childish manner, especially about one's disabilities). See, e.g., M. R. Rosenzweig et al., Biological Psychology: An Introduction to Behavioral, Cognitive, and Clinical Neuroscience, Sunderland, MA: Sinauer Associates, 1999. b. Incorrect Depressed mood, mood fluctuations, anxiety, and irritability are symptoms of left frontal lobe damage. c. Incorrect Pathological laughing and crying are usually caused by bilateral hemispheric lesions. d. Incorrect Visual and/or auditory hallucinations are not symptoms that occur in conjunction with right hemisphere damage that produces indifference and verbal disinhibition.

Problems involving sensation in the inner forearm and the ring and little fingers are most likely due to spinal injury at the ______ level: a) L1 b) S1 to S4 c) T2 to T12 d) C8 to T1

The spinal cord is divided into four regions - from top to bottom, cervical, thoracic, lumbar, and sacral. a. Incorrect The lumbar region of the spine contains nerves that connect to the thigh, hip, and leg. b. Incorrect The sacral region contains nerves that innervate the foot and the posterior and lateral aspects of the leg. c. Incorrect T2 through T12 innervate the trunk. D. CORRECT T1 is the first segment of the thoracic spinal chord and contains nerves that innervate the forearm and C8 innervates the ring and little fingers.

The __________ is responsible for the body's "fight or flight" reaction: a) limbic system b) thalamus c) parasympathetic nervous system d) sympathetic nervous system

The term "fight or flight" refers to the response of the glands and smooth muscles to dangerous or emergency situations. D. CORRECT It is the sympathetic branch of the autonomic nervous system that is responsible for the body's fight or flight reaction (e.g., increased heart rate, cessation of digestive processes).

Oversecretion of _________ by the hypothalamic-pituitary-adrenal axis has been linked to depression. a) glutamate b) cortisol c) adrenalin d) melatonin

The structures of hypothalamic-pituitary-adrenal (HPA) axis interact and regulate a number of important functions including the body's reaction to stress and the activities of the immune system. Recently, the HPA axis has also been linked to mood and anxiety disorders. a. Incorrect Although there is some evidence that glutamate plays a role in depression, it is not secreted by the HPA axis. b. CORRECT Secretion of cortisol, a stress hormone, by the adrenal gland is influenced by the activity of the hypothalamus and pituitary gland. Although cortisol plays an important role in helping the body respond to stress, recent research has found that chronic excessive cortisol levels are associated with disturbances in sleep, decreased appetite, a reduced sex drive, and other symptoms of depression. c. Incorrect Lower-than-normal levels of noradrenaline - but not adrenaline - in noradrenergic centers in the brain (locus coeruleus and caudal raphe nucleus) have been linked to depression. d. Incorrect Abnormalities in melatonin have been linked to seasonal affective disorder but not to depression more generally. In addition, melatonin is secreted by the pineal gland.

Research on "motor imagination" indicates that which area of the brain is most likely to be active when a person is imagining that he/she is engaging in a motor activity: a) posterior occipital lobe b) mammillary bodies c) supplementary motor area d) ascending reticular activating system

The studies on motor imagination have found that the same areas of the brain that are active during the execution of a motor activity are active when a person is imagining that he/she is engaging in that activity. See, e.g., M. Jeannerod, Mental imagery in the motor context, Neuropsychologia, 1995, 33(11), 1419-1432. c. CORRECT The specific areas that are activated when a person imagines he/she is engaging in a motor activity depend on the specific nature of the activity. However, the areas most likely to be involved include the supplementary motor area, cerebellum, basal ganglia, and parietal lobe (especially the somatosensory cortex).

The sleep-wake cycle and other circadian rhythms are controlled by which of the following structures of the brain? a) extrapyramidal system b) reticular activating system c) lateral geniculate nucleus d) suprachiasmatic nucleus

The suprachiasmatic nucleus (SCN) is located in the hypothalamus just above the optic chiasm. a. Incorrect The extrapyramidal system is a network of neurons that is involved in movement. b. Incorrect The reticular activating system is a network of neurons that extends from the spinal cord into the brain and is involved in motivation and arousal. c. Incorrect The lateral geniculate nucleus is part of the thalamus and is involved in the processing of information related to vision. d. CORRECT The SCN is the "primary circadian pacemaker" in mammals and is responsible for circadian rhythms related to sleep, temperature, blood pressure, the production of hormones, and other functions.

A 50-year old man with a frontal lobe tumor has discrepant motor and verbal behavior, occasional episodes of aggression, and problems in initiating behavioral activities. Other symptoms likely to be exhibited by this man include: a) impulsivity and inappropriate euphoria b) impaired sustained attention and motor perseveration c) sparse verbal output and lower extremity weakness d) occasional hallucinations and a "dazed" feeling

The symptoms caused by a tumor in the frontal lobe depends on its location. The symptoms given in the question are characteristic of frontal convexity syndrome, which is characterized by apathy. a. Incorrect These symptoms are consistent with orbitofrontal syndrome, which is marked by disinhibited behavior. B. CORRECT These symptoms are consistent with frontal convexity syndrome. c. Incorrect These symptoms suggest medial frontal (akinetic) syndrome. d. Incorrect These symptoms are suggestive of a temporal lobe tumor.

A person with a temporal lobe lesion will most likely exhibit which of the following? a) memory impairment, depression or anxiety, and hallucinations b) poor judgment, impulsivity, and inappropriate jocularity c) homonymous hemianopia, prosopagnosia, and irritability and suspiciousness d) dysgraphia, depression and apathy, and denial of his/her problems

The symptoms listed in each answer to this question are associated with lesions in a different lobe of the cerebral cortex. a. CORRECT The temporal lobe mediates auditory perception and memory, and a lesion in this lobe affects memory, may produce hallucinations (especially auditory hallucinations), and often affects mood. b. Incorrect These symptoms are associated with lesions in certain areas of the frontal lobe. c. Incorrect These symptoms are more likely to be caused by an occipital lobe lesion. d. Incorrect A lesion in the parietal lobe may produce these symptoms.

A patient taking clozapine exhibits several symptoms including tachycardia, muscle rigidity, hyperthermia, altered consciousness, and autonomic dysfunction. The best action in this case would be to: a) gradually reduce the drug dose until the symptoms are alleviated b) immediately stop the drug and administer fluids and electrolytes c) switch the patient to a traditional antipsychotic drug d) check to see what other drugs the patient is taking since clozapine does not produce these symptoms

The symptoms listed in the question are characteristic of neuroleptic malignant syndrome (NMS), which is a rare disorder that can occur with any class of neuroleptic agent. B. CORRECT NMS can be fatal unless it is recognized early and the medication is immediately stopped.

Drugs that block the activity of __________ produce dry mouth, blurred vision, postural hypotension, tachycardia, and sedation. a) 5-HT b) ACh c) glycine d) glutamate

The symptoms listed in the question are collectively referred to as "anticholinergic effects." b. CORRECT ACh (acetylcholine) is one of the neurotransmitters classified as "cholinergic." Drugs that block acetylcholine activity produce anticholinergic effects. c. Incorrect Glycine is found in the spinal cord and exerts an inhibitory effect. d. Incorrect Glutamate is associated with learning and memory and the effects of alcohol.

Wernicke's aphasia is characterized by: a) fluent output, impaired repetition, intact comprehension b) nonfluent output, disturbed repetition, abnormal comprehension c) nonfluent output, poor repetition, intact comprehension d) fluent output, impaired repetition, poor comprehension

The term "aphasia" refers to a loss of language functioning as the result of brain impairment. a. Incorrect This describes conduction aphasia. b. Incorrect This describes global aphasia. c. Incorrect These symptoms are characteristic of Broca's aphasia. D. CORRECT This combination of symptoms is indicative of Wernicke's aphasia.

A person with associative visual agnosia: a) can name a familiar object she sees but does not know how to use it b) cannot name a familiar object she sees but can name it when it is placed in her hand c) cannot name a familiar object because she doesn't recognize it d) recognizes a familiar object but is unable to name it

The term visual agnosia is generally used to describe the inability to recognize familiar objects by sight. a. Incorrect This is not characteristic of visual agnosia. b. incorrect This describes the condition known as aperceptive visual agnosia. c. Incorrect This doesn't describe associative visual agnosia. D. CORRECT Aperceptive agnosia and associative visual agnosia are the two most common types of visual agnosia. This response describes associative visual agnosia, which occurs when visual and language areas become disconnected.

According to Schachter and Singer (1962), a person's subjective emotional response to an event: a) is determined entirely by environmental cues. b) is determined entirely by the person's physiological reaction to the event. c) depends primarily on the simultaneous activation of certain peripheral and brain mechanisms. d) depends on a combination of the person's physical reaction to and cognitive interpretation of the event.

The theories of emotion differ with regard to the degree to which they emphasize the role of peripheral and central factors. a. Incorrect Schachter and Singer's theory recognizes the role of environmental factors in the experience of emotion, but does not attribute emotion entirely to environmental cues. b. Incorrect This sounds like the James-Lange theory of emotion, which stresses the importance of peripheral factors. c. Incorrect This sounds like the Cannon-Bard theory of emotions. d. CORRECT According to Schachter and Singer's (1962) two-factor theory, subjective emotional experience is the consequence of a combination of physical arousal and cognitive interpretation of that arousal and the context in which it occurs.

Huntington's disease, Marfan's syndrome, and Von Willebrand's disease are due to: a) an autosomal dominant gene b) an autosomal recessive gene c) an X-linked dominant gene d) an X-linked recessive gene

The three disorders listed in the question are all genetic disorders. A. CORRECT All three disorders are caused by an autosomal dominant gene, which means that they occur in the presence of only one gene on a chromosome that is not a sex (X or Y) chromosome

When taken as a treatment for depression, the average onset of the therapeutic effects of the SSRI's (selective serotonin reuptake inhibitors) is ________ after beginning to take the drug: a) three to five days. b) two to four weeks. c) eight to twelve weeks. d) two to five months.

The time delay for the onset of the therapeutic effects of the SSRIs varies somewhat for different SSRIs and for different individuals. However, in most cases, the effects are evident within two to four weeks. B. CORRECT: Individuals usually experience a substantial improvement in depressive symptoms within two to four weeks after beginning to take an SSRI (although, for some patients, the full therapeutic effects may not be evident for up to six to eight weeks).

Drowsiness (sedation) is most likely to be an initial side effect of which of the following antidepressants? a) fluoxetine (Prozac) b) paroxetine (Paxil) c) sertraline (Zoloft) d) doxepin (Sinequan)

The tricyclic antidepressants are, in general, more sedating than the SSRIs. d. CORRECT Doxepin is a tricyclic antidepressant, and drowsiness is one of the most common side effects of this drug (although it tends to decrease over time). In contrast, the drugs listed in answers a, b, and c are SSRIs and are less likely than doxepin to produce sedation.

Subjects in a research study are asked to tap with a finger to keep time with a metronome. After several taps, the metronome is turned off and subjects are asked to continue with the same beat. Subjects with damage to which of the following areas of the brain will have the most difficulty with this task: a) cerebellum b) basal ganglia c) cortex d) hippocampus

This is a difficult question because more than one of the structures listed are involved in motor function. A. CORRECT This question describes a study conducted by Keele and Ivry who propose that an underlying impairment in cerebellar damage is a loss of timing. In their study, patients with cerebellar damage had more trouble on this task than patients with damage to the cortex or basal ganglia, leading these authors to conclude that the cerebellum is involved in the timing of movements. (S. W. Keele and R. Ivry, Does the cerebellum provide a common computation for diverse tasks? A timing hypothesis. In A. Diamond, ed., The Development and Neural Bases of Higher Cognitive Functions, Annals of the New YorkAcademy of Sciences, 1991, 608, 197-211.)

A person with agnosia cannot identify a familiar object visually but can identify it kinesthetically when it is placed in his hand. This condition is due to: a) lesions in the visual association cortex b) lesions in the somatosensory cortex c) disconnection of the visual association cortex from language areas d) disconnection of the somatosensory cortex from language areas

The type of visual agnosia described in this question is referred to as apperceptive agnosia. A. CORRECT Apperceptive agnosia involves gross perceptual distortions that make it difficult to identify an object visually. It is believed to be caused by lesions in the visual association cortex, which is located in the occipital lobe. b. Incorrect A lesion in the somatosensory cortex would make it difficult to identify an object kinesthetically. c. Incorrect This would cause associative visual agnosia, which is characterized by an inability to name an object while being able to demonstrate its use or matching it to similar objects. It is often caused by a stroke involving the posterior cerebral artery. (See R. L. Strub and F. W. Black, Neurobehavioral Disorders: A Clinical Approach, 1988, Philadelphia, F. A. Davis Co.)

Which of the following is incorrectly matched? a) migraine headache: unilateral throbbing pain, nausea, irritability, and pain that worsens with routine physical activity b) cluster headache: dull throbbing pain that begins with an aura and is exacerbated by bright light c) tension headache: bilateral steady dull pain and a feeling of tightness in the head that may be accompanied by sleep disturbances and impaired concentration d) sinus headache: throbbing pain that is localized around the eyes and is made worse by bending over

The various types of headache listed in the answers involve different symptoms. Note that this question is asking which type of headache is described incorrectly. a. Incorrect This accurately describes a migraine headache. b. CORRECT A cluster headache is characterized by sharp, penetrating, or burning pain behind the eye or in the temple, a stuffy or runny nose, and a red, flushed face. c. Incorrect This accurately describes a tension headache. d. Incorrect This accurately describes a sinus headache.

Soon after starting to take an antidepressant, a young woman develops several undesirable side effects including dry mouth, urinary retention, constipation, and blurred vision. Which of the following would be most useful for alleviating these symptoms? a) caffeine b) antihistamine c) dopamine agonist d) cholinergic agonist

The woman's symptoms are anticholinergic side effects that are caused by the antidepressant drug's interference with acetylcholine activity. d. CORRECT Cholinergic agonists (e.g., bethanechol) increase acetylcholine activity by enhancing the activity of endogenous acetylcholine or by binding to and activating acetylcholine receptors. A cholinergic agonist would help alleviate the woman's anticholinergic side effects.

Soon after starting to take an antidepressant, a young woman develops several troublesome symptoms including dry mouth, urinary retention, constipation, and blurred vision. Which of the following would be most useful for alleviating these side effects: a) caffeine b) an antihistamine c) a dopamine agonist d) a cholinergic agonist

The woman's symptoms are collectively referred to as "anticholinergic side effects" and are caused by interference with acetylcholine activity. D. CORRECT A drug that increases activity at acetylcholine receptors would be appropriate in this situation. Cholinergic agonists (e.g., bethanechol) work by binding to and stimulating acetylcholine receptors.

Expressive aphasia is to Broca's area as ____________ aphasia is to Wernicke's area: a) global b) conduction c) impressive d) motor

There are many different kinds of aphasia, and, unfortunately, several go by more than one name. a. Incorrect Global aphasia involves deficits in comprehension, repetition, and output and is due to damage to several regions of the brain including the cortex and basal ganglia. b. Incorrect Conduction aphasia is caused by damage to nerves that connect Broca's and Wernicke's areas. C. CORRECT Wernicke's aphasia has several names including sensory, receptive, and impressive aphasia. d. Incorrect Motor aphasia is another name for Broca's aphasia.

A __________ seizure affects movement and sensation, usually on one side of the body, without a loss of consciousness: a) petit mal b) atonic c) clonic d) simple partial

There are many different types of seizures. Each involves a different etiology and combination of symptoms. a. Incorrect Petit mal (absence) seizures involve a loss of consciousness without prominent motor symptoms. b. Incorrect Atonic seizures involve a sudden loss of muscle tone that causes the individual to fall down. c. Incorrect Clonic seizures are characterized by jerky movements. D. CORRECT A simple partial seizure begins on one side of the brain and, at least initially, affects only one side of the body. It does not involve a loss of consciousness.

The treatment of acute myelogenous leukemia (AML) ordinarily begins with: a) splenectomy b) cranial irradiation c) induction chemotherapy d) allogenic bone marrow transplant

There are several types of leukemia, and each is associated with a different treatment regimen. a. Incorrect Splenectomy is sometimes used as a treatment for hairy cell leukemia. b. Incorrect Cranial irradiation is used for some types of leukemia but is not ordinarily the initial intervention. The purpose of cranial irradiation is to keep the cancer from spreading to the nervous system. C. CORRECT Induction chemotherapy is the initial treatment for AML. Its purpose is to bring about remission. d. Incorrect Allogenic bone marrow transplant is used in some cases of leukemia but not as the initial intervention.

There are two major types of color-blindness: Monochromats are totally color-blind, while dichromats are insensitive to one of two color pairs (red/green or blue/yellow). The existence of dichromats supports which of the following theories of color vision? a) trichromat theory b) opponent-process theory c) place theory d) dual-process theory

There are two major theories of color vision: trichromatic and opponent-process. a. Incorrect Trichromatic theory postulates that the human eye has three types of color receptors (red, green, and blue), and that the perception of different colors is due to a combination of the responses of these three receptors. b. CORRECT According to opponent-process theory, there are three sets of cones: one that is sensitive to white and black, another to red and green, and a third to yellow and blue. Color-blindness results from the absence of one or more of these sets of cones and, therefore, explains why dichromats cannot distinguish between either red and green or blue and yellow. c. Incorrect Place theory is a theory of audition, not vision. d. Incorrect According to dual process theory, a response made by an individual to a stimulus involves two stages - a decision as to whether or not to respond and a decision as to which alternative response to make. It is not a theory of color vision.

Loss of fine motor movements and motor strength, poor response inhibition, altered sexual behavior, and impaired olfactory discrimination are suggestive of damage to the: a) parietal lobe b) frontal lobe c) temporal lobe d) occipital lobe

These symptoms are not necessarily the ones you may have associated with the frontal lobe, but disturbances in certain types of motor functions, difficulties in using feedback from the environment to regulate behavior, and impaired sexual functioning are possible sequelae of frontal lobe damage. a. Incorrect Parietal lobe damage is associated with apraxia, tactile agnosia, and contralateral neglect. B. CORRECT Other symptoms of frontal lobe damage include poor temporal memory, loss of divergent thinking, and impaired social behavior. c. Incorrect Impaired auditory sensation and perception, disturbances in personality and affect, and changes in sexual activity are possible consequences of temporal lobe lesions. d. Incorrect Damage to the occipital lobe produces disturbances in vision such as visual agnosia and blindness.

Hemiplegia involving the right side of the body suggests damage to the: a) motor cortex in the left hemisphere b) motor cortex in the right hemisphere c) somatosensory cortex in the left hemisphere d) somatosensory cortex in the right hemisphere

To answer this question, you have to know that hemiplegia refers to partial or complete paralysis on one side of the body and that the right side of the body is controlled by the left side of the brain. A. CORRECT Damage to the motor cortex could result in paralysis. Since the paralysis is on the right side of the body, this implies that the damage is to the motor cortex in the left hemisphere.

A handwriting disability that is characterized by difficulties in learning to write, and writing that is accurate but painstakingly slow or of poor quality is referred to as: a) motor agraphia b) dysgraphia c) pure alexia d) dyslexia

This is a difficult question because the terms "agraphia" and "dysgraphia" are often used interchangeably in the literature. a. Incorrect Motor agraphia is a specific type of agraphia (inability to write) that is due to brain damage that affects motor (expressive) functioning. B. CORRECT Some experts use the term dysgraphia to describe an inability to write that is not due to motor impairment (see., e.g., Renee M. Newman, Dysgraphia: Causes and Treatment, http://www.dyscalculia.org/Edu563.html). c. Incorrect Pure alexia refers to "word blindness." d. Incorrect Dyslexia is also defined differently by different experts. According to one definition, it is "a neurologically-based, often familial, disorder which interferes with the acquisition and processing of language" (www.ldteens.org/Dyslexia-Defin.html). Although dyslexia can cause problems related to handwriting, it is not limited to this type of impairment. Consequently, this is not the best answer of those given.

When used as a treatment for ADHD, the most common initial side effects of methylphenidate (Ritalin) are: a) skin rash and itching. b) paresthesias. c) joint pain and muscle cramps. d) insomnia and reduced appetite.

This is a relatively easy question as long as you're familiar with the common side effects of methylphenidate and other CNS stimulants. a. Incorrect Skin rash and itching may occur but are less common that insomnia and appetite loss. b. Incorrect Paresthesias are not a side effect associated with methylphenidate. c. Incorrect Joint pain and muscle cramps may also occur, but they are much less common than insomnia and appetite loss. d. CORRECT For most individuals taking methylphenidate as a treatment for ADHD, the drug's initial side effect is difficulty sleeping. Other common side effects include decreased appetite, nervousness, irritability, and nausea and abdominal pain.

You've noticed that when you go to the dentist and are very anxious, the pain of the drill is much worse than when you're able to remain calm and relaxed. This phenomenon is best explained by which of the following: a) minimum threshold theory b) gate control theory c) signal detection theory d) pattern theory

This is one of those questions that you wouldn't want to spend a long time trying to figure out if you didn't know the answer. a. Incorrect This is a "made-up" term. B. CORRECT Gate control theory suggests that the spinal cord contains a mechanism ("gate") that can block the transmission of pain to the brain. Apparently, negative emotional states serve to keep the gate open. c. Incorrect Signal detection theory proposes that there are no absolute thresholds for sensations and, instead, that the detection of stimulation depends on several factors including the physical energy of the stimulus and the costs and benefits associated with detecting or not detecting it. d. Incorrect Pattern theory predicts that certain combinations of stimuli evoke coded patterns of neural impulses that lead to the experience of pain.

According to ________________, physical stimulus changes are logarithmically related to their psychological sensations. a) Weber's law b) Fechner's law c) Lashley's law of equipotentiality d) Thorndike's law of effect

This question is asking about one of the psychophysical laws that attempt to explain the relationship between sensation and perception. a. Incorrect According to Weber's law, the more intense the stimulus, the greater the increase in stimulus intensity required for the increase to be perceived. b. CORRECT Fechner's law proposes that sensation is a logarithmic function of stimulus intensity - i.e., that a person's experience of stimulus intensity increases arithmetically as the stimulus intensity increases geometrically. c. Incorrect Lashley's law of equipotentiality is not a psychophysical law but proposes that, when an area of the cortex is damaged, other areas can take over the functions of the damaged area. d. Incorrect Thorndike's law of effect is not a psychophysical law but, instead, a precursor to Skinner's operant conditioning. It predicts that behaviors that are followed by "satisfying consequences" are likely to occur again.

A primary concern about the long-term use of chlorpromazine is that it may result in the development of which of the following? a) acute hypertensive crisis b) profound anterograde amnesia c) tardive dyskinesia d) renal toxicity

This question requires you to know that chlorpromazine is an antipsychotic drug and that antipsychotics are associated with a number of adverse side effects. a. Incorrect This is a side effect of the MAOIs. b. Incorrect This is a potential side effect of triazolam and other benzodiazepines. c. CORRECT The long-term use of an antipsychotic drug - especially chlorpromazine or other conventional antipsychotic - can result in tardive dyskinesia, a neurological movement disorder involving involuntary movements of the lips, tongue, face, trunk, and extremities (e.g., facial grimacing, tongue protrusion, guitar and piano-playing movements). d. Incorrect This is associated with the use of lithium.

As a result of brain injury, Thom Tenacity often has difficulty stopping a behavior once he starts it. For example, when Thom goes to the drawer to get a fork before dinner, he often ends up taking out all of the silverware; and when he intends to take only one jacket out of his closet, he ends up removing all of his jackets. Most likely, Thom's problem is due to damage to which of the following areas of the brain: a) frontal lobe b) parietal lobe c) temporal lobe d) occipital lobe

Thom's behavior is referred to as perseveration. A. CORRECT Perseveration (the inability to stop a behavior once it begins) has been linked to lesions in certain areas of the frontal lobes.

The venom of the black widow spider is believed to cause violent and uncontrollable muscle contractions by affecting the activity of which of the following neurotransmitters: a) acetylcholine b) norepinephrine c) enkephalin d) serotonin

To answer this question, you must know which of the neurotransmitters listed is involved in the control of the skeletal muscles. Alternatively, you might be able to choose the right answer through a process of elimination. A. CORRECT Acetylcholine is found in the peripheral nervous system, the spinal cord, and certain regions of the brain. In the peripheral nervous system, it activates both muscles and glands. The venom of the black widow spider is believed to work by causing a continuous flow of acetylcholine into the neuromuscular junction, resulting in uncontrollable muscle contractions. b. Incorrect Norepinephrine, a catecholamine, is most associated with personality, mood, and drive states. c. Incorrect Enkephalin, one of the endogenous morphine substances found in the brain, is believed to be involved in the mediation of pain. d. Incorrect Serotonin is believed to be involved in the suppression of the ARAS, the regulation of temperature, hunger, and aggression, and the affective disorders and schizophrenia.

The beneficial effects of a phenothiazine would most likely be apparent first for which of the following symptoms: a) hallucinations b) social withdrawal c) anergia d) thought disorder

To answer this question, you need to know that phenothiazines are "typical" antipsychotic drugs, which are better at alleviating the positive (versus the negative) symptoms of Schizophrenia. A. CORRECT The positive symptoms include delusions, hallucinations, and agitation. b. Incorrect Social withdrawal is a negative symptom. c. Incorrect Anergia is a negative symptom and refers to reduced activity. d. Incorrect Since the term thought disorder includes both positive and negative symptoms, this is not the best response. (Poverty of speech content, for example, is a negative symptom.)

As a consequence of head trauma caused by a serious car accident, a middle-aged woman experiences a loss of sensation for touch, temperature, and pain in her left hand. Most likely the damage responsible for this deficit is in her right ________ lobe: a) frontal b) temporal c) occipital d) parietal

To answer this question, you need to know what part of the cerebral cortex mediates cutaneous sensation. D. CORRECT Cutaneous sensation is mediated by the somatosensory cortex which is located in the parietal lobe.

Tom J. was involved in a car accident in which he received extensive head trauma. Since the accident, Tom's movements are jerky and uncoordinated, and he is unable to walk across the room unaided. Tom most likely suffered damage to the: a) medulla b) cerebellum c) parietal lobes d) temporal lobes

To answer this question, you need to know which of the brain areas listed is involved in the control of motor movement and balance. a. Incorrect The medulla is involved in the control of autonomic functioning (e.g., breathing, heartbeat, blood pressure). B. CORRECT The cerebellum is located at the rear of the brain stem and is involved in the coordination of voluntary movements of the skeletal muscles and the regulation of balance. c. Incorrect Damage to the parietal lobes is associated with aphasia, visual deficits, and somatosensory disturbances. d. Incorrect The temporal lobes are involved in auditory and visual processing, and damage is associated with visual deficits and memory loss.

The primary motor cortex in the left hemisphere controls movement on the __________ side of the body: a) rostral b) caudal c) contralateral d) ipsilateral

To choose the correct response to this question, you must be familiar with the concept of "contralateral representation." a. Incorrect Several terms are used to describe locations in the nervous system. Rostral means "toward the beak" and is a synonym for anterior. b. Incorrect The term caudal means "toward the tail" and is a synonym for posterior. C. CORRECT Contralateral means "located on the opposite side of the body." Most functions of the body are controlled by the opposite side of the brain - e.g., voluntary motor movements on the right side of the body are controlled by motor centers in the left side of the brain. d. Incorrect Ipsilateral means "located on the same side of the body."

Tourette's syndrome has been most consistently linked to abnormalities in which of the following structures of the brain? a) dentate gyrus b) pons c) basal ganglia d) mammillary bodies

Tourette's syndrome is a tic disorder that involves a combination of motor and vocal tics. a. Incorrect The dentate gyrus is part of the hippocampus and is believed to be involved in memory consolidation. b. Incorrect The pons is a hindbrain structure that relays sensory information between the cerebellum and cortex, mediates reflexes related to breathing, and may play a role in sleep and dreaming. c. CORRECT The basal ganglia are involved in the control of movement and include the corpus striatum, caudate nucleus, putamen, and globus pallidus. Basal ganglia dysfunction has been implicated in a number of disorders including Tourette's syndrome, ADHD, OCD, and schizophrenia. d. Incorrect The mammillary bodies are connected to the hypothalamus, are involved in memory, and may play a role in the anterograde amnesia associated with Korskakoff's syndrome.

A patient with schizophrenia is prescribed a traditional neuroleptic drug. Most likely this patient will first experience an improvement in which of the following symptoms: a) cognitive deficits b) avolition c) dysphoria d) hallucinations

Traditional neuroleptic (antipsychotic) drugs are most effective for the positive symptoms of schizophrenia. a. Incorrect The effectiveness of the traditional neuroleptics for cognitive symptoms has not been consistently demonstrated. In addition, the drugs themselves can produce cognitive disturbances. b. Incorrect Avolition is a negative symptom and is not usually affected by the traditional neuroleptics. c. Incorrect For some patients, the traditional neuroleptics actually contribute to dysphoria. D. CORRECT Hallucinations are positive symptoms and are responsive to the traditional neuroleptics. For a recent review of drug treatments for Schizophrenia, see R. A. Rivas-Vazquez et al., Atypical antipsychotic medications: Pharmacological profiles and psychological implications, Professional Psychology: Research and Practice, 2000, 31(6), 628-648.

Which one of the following is true regarding treatment for premature ejaculation: a) premature ejaculation is one of the more difficult sexual dysfunctions to treat b) treatment usually involves temporary abstinence from sexual contact c) treatment will involve helping the client maintain an erection for at least four minutes d) treatment will probably include the use of the "stop-start" or "squeeze" technique

Treatment for premature ejaculation usually involves either the stop-start or squeeze technique. a. Incorrect Premature ejaculation is actually one of the easiest sexual dysfunctions to treat. b. Incorrect This would more likely be part of the treatment for secondary impotence. c. Incorrect The diagnosis of premature ejaculation is not based on the amount of time a man can sustain an erection, and maintaining an erection for a particular length of time would probably not be a goal in the treatment of this disorder. D. CORRECT The start-stop and squeeze techniques are used to help the male gain control over ejaculation and thereby alleviate premature ejaculation.

Feature integration theory would be most useful when a researcher is investigating: a) focused auditory attention b) focused visual attention c) divided attention d) selective attention

Treisman's feature integration theory is used to explain how we perceive objects as complete entities rather than as meaningless collections of features (e.g., color, texture, size). See A. Treisman, Features and objects: The 14th Bartlett's memorial lecture, Quarterly Journal of Experimental Psychology, 1988, 40A, 201-237.) a. Incorrect Feature integration theory is a theory of focused visual attention. B. CORRECT According to Treisman, focused visual attention is necessary to perceive objects as objects rather than as a collection of unrelated features.

Nausea, stomach cramps, excessive thirstiness, increased frequency of urination, muscle weakness, impaired memory, fine hand tremor, and weight gain are side effects most associated with which of the following? a) lithium b) carbamazepine c) methylphenidate d) fluoxetine

Unfortunately, some of the symptoms included in the question are side effects of more than one of the drugs listed in the answers, which makes this a difficult question. a. CORRECT The symptoms given in the question are potential side effects of lithium, which is used to treat bipolar disorder. b. Incorrect Carbamazepine (Tegretol), an anticonvulsant, is also used to treat bipolar disorder. However, its common side effects include dizziness, drowsiness, nausea and vomiting, headache, and ataxia. c. Incorrect Methylphenidate, a CNS stimulant, is associated with insomnia, reduced appetite, nervousness, tremor, and cardiac arrhythmias. d. Incorrect Nausea, appetite loss, anxiety, tremor, dry mouth, sweating, tiredness or weakness, and disturbances in sexuality are common side effects of fluoxetine and other SSRIs.

Visual agnosia is best conceptualized as: a) visual "not caring" b) visual "not knowing" c) visual repression d) visual blindness

Visual agnosia is the loss of the ability to recognize visual stimuli even though visual acuity is intact. B. CORRECT The term "agnosia" is derived from the Greek word "agnostos," which means "unknown, unknowable, not knowing."

As the result of a brain injury, Walter W. cannot recognize familiar objects by touch. Most likely, Walter has damage to his _______ lobes. a) parietal b) occipital c) temporal d) frontal

Walter is exhibiting somatosensory (tactile) agnosia. a. CORRECT The parietal lobes contain the somatosensory cortex, which processes information from the skin senses (touch, temperature, pressure, etc.). Damage to the parietal lobes may produce somatosensory agnosia which is characterized by a lack of awareness of one side of the body and/or an inability to recognize familiar objects by touch.

Weber's law explains why: a) you perceive objects that are close together as a single stimulus or unity b) you can be heard when you whisper in a very quiet room but must shout to be heard in a very loud, noisy room c) you perform better in the presence of others on simple tasks but worse in the presence of others on difficult or complex tasks d) you feel less pain after banging your elbow when you vigorously rub your elbow or apply heat to it

Weber's law is one of the psychophysical laws that explains the relationship between physical stimuli and their psychological effects. a. Incorrect This is the prediction made by the Gestaltian "law of proximity." b. CORRECT According to Weber's law, the "just-noticeable difference" in a stimulus is proportional to the magnitude of the original stimulus. It explains why a whisper is audible in a quiet room but not in a very noisy room. c. Incorrect This answer describes social facilitation (performing better in the presence of others on simple tasks) and social inhibition (performing worse in the presence of others on complex tasks). d. Incorrect This answer illustrates the prediction made by the gate-control theory of pain.

A person with Wernicke's aphasia has impaired language comprehension and: a) nonfluent and unintelligible speech. b) nonfluent but intelligible speech. c) fluent but unintelligible speech. d) fluent and intelligible speech.

Wernicke's aphasia is characterized by a disruption in the ability to produce and comprehend language. c. CORRECT People with Wernicke's aphasia have fluent speech, but their speech is unintelligible because it contains multiple errors including sound and word substitutions.

Wernicke's area is located in the: a) occipital lobe b) temporal lobe c) frontal lobe d) parietal lobe

Wernicke's area is one of the primary speech areas. a. Incorrect The occipital lobe mediates vision and visual perception. B. CORRECT Wernicke's area is located in the temporal lobe. Damage to this area causes Wernicke's (receptive) aphasia. c. Incorrect Broca's area (the other primary speech area) is located in the frontal lobe. d. Incorrect The parietal lobe contains the somatosensory cortex.

Wernicke's encephalopathy is caused by: a) a lack of blood flow to the brain b) inadequate intake or absorption of thiamine c) renal failure d) prolonged exposure to a solvent, industrial chemical, or other toxin

Wernicke's encephalopathy is characterized by ataxia, abnormal eye movements (opthalmoplegia), and confusion. a. Incorrect Encephalopathy (diffuse brain disease) has several causes, including those listed in all of the answers to this question. However, Wernicke's encephalopathy is due to a thiamine deficiency. B. CORRECT Wernicke's encephalopathy is caused by a thiamine deficiency, often as the result of chronic alcoholism.

__________ syndrome is due to a thiamine deficiency that causes atrophy of neurons in certain areas of the thalamus and the mammillary bodies of the hypothalamus and is usually the result of chronic alcoholism.

Wernicke-Korsakoff

The complete loss of color vision resulting from a lack of functioning cone cells is referred to as: a) color agnosia b) akinetopsia c) prosopagnosia d) achromatopsia

You may have been able to select the correct answer to this question by knowing that "chroma" refers to color. a. Incorrect Color agnosia is the inability to name or discriminate between colors. b. Incorrect Akinetopsia is the inability to perceive moving objects as the result of brain damage. c. Incorrect Prosopagnosia is the inability to recognize familiar faces. D. CORRECT Total color blindness, or complete achromatopsia, is an autosomal recessive disorder that is caused by a lack of functioning of the cone cells.

The thalamus acts as a "relay station" for incoming signals from all of the senses except: a) olfaction. b) gestation. c) vision. d) audition.

a. CORRECT As noted above, incoming signals related to smell (olfaction) are not relayed to higher areas of the brain through the thalamus. Instead, olfactory signals are sent to the olfactory bulb which then forwards the signals to other areas of the brain.

When an individual's alcohol consumption affects the normal functioning of his or her cerebellum, the individual may exhibit which of the following? a) ataxia b) aphagia c) akathisia d) apraxia

a. CORRECT Ataxia involves the loss of ability to coordinate voluntary movements and can be caused by a variety of factors including damage to the cerebellum, the ingestion of certain drugs, and diseases that affect muscle coordination (e.g., cerebral palsy). b. Incorrect Aphagia (also known as dysphagia) refers to difficulty in swallowing or eating. c. Incorrect Akathisia is characterized by motor restlessness, especially in the arms and legs. It is one of the side effects of the neuroleptic drugs. d. Incorrect Apraxia involves an inability to perform skilled movements that is not due to muscle weakness, a loss of motor coordination, sensory loss, or a lack of comprehension or cooperation.

Which of the following neuroimaging techniques would not be useful for studying the functional activity of the brain (e.g., blood flow, glucose metabolism, or oxygen consumption)? a) CT b) SPECT c) fMRI d) PET

a. CORRECT CT (computed tomography) is a structural technique and provides information on brain structure. In contrast, the techniques listed in answers b, c, and d are functional techniques and provide information on the functional activities of the brain.

If you're in an accident that causes damage to your cerebellum, you're most likely to: a) have trouble playing tennis. b) forget what occurred just prior to the accident. c) have trouble identifying the source of familiar odors. d) find it difficult to do simple arithmetic calculations.

a. CORRECT Of the activities listed in the answers, playing tennis is most likely to be adversely affected by damage to the cerebellum.

Jennifer J. always experiences certain colors in response to certain sounds (e.g., the meow of a cat always elicits a reddish-pink color while the chirp of a bird elicits light blue). Jennifer's experience is referred to as: a) synesthesia. b) somesthesis. c) anosognosia. d) alexithymia

a. CORRECT Synesthesia occurs when two or more senses are automatically and involuntarily joined so that stimulation of one sense automatically elicits another sense (e.g., a certain sound elicits a specific color). b. Incorrect Somesthesis refers to the sensory perception associated with the skin, muscles, joints, and viscera and includes touch-pressure, warmth-coolness, and proprioception. c. Incorrect Anosognosia is a lack of awareness of one's own disability or illness. d. Incorrect Alexithymia is an inability to describe one's own feelings.

The research has confirmed that the _________ is responsible for our ability to recall pleasant and unpleasant events better than neutral events. a) amygdala b) medulla oblongata c) substantia nigra d) putamen

a. CORRECT The amygdala is believed to be responsible for emotional memory, which refers to memory for events that evoke strong emotions.

The neurotransmitter acetylcholine: a) is involved in voluntary movements. b) decreases alertness and increases the duration of sleep. c) is implicated in the development of the psychotic disorders. d) is involved in the control of pain.

a. CORRECT The skeletal muscles, which are involved in voluntary movement, are activated by the release of ACh by motor neurons. b. Incorrect This due to the effects of serotonin. c. Incorrect This describes dopamine. d. Incorrect This describes the endorphins.

As defined in DSM-5, postconcussional disorder (syndrome) is characterized by which of the following symptoms? a) diaphoresis, dysphagia, confusion, and tachycardia b) slurred speech, incoordination, nystagmus, and impaired memory c) dysphoria, nausea and vomiting, diarrhea, and insomnia d) headache, vertigo or dizziness, sleep disturbances, and affective lability

a. Incorrect These are characteristic symptoms of neuroleptic malignant syndrome. b. Incorrect These are symptoms of alcohol intoxication. c. Incorrect These are characteristic symptoms of opioid withdrawal. d. CORRECT A diagnosis of postconcussional disorder requires a history of closed head trauma that caused significant cerebral concussion, disturbances in attention or memory, and at least three characteristic symptoms that have persisted for at least three months. The characteristic symptoms include those listed in this answer. Other symptoms are becoming fatigued easily, apathy or reduced spontaneity, and changes in personality.

Damage to the temporal lobe is most likely to result in: a) contralateral neglect. b) apraxia. c) memory loss. d) homonymous hemianopsia

a. Incorrect Contralateral neglect is a possible consequence of parietal lobe damage. b. Incorrect Apraxia is caused by damage to the parietal or frontal lobe. c. CORRECT Damage to the dominant temporal lobe may cause verbal memory loss, while damage to the nondominant temporal lobe may produce nonverbal memory loss. d. Incorrect This refers to loss of one-half of the visual field in each eye and is due to damage to the occipital lobe.

After the image of an object is projected exclusively to Olivia O.'s left visual field (right hemisphere), Olivia is unable to verbally name the object but can pick the object out of a group of objects hidden from view with her left hand. Damage to which of the following would explain this phenomenon? a) visual association cortex b) prefrontal association cortex c) arcuate fasciculus d) corpus callosum

a. Incorrect Damage to the visual association cortex can result in visual agnosia (an inability to recognize a familiar object by sight). Olivia recognizes the object but cannot verbally name it. b. Incorrect The prefrontal association cortex is involved in complex functions related to perception, emotion, memory, and thinking. Damage to this area may produce subtle personality and behavioral changes (e.g., apathy, impulsivity, inappropriate silliness). c. Incorrect The arcuate fasciculus connects the brain's receptive language center (Wernicke's area) with its expressive language center (Broca's area). Damage to the arcuate fasciculus causes conduction aphasia. d. CORRECT The corpus callosum is the major bundle of fibers that connects the right and left hemispheres of the brain, and damage to this structure reduces interhemispheric communication and can produce the behavior described in this question

Parkinson's disease is caused by a progressive degeneration of dopamine-producing cells in the: a) suprachiasmatic nucleus. b) insular cortex. c) substantia nigra. d) entorhinal cortex.

a. Incorrect The suprachiasmatic nucleus (SCN) is located in the hypothalamus and is involved the regulation of the body's circadian rhythms. b. Incorrect The insular cortex is involved in speech and in the processing of autonomic and sensory information. c. CORRECT Knowing the substantia nigra is involved in the control of movement would have helped you identify the correct answer to this question. The degeneration of dopamine-producing cells in the substantia nigra results in the motor symptoms associated with this disorder. d. Incorrect The entorhinal cortex plays a role (in conjunction with the hippocampus) in memory consolidation and is one of the first areas of the brain to be affected by Alzheimer's disease.

The research has most consistently linked which of the following characteristics of the Type A personality pattern to an increased risk for coronary heart disease? a) a sense of time urgency b) hostility and anger c) perfectionism d) competitiveness and ambitiousness

b. CORRECT Most of the research on the Type A personality has found that, in general, it is not predictive of an increased risk for coronary heart disease. However, anger and hostility, which are characteristic of the Type A personality, have been linked to this disease and other stress-related illnesses. Note that some recent research has found that an elevated risk for coronary heart disease is associated with the Type D personality pattern

The onset of puberty in humans occurs when gonadotropin-releasing hormone (GnRH) is secreted by the: a) hippocampus b) hypothalamus c) adrenal glands d) thalamus

b. CORRECT Secretion of gonadotropin-releasing hormone by the hypothalamus stimulates production and release of the gonadotropic hormones by the pituitary gland. These hormones then stimulate the gonads to release the sex hormones which trigger sexual development.

Selye (1956) proposed that which of the following endocrine glands mediate the general adaptation syndrome? a) thyroid and thymus b) adrenal and pituitary c) thyroid and parathyroid d) pancreas and pineal

b. CORRECT Selye proposed that the general adaptation syndrome is mediated primarily by the adrenal and pituitary glands. For example, during the initial alarm stage, the hypothalamus activates the adrenal medulla which increases the production of epinephrine.

In the peripheral nervous system, the neurotransmitter __________ is released into the neuromuscular junction where it causes muscles to contract; and, in the central nervous system, it's involved in the regulation of the sleep-wake cycle and learning and memory.

acetylcholine (ACh)

Many psychoactive drugs produce __________ side effects such as dry mouth, blurred vision, urinary retention, constipation, decreased perspiration, and tachycardia. a) serotoninergic b) cholinergic c) anticholinergic d) dopaminergic

c. CORRECT Anticholinergic side effects result when the amount of acetylcholine (ACh) in the central and peripheral nervous systems is reduced. Anticholinergic side effects are usually more severe in older adults due to the general decrease in ACh production associated with increasing age.

Generally, the __________ branch of the autonomic nervous system activates bodily functions while the __________ branch returns the body to a resting state. a) peripheral; central b) central; peripheral c) sympathetic; parasympathetic d) parasympathetic; sympathetic

c. CORRECT The sympathetic nervous system mediates arousal and expenditure of energy and prepares the body for action, while the parasympathetic nervous system allows the body to increase its store of energy and is active during states of relaxation.

Recent research suggests that the tics associated with Tourette's syndrome are due to excessive sensitivity to the production of which of the following neurotransmitters of the basal ganglia? a) gaba b) glutamate c) dopamine d) cortisol

c. dopamine

The hippocampus is involved more with learning and memory than with emotions and is responsible for __________ declarative memories.

consolidating

The right and left hemispheres of the cerebral cortex are connected by several bundles of fibers, the largest of which is the __________.

corpus collosum

Prolonged stress increases the chance for illness by suppressing the immune system as a result of chronically elevated levels of __________ and other stress hormones.

cortisol

An agonist drug exerts its effects by: a) blocking an endogenous neurotransmitter's access to a receptor site. b) exerting the opposite effect of an endogenous neurotransmitter. c) facilitating the reuptake of an endogenous neurotransmitter. d) mimicking the effect of an endogenous neurotransmitter at a receptor site.

d. CORRECT Agonist drugs exert their effects by mimicking a neurotransmitter and thereby activating or stimulating receptor cells or by facilitating the action of a neurotransmitter at receptor sites.

Research on the Type A behavior pattern has found that __________ is most strongly related to coronary heart disease.

cynical or antagonistic hostility

The hypothalamus is believed to be involved in all of the following functions EXCEPT: a) hunger and thirst. b) sexual arousal. c) sleep-wake cycle. d) vision and audition.

d. CORRECT The hypothalamus mediates a wide range of autonomic, endocrine, and behavioral functions including emotional states, the sleep-wake cycle, hunger and thirst, sexual behavior and reproduction, and temperature. However, it is not involved in vision or audition.

Intense hunger, headaches, anxiety, dizziness, weakness, heart palpitations, and confusion are symptoms of which of the following endocrine disorders? a) hyperthyroidism b) hyperglycemia c) hypothyroidism d) hypoglycemia

d. CORRECT The symptoms described in this question are characteristic of hypoglycemia (low blood sugar), which has several causes including excessive production of insulin by the pancreas.

Because it regulates heart rate, breathing, and other bodily functions, damage to which structures can be fatal? a) reticular activating system b) cerebellum c) substantia nigra d) medulla oblongata

d. medulla oblongata

The symptoms of obsessive-compulsive disorder are most likely to be alleviated by a drug that effects which of the following systems? a) cholinergic b) anticholinergic c) noradrenergic d) serotonergic

d. serotonergic

Hyposecretion of ADH by the pituitary results in __________.

diabetes insipidus

Research using brain imaging techniques has confirmed that the brain is sexually __________ - i.e., that there are sex-related differences in several brain structures including the corpus callosum, hippocampus, and SCN.

dimorphic

Psychophysiological disorders are characterized by physical symptoms that are caused, maintained, and exacerbated by __________ factors

emotional (psychological)

The development of the CNS involves five stages. During the __________ stage, each cell moves to its ultimate destination in the nervous system.

migration

PMS usually develops after __________ and lasts until just before or after menstruation begins.

ovulation

As one ascends the __________, the sex hormones become increasingly less important as determinants of adult sexual behavior.

phylogenetic scale

The midbrain includes the __________, which is vital to consciousness, arousal, and wakefulness.

reticular activating system (RAS)

Because of the increased risk for adverse side effects, a general rule when prescribing drugs for older adults and members of some ethnic minority groups is to __________.

start low and go slow

The __________ mediates the sleep-wake cycle and other circadian rhythms.

subarachiasmatic nucleus (SCN)

Recovery from TBI is moderated by a number of factors but, in general, the greatest amount of recovery occurs during the first __________ months.

three


Related study sets

ARRT The Fetal Genitourinary System - Chapter 29 (Review Questions)

View Set

Shadow Health - Tina Jones, Health History

View Set

Combo with "LabSim Linux+ 10.3" and 1 other

View Set

TestOut Linux Pro practice questions

View Set

Great Gatsby Quote Identification

View Set

AP Psych - Unit 4 Test Review (AP Classroom Questions)

View Set